Day-570 | Daily MCQs | UPSC Prelims | CURRENT DEVELOPMENTS

Day-570

Time limit: 0

Quiz-summary

0 of 5 questions completed

Questions:

  1. 1
  2. 2
  3. 3
  4. 4
  5. 5

Information

DAILY MCQ

You have already completed the quiz before. Hence you can not start it again.

Quiz is loading...

You must sign in or sign up to start the quiz.

You have to finish following quiz, to start this quiz:

Results

0 of 5 questions answered correctly

Your time:

Time has elapsed

You have reached 0 of 0 points, (0)

Categories

  1. Not categorized 0%
  1. 1
  2. 2
  3. 3
  4. 4
  5. 5
  1. Answered
  2. Review
  1. Question 1 of 5
    1. Question

    1. Consider the following statements:
    Statement I: Sodium-ion batteries are environmentally more friendly than Lithium-ion batteries.
    Statement II: Sodium is a more abundant as well as a widely distributed element than Lithium.
    Which one of the following is correct in respect of the above statements?

    Correct

    Answer: A
    Explanation
    Sodium-ion batteries are a type of rechargeable battery just like the commonly known lithium-ion battery, and their overall working principle, set-up and materials are like lithium-ion batteries. However, there are certain benefits of replacing Lithium with Sodium which have attracted the attention of researchers for increasing its scalability and commercialization.
    Statement 1 is correct: Sodium-ion batteries have a lower environmental impact than lithium-ion batteries due to the availability of sodium as a more abundant and widely distributed resource. Additionally, the manufacturing process of sodium-ion batteries may be less energy-intensive than lithium-ion batteries.
    Statement 2 is correct and is the correct explanation of statement 1: Sodium is one of the most widely distributed elements. It can be extracted from sea water at a relatively less energy intensive process. Since it is widely distributed so it brings down the environmental impact of transportation of raw materials from few places. Also, decentralized generation systems and storage systems can be created which have a lower environmental impact than transmission through various lines.

    Incorrect

    Answer: A
    Explanation
    Sodium-ion batteries are a type of rechargeable battery just like the commonly known lithium-ion battery, and their overall working principle, set-up and materials are like lithium-ion batteries. However, there are certain benefits of replacing Lithium with Sodium which have attracted the attention of researchers for increasing its scalability and commercialization.
    Statement 1 is correct: Sodium-ion batteries have a lower environmental impact than lithium-ion batteries due to the availability of sodium as a more abundant and widely distributed resource. Additionally, the manufacturing process of sodium-ion batteries may be less energy-intensive than lithium-ion batteries.
    Statement 2 is correct and is the correct explanation of statement 1: Sodium is one of the most widely distributed elements. It can be extracted from sea water at a relatively less energy intensive process. Since it is widely distributed so it brings down the environmental impact of transportation of raw materials from few places. Also, decentralized generation systems and storage systems can be created which have a lower environmental impact than transmission through various lines.

  2. Question 2 of 5
    2. Question

    2. Dark Pattern, which is often heard in the news, is related to which of the following?

    Correct

    Answer: D
    Explanation
    • A dark pattern or deceptive design pattern is a user interface that has been carefully crafted to trick users into doing unnecessary and unwanted things, such as buying overpriced insurance with their purchase or signing up for recurring bills.
    • These are done by prompting users into agreeing to certain conditions or clicking a few links. The user interface is made clumsy and deceptive like making cancellation button very small and hard to find, pop-ups which are hard to ignore, deliberately writing important things in smaller fonts and at unusual places on websites.
    • Big Tech firms such as Apple, Amazon, Facebook, Microsoft, and Google use dark or deceptive patterns to mold user experience to their advantage.
    • It is to be noted that use of dark pattern is not illegal in India currently. However, misleading ads and unnecessary promotions are challenged under the Consumer Protection Act.
    Note: Option C explains Dark Web.

    Incorrect

    Answer: D
    Explanation
    • A dark pattern or deceptive design pattern is a user interface that has been carefully crafted to trick users into doing unnecessary and unwanted things, such as buying overpriced insurance with their purchase or signing up for recurring bills.
    • These are done by prompting users into agreeing to certain conditions or clicking a few links. The user interface is made clumsy and deceptive like making cancellation button very small and hard to find, pop-ups which are hard to ignore, deliberately writing important things in smaller fonts and at unusual places on websites.
    • Big Tech firms such as Apple, Amazon, Facebook, Microsoft, and Google use dark or deceptive patterns to mold user experience to their advantage.
    • It is to be noted that use of dark pattern is not illegal in India currently. However, misleading ads and unnecessary promotions are challenged under the Consumer Protection Act.
    Note: Option C explains Dark Web.

  3. Question 3 of 5
    3. Question

    3. Consider the following statements:
    Statement I: Ballistic missiles have more payload carrying capacity than cruise missiles.
    Statement II: Ballistic missiles require less fuel for the same size of payload.
    Which one of the following is correct in respect of the above statements?

    Correct

    Answer: A
    Explanation
    There are basically two missile systems in operation- ballistic missile system and cruise missile system. Ballistic missiles use projectile motion to attack targets whereas cruise missiles are guided missiles that remain in the atmosphere and fly the major portion of their flight path at an approximately constant speed.
    • Statement 1 is correct: Ballistic missiles have more payload carrying capacity as they are fuel efficient.
    • Statement 2 is correct and is the correct explanation of statement 1: Since ballistic missile use projectile motion and is assisted by gravity. They carry fuel for only the initial lift off. They are not required to carry extra fuel to maintain speed as is required in cruise missiles. This reduces the mass of the ballistic missile for similar range as compared to cruise missiles. Thus, they can carry extra payload or can reach larger distance. That is why all intercontinental missiles are ballistic in design.

    Incorrect

    Answer: A
    Explanation
    There are basically two missile systems in operation- ballistic missile system and cruise missile system. Ballistic missiles use projectile motion to attack targets whereas cruise missiles are guided missiles that remain in the atmosphere and fly the major portion of their flight path at an approximately constant speed.
    • Statement 1 is correct: Ballistic missiles have more payload carrying capacity as they are fuel efficient.
    • Statement 2 is correct and is the correct explanation of statement 1: Since ballistic missile use projectile motion and is assisted by gravity. They carry fuel for only the initial lift off. They are not required to carry extra fuel to maintain speed as is required in cruise missiles. This reduces the mass of the ballistic missile for similar range as compared to cruise missiles. Thus, they can carry extra payload or can reach larger distance. That is why all intercontinental missiles are ballistic in design.

  4. Question 4 of 5
    4. Question

    4. ‘Tokamak’, a technology often heard in the news, is related to which of the following scientific developments?

    Correct

    Answer: C
    Explanation
    A tokamak is a device which uses a powerful magnetic field to confine plasma in the shape of a torus. The tokamak is one of several types of magnetic confinement devices being developed to produce controlled thermonuclear fusion power. The word “tokamak” is derived from a Russian acronym meaning “toroidal chamber with magnetic coils”.
    When heated to fusion temperatures, the electrons in atoms dissociate, resulting in a fluid of nuclei and electrons known as plasma. Unlike electrically neutral atoms, a plasma is electrically conductive, and can, therefore, be manipulated by electrical or magnetic fields.

    Incorrect

    Answer: C
    Explanation
    A tokamak is a device which uses a powerful magnetic field to confine plasma in the shape of a torus. The tokamak is one of several types of magnetic confinement devices being developed to produce controlled thermonuclear fusion power. The word “tokamak” is derived from a Russian acronym meaning “toroidal chamber with magnetic coils”.
    When heated to fusion temperatures, the electrons in atoms dissociate, resulting in a fluid of nuclei and electrons known as plasma. Unlike electrically neutral atoms, a plasma is electrically conductive, and can, therefore, be manipulated by electrical or magnetic fields.

  5. Question 5 of 5
    5. Question

    5. With reference to the comparison between 5G and 4G wireless communication technology, consider the following statements:

    1. 5G works at a higher frequency, but a lower bandwidth as compared to the 4G technology.

    2. 5G ensures very high speed but consumes more energy per unit data transfer than the 4G technology.

    3. While 4G supports only half duplex mode, 5G supports both half as well as full duplex mode of communication.

    How many of the above statements are correct?

    Correct

    Answer: D

    Explanation

    5G is the fifth generation of cellular network technology, succeeding 4G LTE. Unlike 3G and 4G before it, 5G isn’t simply an incremental technological advancement but a true leapfrog technology. One of the biggest differences between 5G and its predecessors is speed. 5G offers theoretical speeds of up to 10 gigabits per second, which is more than 30 times faster than the 300Mbps theoretical maximum speed of 4G LTE-Advanced. This technology also uses higher frequency bands than previous generations and is deployable in low-band, mid-band or high-band millimetre-waves from 24 GHz up to 66 GHz. This aspect allows massively faster data transfer speeds and greater bandwidth capacity.

    • Statement 1 is incorrect: 5G can operate on a new, high-frequency spectrum called a millimetre wave (mm wave.) This operates on wavelengths between 30 GHz and 300 GHz, in comparison to the 4G LTE’s wavelengths of under 6 GHz. Higher frequency range increases the bandwidth of operation. A bandwidth is the measure of capacity of the network to onboard multiple devices at the same time. A higher bandwidth means more devices can be connected at the same time.

    • Statement 2 is incorrect: 5G is more energy efficient than 4G, starting with cell base designs that allow carriers to reduce their energy consumption. More importantly, mobile devices operating in 5G use less energy as well, providing extended battery performance and life span. It’s estimated that energy consumption per bit with 5G is just 10 percent of what 4G requires. Thus, it is a win-win situation as 5G technology ensures high speed at a lower energy input.

    • Statement 3 is incorrect: Both 5G as well as 4G offer full duplex mode of communication. There are 3 modes of transmission of data:

    1. Simplex mode: It is a mode of transmission in which the flow of data is unidirectional. It means that the participant can do only one task at a time in a single direction. The receiver can only receive data and not send it (and vice versa).

    2. Half Duplex mode: It is a mode of transmission in which the flow of data is bi-directional but only in one direction at a time. It means that the receiver can receive as well as send data but can perform only one of these tasks at a time.

    3. Full Duplex mode: It is a mode of transmission in which the flow of data is bi-directional. But here, the devices can send as well as receive data simultaneously at the same time. It means that the receiver can receive as well as send data at the very same time without any interference.

    Incorrect

    Answer: D

    Explanation

    5G is the fifth generation of cellular network technology, succeeding 4G LTE. Unlike 3G and 4G before it, 5G isn’t simply an incremental technological advancement but a true leapfrog technology. One of the biggest differences between 5G and its predecessors is speed. 5G offers theoretical speeds of up to 10 gigabits per second, which is more than 30 times faster than the 300Mbps theoretical maximum speed of 4G LTE-Advanced. This technology also uses higher frequency bands than previous generations and is deployable in low-band, mid-band or high-band millimetre-waves from 24 GHz up to 66 GHz. This aspect allows massively faster data transfer speeds and greater bandwidth capacity.

    • Statement 1 is incorrect: 5G can operate on a new, high-frequency spectrum called a millimetre wave (mm wave.) This operates on wavelengths between 30 GHz and 300 GHz, in comparison to the 4G LTE’s wavelengths of under 6 GHz. Higher frequency range increases the bandwidth of operation. A bandwidth is the measure of capacity of the network to onboard multiple devices at the same time. A higher bandwidth means more devices can be connected at the same time.

    • Statement 2 is incorrect: 5G is more energy efficient than 4G, starting with cell base designs that allow carriers to reduce their energy consumption. More importantly, mobile devices operating in 5G use less energy as well, providing extended battery performance and life span. It’s estimated that energy consumption per bit with 5G is just 10 percent of what 4G requires. Thus, it is a win-win situation as 5G technology ensures high speed at a lower energy input.

    • Statement 3 is incorrect: Both 5G as well as 4G offer full duplex mode of communication. There are 3 modes of transmission of data:

    1. Simplex mode: It is a mode of transmission in which the flow of data is unidirectional. It means that the participant can do only one task at a time in a single direction. The receiver can only receive data and not send it (and vice versa).

    2. Half Duplex mode: It is a mode of transmission in which the flow of data is bi-directional but only in one direction at a time. It means that the receiver can receive as well as send data but can perform only one of these tasks at a time.

    3. Full Duplex mode: It is a mode of transmission in which the flow of data is bi-directional. But here, the devices can send as well as receive data simultaneously at the same time. It means that the receiver can receive as well as send data at the very same time without any interference.

window.wpAdvQuizInitList = window.wpAdvQuizInitList || []; window.wpAdvQuizInitList.push({ id: '#wpAdvQuiz_610', init: { quizId: 610, mode: 0, globalPoints: 10, timelimit: 0, resultsGrade: [0], bo: 0, qpp: 0, catPoints: [10], formPos: 0, lbn: "Finish quiz", json: {"2839":{"type":"single","id":2839,"catId":0,"points":2,"correct":[1,0,0,0]},"2840":{"type":"single","id":2840,"catId":0,"points":2,"correct":[0,0,0,1]},"2841":{"type":"single","id":2841,"catId":0,"points":2,"correct":[1,0,0,0]},"2842":{"type":"single","id":2842,"catId":0,"points":2,"correct":[0,0,1,0]},"2843":{"type":"single","id":2843,"catId":0,"points":2,"correct":[0,0,0,1]}} } });



Day-569 | Daily MCQs | UPSC Prelims | ENVIRONMENT AND ECOLOGY

Day-569

Time limit: 0

Quiz-summary

0 of 5 questions completed

Questions:

  1. 1
  2. 2
  3. 3
  4. 4
  5. 5

Information

DAILY MCQ

You have already completed the quiz before. Hence you can not start it again.

Quiz is loading...

You must sign in or sign up to start the quiz.

You have to finish following quiz, to start this quiz:

Results

0 of 5 questions answered correctly

Your time:

Time has elapsed

You have reached 0 of 0 points, (0)

Categories

  1. Not categorized 0%
  1. 1
  2. 2
  3. 3
  4. 4
  5. 5
  1. Answered
  2. Review
  1. Question 1 of 5
    1. Question

    1. Consider the following statements about the practice of mulching in agriculture:
    1. It improves the soil quality by increasing the compaction and water-retention capacity of soil.
    2. It is always effective in reducing the soil surface temperature.
    Which of the statements given above are correct?

    Correct

    Answer. D
    Explanation:
    Mulches could be both organic (using straw, husk, grass, cover crops, saw dust, compost and manures) and synthetic (polyethylene plastic mulch).
    • Statement 1 is incorrect: Mulching is beneficial for the soil health as it increases the water-retention capacity of the soil and decreases the soil compaction. Studies have shown that mulches can potentially reduce weed infestation and soil moisture losses through evaporation. They enhance the percolation and retention rate of soil. Straw mulch has been noticed to reduce evaporation losses of moisture from soil by up to 35%.
    • Organic mulches act as sponge which helps retain irrigation and rainfall water in soil. The straw mulches can reduce run off by 43%.
    • The problem of soil compaction in agriculture due to use of heavy machinery or implements is becoming severe, which can be addressed by addition of organic mulch materials such as bark. Also, the decomposition of organic mulches provides additional benefit of nutrient enrichment in the soil. This improves the fertility of the soil.
    • Statement 2 is incorrect: Since mulches cover the soil surface, in general, they help in controlling the temperature of the soil. However, the type of materials used in mulching decide the effectiveness of mulches in controlling the soil temperature. Studies indicate that mulches can reduce soil temperature by 10 degrees Celsius in hot and dry conditions. But the mulches comprising cobbles can increase the overall soil temperature as compared to living mulches due to absorption of solar radiation.

    Incorrect

    Answer. D
    Explanation:
    Mulches could be both organic (using straw, husk, grass, cover crops, saw dust, compost and manures) and synthetic (polyethylene plastic mulch).
    • Statement 1 is incorrect: Mulching is beneficial for the soil health as it increases the water-retention capacity of the soil and decreases the soil compaction. Studies have shown that mulches can potentially reduce weed infestation and soil moisture losses through evaporation. They enhance the percolation and retention rate of soil. Straw mulch has been noticed to reduce evaporation losses of moisture from soil by up to 35%.
    • Organic mulches act as sponge which helps retain irrigation and rainfall water in soil. The straw mulches can reduce run off by 43%.
    • The problem of soil compaction in agriculture due to use of heavy machinery or implements is becoming severe, which can be addressed by addition of organic mulch materials such as bark. Also, the decomposition of organic mulches provides additional benefit of nutrient enrichment in the soil. This improves the fertility of the soil.
    • Statement 2 is incorrect: Since mulches cover the soil surface, in general, they help in controlling the temperature of the soil. However, the type of materials used in mulching decide the effectiveness of mulches in controlling the soil temperature. Studies indicate that mulches can reduce soil temperature by 10 degrees Celsius in hot and dry conditions. But the mulches comprising cobbles can increase the overall soil temperature as compared to living mulches due to absorption of solar radiation.

  2. Question 2 of 5
    2. Question

    2. Which of the following marine species is known for feeding by knocking off its prey from the branch of a terrestrial plant using a powerful jet of water?

    Correct

    Answer. A
    Explanation:
    • Most archerfish species live in freshwater habitats, including rivers, streams, and pools. A few species are euryhaline, which means that they can live in both freshwater and brackish waters. They are generally found in the lowlands of tropical countries like India, Sri Lanka, Bangladesh, and Southeast Asia. They also live in Northern Australia and Melanesia.
    • Uniqueness: Archerfish are known for their ability to shoot water from their mouths at a target above the water’s surface. They do this by dilating and contracting their gill covers, which are controlled by a special lateral line organ. The archerfish may also move its tail back and forth to aim itself after its target.
    • In addition, Archerfish have very sharp eyesight, which is a factor for their ability to shoot the water with great accuracy.

    Incorrect

    Answer. A
    Explanation:
    • Most archerfish species live in freshwater habitats, including rivers, streams, and pools. A few species are euryhaline, which means that they can live in both freshwater and brackish waters. They are generally found in the lowlands of tropical countries like India, Sri Lanka, Bangladesh, and Southeast Asia. They also live in Northern Australia and Melanesia.
    • Uniqueness: Archerfish are known for their ability to shoot water from their mouths at a target above the water’s surface. They do this by dilating and contracting their gill covers, which are controlled by a special lateral line organ. The archerfish may also move its tail back and forth to aim itself after its target.
    • In addition, Archerfish have very sharp eyesight, which is a factor for their ability to shoot the water with great accuracy.

  3. Question 3 of 5
    3. Question

    3. Consider the following species:
    1. Butterflies and turtle
    2. Clownfish and sea anemone
    3. Aphids and ants
    How many of the above-mentioned species share a symbiotic relationship with each other?

    Correct

    Answer. C
    Explanation:
    Options 1, 2 and 3 are correct.
    • Butterflies and turtle: While butterfly drinks turtle tears to obtain nutrients like sodium which it needs for egg production and metabolism, this activity benefits the turtle by cleaning its eyes. It also helps turtle to get rid of the excess salt from its body. The behaviour is called ‘lachryphagy’.
    • Clownfish and sea anemone: Sea anemones are marine predatory animals related to jellies and corals. They provide the clownfish with protection and shelter, while the clownfish provides the anemone nutrients in the form of waste while also scaring off potential predator fish.
    • Sea anemones live attached to the surface of coral reefs. They trap their prey with stinging cells called nematocysts, which are located on their tentacles. Nematocysts release toxins when a small animal contacts an anemone’s tentacle. This paralyzes the stung animal, allowing the anemone to easily bring the animal into its mouth for ingestion. While other fish succumb to these toxic stings, clownfish secrete a substance in the mucus covering their bodies that suppresses the firing of nematocysts. This allows the clownfish to swim comfortably between the tentacles of anemones, creating a protected environment in which potential predators are killed off by anemone stings. This clearly benefits the clownfish, but how about the sea anemones? The brightly colored clownfish attract other fish looking for a meal. These unsuspecting would-be predators are then caught and eaten by the anemones.
    • Aphids and ants: Aphids are sucking insects that are common on both outdoor and indoor plants. They feed on the sap of plants and secrete a substance called honeydew. This sticky resin is a favourite food of ants, who actually “milk” the aphids for it by stroking their abdomen.
    • The unique relationship between these two organisms provides protection for the aphids and food for the ants. Ants protect the aphids from predators, such as lacewings and ladybugs.

    Incorrect

    Answer. C
    Explanation:
    Options 1, 2 and 3 are correct.
    • Butterflies and turtle: While butterfly drinks turtle tears to obtain nutrients like sodium which it needs for egg production and metabolism, this activity benefits the turtle by cleaning its eyes. It also helps turtle to get rid of the excess salt from its body. The behaviour is called ‘lachryphagy’.
    • Clownfish and sea anemone: Sea anemones are marine predatory animals related to jellies and corals. They provide the clownfish with protection and shelter, while the clownfish provides the anemone nutrients in the form of waste while also scaring off potential predator fish.
    • Sea anemones live attached to the surface of coral reefs. They trap their prey with stinging cells called nematocysts, which are located on their tentacles. Nematocysts release toxins when a small animal contacts an anemone’s tentacle. This paralyzes the stung animal, allowing the anemone to easily bring the animal into its mouth for ingestion. While other fish succumb to these toxic stings, clownfish secrete a substance in the mucus covering their bodies that suppresses the firing of nematocysts. This allows the clownfish to swim comfortably between the tentacles of anemones, creating a protected environment in which potential predators are killed off by anemone stings. This clearly benefits the clownfish, but how about the sea anemones? The brightly colored clownfish attract other fish looking for a meal. These unsuspecting would-be predators are then caught and eaten by the anemones.
    • Aphids and ants: Aphids are sucking insects that are common on both outdoor and indoor plants. They feed on the sap of plants and secrete a substance called honeydew. This sticky resin is a favourite food of ants, who actually “milk” the aphids for it by stroking their abdomen.
    • The unique relationship between these two organisms provides protection for the aphids and food for the ants. Ants protect the aphids from predators, such as lacewings and ladybugs.

  4. Question 4 of 5
    4. Question

    4. Consider the following statements:
    Statement I: The use of Green Methanol reduces the emission of the oxides of sulphur and carbon.
    Statement II: It is produced during the process of Direct Air Capture and Green Hydrogen Production.
    Which one of the following is correct in respect of the above statements?

    Correct

    Answer: A
    Explanation:
    • Statement 1 is correct: Compared to conventional fuels, green methanol can reduce carbon dioxide emissions by 60-95%, reduce nitrogen oxide emissions by 60-80%, and almost completely eliminate sulphur oxide and particulate matter emissions, according to the Methanol Institute.
    • Statement 2 is correct: Green methanol is obtained by using CO2 captured from renewable sources (bioenergy with carbon capture and storage [BECCS] and direct air capture [DAC]) and green hydrogen, i.e., hydrogen produced with renewable electricity.
    Additional information
    • Renewable methanol can be produced from a variety of sustainable feedstocks, such as biomass, waste or CO2 and hydrogen.
    • It is a versatile fuel that can be used in internal combustion engines, and in hybrid and fuel cell vehicles and vessels. It is compatible with existing distribution infrastructure and can be blended with conventional fuels.
    • It is a liquid at ambient temperature and pressures.
    • The production of green methanol also offers a range of environmental benefits. It can be blended with traditional gasoline or diesel fuel, offering a seamless transition for vehicle owners and minimizing the need for infrastructure upgrades. This makes it an attractive option for countries and industries looking to reduce their carbon footprint without disrupting their current operations.
    • In the maritime sector, methanol is being considered as a potential alternative to traditional bunker fuels. With global shipping responsible for 90% of world trade and 3-4% of greenhouse gas emissions, the sector is under increasing pressure to decarbonize.
    • It can be used not only as a transportation fuel but also as a feedstock for the production of other valuable chemicals and materials such as formaldehyde, acetic acid and plastics.

    Incorrect

    Answer: A
    Explanation:
    • Statement 1 is correct: Compared to conventional fuels, green methanol can reduce carbon dioxide emissions by 60-95%, reduce nitrogen oxide emissions by 60-80%, and almost completely eliminate sulphur oxide and particulate matter emissions, according to the Methanol Institute.
    • Statement 2 is correct: Green methanol is obtained by using CO2 captured from renewable sources (bioenergy with carbon capture and storage [BECCS] and direct air capture [DAC]) and green hydrogen, i.e., hydrogen produced with renewable electricity.
    Additional information
    • Renewable methanol can be produced from a variety of sustainable feedstocks, such as biomass, waste or CO2 and hydrogen.
    • It is a versatile fuel that can be used in internal combustion engines, and in hybrid and fuel cell vehicles and vessels. It is compatible with existing distribution infrastructure and can be blended with conventional fuels.
    • It is a liquid at ambient temperature and pressures.
    • The production of green methanol also offers a range of environmental benefits. It can be blended with traditional gasoline or diesel fuel, offering a seamless transition for vehicle owners and minimizing the need for infrastructure upgrades. This makes it an attractive option for countries and industries looking to reduce their carbon footprint without disrupting their current operations.
    • In the maritime sector, methanol is being considered as a potential alternative to traditional bunker fuels. With global shipping responsible for 90% of world trade and 3-4% of greenhouse gas emissions, the sector is under increasing pressure to decarbonize.
    • It can be used not only as a transportation fuel but also as a feedstock for the production of other valuable chemicals and materials such as formaldehyde, acetic acid and plastics.

  5. Question 5 of 5
    5. Question

    5. Consider the following:
    1. Plastic products
    2. Electrical and electronic goods
    3. Food items
    4. Batteries
    How many of the above products are under the ambit of Eco-Mark scheme of Bureau of Indian Standards (BIS)?

    Correct

    Answer. D
    Explanation:
    Eco Mark is a certification scheme for environment-friendly products which is administered by the Bureau of Indian Standards (BIS). It was first started in 1991. It is a voluntary non-binding scheme which labels consumer products as environment-friendly on the basis of certain environmental and quality parameters.
    Any product which has less potential for pollution during its entire lifecycle (raw material, manufacturing, use and disposal) could be considered ‘environment-friendly’.
    Logo- Earthen pot, which signifies that renewable resource like clay should be used so that it generates little or no hazardous waste and also consumes less energy in its making.
    The BIS implements the Eco Mark scheme under the BIS Act,1986. It is awarded for minimum one year, after which it is re-assessed for renewal of license. The BIS can withdraw the certification at any time if any misleading information is found or if nay criterion is changed related to technology.
    The Central Pollution Control Board (CPCB) administers the Ecomark Scheme in partnership with Bureau of Indian Standards (BIS), which is the national body for standards and certification.
    The Eco Mark scheme is applicable for the following goods, as specified by the BIS:
    • Soaps and detergents
    • Paints
    • Food items
    • Lubricating oil
    • Packing/packaging material
    • Architectural paints and powder coatings
    • Batteries
    • Electrical and electronic goods
    • Food additives
    • Wood substitutes
    • Cosmetics
    • Aerosols and propellants
    • Plastic products
    • Textiles
    • Fire extinguishers
    • Leather
    • Coir and coir products

    Incorrect

    Answer. D
    Explanation:
    Eco Mark is a certification scheme for environment-friendly products which is administered by the Bureau of Indian Standards (BIS). It was first started in 1991. It is a voluntary non-binding scheme which labels consumer products as environment-friendly on the basis of certain environmental and quality parameters.
    Any product which has less potential for pollution during its entire lifecycle (raw material, manufacturing, use and disposal) could be considered ‘environment-friendly’.
    Logo- Earthen pot, which signifies that renewable resource like clay should be used so that it generates little or no hazardous waste and also consumes less energy in its making.
    The BIS implements the Eco Mark scheme under the BIS Act,1986. It is awarded for minimum one year, after which it is re-assessed for renewal of license. The BIS can withdraw the certification at any time if any misleading information is found or if nay criterion is changed related to technology.
    The Central Pollution Control Board (CPCB) administers the Ecomark Scheme in partnership with Bureau of Indian Standards (BIS), which is the national body for standards and certification.
    The Eco Mark scheme is applicable for the following goods, as specified by the BIS:
    • Soaps and detergents
    • Paints
    • Food items
    • Lubricating oil
    • Packing/packaging material
    • Architectural paints and powder coatings
    • Batteries
    • Electrical and electronic goods
    • Food additives
    • Wood substitutes
    • Cosmetics
    • Aerosols and propellants
    • Plastic products
    • Textiles
    • Fire extinguishers
    • Leather
    • Coir and coir products

window.wpAdvQuizInitList = window.wpAdvQuizInitList || []; window.wpAdvQuizInitList.push({ id: '#wpAdvQuiz_609', init: { quizId: 609, mode: 0, globalPoints: 10, timelimit: 0, resultsGrade: [0], bo: 0, qpp: 0, catPoints: [10], formPos: 0, lbn: "Finish quiz", json: {"2834":{"type":"single","id":2834,"catId":0,"points":2,"correct":[0,0,0,1]},"2835":{"type":"single","id":2835,"catId":0,"points":2,"correct":[1,0,0,0]},"2836":{"type":"single","id":2836,"catId":0,"points":2,"correct":[0,0,1,0]},"2837":{"type":"single","id":2837,"catId":0,"points":2,"correct":[1,0,0,0]},"2838":{"type":"single","id":2838,"catId":0,"points":2,"correct":[0,0,0,1]}} } });




Day-568 | Daily MCQs | UPSC Prelims | ECONOMICS

Day-568

Time limit: 0

Quiz-summary

0 of 5 questions completed

Questions:

  1. 1
  2. 2
  3. 3
  4. 4
  5. 5

Information

DAILY MCQ

You have already completed the quiz before. Hence you can not start it again.

Quiz is loading...

You must sign in or sign up to start the quiz.

You have to finish following quiz, to start this quiz:

Results

0 of 5 questions answered correctly

Your time:

Time has elapsed

You have reached 0 of 0 points, (0)

Categories

  1. Not categorized 0%
  1. 1
  2. 2
  3. 3
  4. 4
  5. 5
  1. Answered
  2. Review
  1. Question 1 of 5
    1. Question

    1. With reference to PM PRANAM scheme, consider the following statements:
    1. The scheme aims to reduce the use of chemical fertilisers and promote alternative fertilizers.
    2. The government has discontinued urea subsidy under the scheme.
    3. It is a central sector scheme.
    How many of the above statements are correct?

    Correct

    Answer: B
    Explanation:
    Statement 1 is correct: PM Programme for Restoration, Awareness, Nourishment and Amelioration of Mother Earth” (PM-PRANAM) scheme was announced in the Budget 2023-24 with the objective to incentivise the states and union territories to promote usage of alternative fertilizers and balanced use of chemical fertilizers.
    Statement 2 is incorrect: The government hasn’t discontinued urea subsidy under the programme. The government decided to continue urea subsidy for three years ending March 2025, with an outlay of Rs 3.68 lakh crore.
    • An outlay of Rs 1451 crore subsidy has been approved to promote organic manure.
    Statement 3 is correct: The scheme is fully funded by the central government, ie, a central scheme.
    How the scheme will work:
    • 50% of the fertilizer subsidy saved by a State/UT in a particular financial year by way of reduction in consumption of chemical fertilizers (Urea, DAP, NPK, MOP) compared to previous 3 years’ average consumption, will be passed on to that State/UT as Grant.
    • To support marketing of organic fertilizers i.e. Fermented Organic Manure (FOM) / Liquid FOM / Phosphate Rich Organic Manure (PROM) produced as a by-product from Biogas Plants / Compressed Bio Gas (CBG) Plants set up under Gobardhan initiative 1500 per metric ton as MDA scheme is included.
    • These organic fertilizers will be available to the farmers at affordable prices.
    • Sulfur Coated Urea (Urea Gold) has been introduced to overcome the sulfur deficiency in the soil and reduce the input cost of the farmers.

    Incorrect

    Answer: B
    Explanation:
    Statement 1 is correct: PM Programme for Restoration, Awareness, Nourishment and Amelioration of Mother Earth” (PM-PRANAM) scheme was announced in the Budget 2023-24 with the objective to incentivise the states and union territories to promote usage of alternative fertilizers and balanced use of chemical fertilizers.
    Statement 2 is incorrect: The government hasn’t discontinued urea subsidy under the programme. The government decided to continue urea subsidy for three years ending March 2025, with an outlay of Rs 3.68 lakh crore.
    • An outlay of Rs 1451 crore subsidy has been approved to promote organic manure.
    Statement 3 is correct: The scheme is fully funded by the central government, ie, a central scheme.
    How the scheme will work:
    • 50% of the fertilizer subsidy saved by a State/UT in a particular financial year by way of reduction in consumption of chemical fertilizers (Urea, DAP, NPK, MOP) compared to previous 3 years’ average consumption, will be passed on to that State/UT as Grant.
    • To support marketing of organic fertilizers i.e. Fermented Organic Manure (FOM) / Liquid FOM / Phosphate Rich Organic Manure (PROM) produced as a by-product from Biogas Plants / Compressed Bio Gas (CBG) Plants set up under Gobardhan initiative 1500 per metric ton as MDA scheme is included.
    • These organic fertilizers will be available to the farmers at affordable prices.
    • Sulfur Coated Urea (Urea Gold) has been introduced to overcome the sulfur deficiency in the soil and reduce the input cost of the farmers.

  2. Question 2 of 5
    2. Question

    2. Consider the following statements:
    Statement I: Capital-output is the ratio of the amount of capital needed to produce one unit of output.
    Statement II: A high capital-output ratio indicates high productivity of capital.
    Which one of the following is correct in respect of the above statements?

    Correct

    Answer: C
    Explanation:
    Statement I is correct and statement II is incorrect.
    Statement I is correct: The concept of capital-output ratio expresses the relationship between the value of capital invested and the value of output.
    • For example, suppose that investment in an economy is 32% (of GDP), and the economic growth corresponding to this level of investment is 8%.
    • Here, a Rs 32 investment produces an output of Rs 8. Capital output ratio is 32/8 or 4. In other words, to produce one unit of output, 4 unit of capital is needed.
    Relevance of capital-output ratio in economic planning
    Suppose the government targets an economic growth of 9% for next year. planners know that the capital output ratio in India is 4. Here, to realize 9% growth, investment should be increased to 36% (9 x4).
    Statement II is incorrect: A lower capital-output ratio shows that only low level of investment is needed to produce a given growth rate in the economy. This is considered as a desirable situation. Lower capital output ratio shows that capital is very productive or efficient.

    Incorrect

    Answer: C
    Explanation:
    Statement I is correct and statement II is incorrect.
    Statement I is correct: The concept of capital-output ratio expresses the relationship between the value of capital invested and the value of output.
    • For example, suppose that investment in an economy is 32% (of GDP), and the economic growth corresponding to this level of investment is 8%.
    • Here, a Rs 32 investment produces an output of Rs 8. Capital output ratio is 32/8 or 4. In other words, to produce one unit of output, 4 unit of capital is needed.
    Relevance of capital-output ratio in economic planning
    Suppose the government targets an economic growth of 9% for next year. planners know that the capital output ratio in India is 4. Here, to realize 9% growth, investment should be increased to 36% (9 x4).
    Statement II is incorrect: A lower capital-output ratio shows that only low level of investment is needed to produce a given growth rate in the economy. This is considered as a desirable situation. Lower capital output ratio shows that capital is very productive or efficient.

  3. Question 3 of 5
    3. Question

    3. Consider the following statements:
    1. India is one of the world’s largest importers of gold.
    2. Gold reserves form the largest part of India’s foreign exchange reserves.
    Which of the above given statements is/are correct?

    Correct

    Answer: A
    Explanation:
    Statement 1 is correct. India is one of the world’s largest importers of gold. High gold imports affect the trade deficit of the country. India’s merchandise trade deficit rose to a record high in October, propelled by a 95% increase in gold imports. In July this year, the government had curbed import of plain gold jewellery. It was meant to check the move of importers over the last few months, who have been using a policy flaw to source plain gold jewellery from Indonesia without paying any import taxes. The government suspected re-routing of gold from Indonesia to India via FTA.
    Statement 2 is incorrect: The Reserve Bank of India is the custodian of foreign exchange reserves.
    The components of foreign exchange reserves include:
    1) Foreign Currency Assets
    2) Gold reserves
    3) Special Drawing Rights (an international reserve asset created by the International Monetary Fund (IMF) to supplement the official reserves of the member countries)
    4) Reserve Tranche Position in the IMF
    Currently, Foreign Currency Assets are the largest component of foreign exchange reserves of the country, followed by gold.

    Incorrect

    Answer: A
    Explanation:
    Statement 1 is correct. India is one of the world’s largest importers of gold. High gold imports affect the trade deficit of the country. India’s merchandise trade deficit rose to a record high in October, propelled by a 95% increase in gold imports. In July this year, the government had curbed import of plain gold jewellery. It was meant to check the move of importers over the last few months, who have been using a policy flaw to source plain gold jewellery from Indonesia without paying any import taxes. The government suspected re-routing of gold from Indonesia to India via FTA.
    Statement 2 is incorrect: The Reserve Bank of India is the custodian of foreign exchange reserves.
    The components of foreign exchange reserves include:
    1) Foreign Currency Assets
    2) Gold reserves
    3) Special Drawing Rights (an international reserve asset created by the International Monetary Fund (IMF) to supplement the official reserves of the member countries)
    4) Reserve Tranche Position in the IMF
    Currently, Foreign Currency Assets are the largest component of foreign exchange reserves of the country, followed by gold.

  4. Question 4 of 5
    4. Question

    4. With respect to employment elasticity, consider the following statements:
    1. Employment elasticity refers to the percentage change in employment with respect to percentage change in labour force.
    2. Jobless growth in the economy implies low employment elasticity.
    Which of the above given statements is/are correct?

    Correct

    Answer: B
    Explanation
    Statement 1 is incorrect: Employment elasticity is a measure of the percentage change in employment associated with a one percentage change in economic growth and not labour force.
    Statement 2 is correct: Less than 1% increase in employment relative to 1% increase in output implies jobless growth, ie, low employment elasticity implies jobless growth.
    • India is said to have been going through the phenomenon of jobless growth. Employment elasticity has consistently fallen between 1983 and 2017 in India. This is because the workers who have been released from agriculture sector have not been absorbed by other sectors of the economy.
    • Construction sector, the second largest employer after agriculture sector, has been a shock absorber as it employs many unskilled and low-skilled workers.
    • India has not produced enough good quality jobs commensurate with its GDP growth.

    Incorrect

    Answer: B
    Explanation
    Statement 1 is incorrect: Employment elasticity is a measure of the percentage change in employment associated with a one percentage change in economic growth and not labour force.
    Statement 2 is correct: Less than 1% increase in employment relative to 1% increase in output implies jobless growth, ie, low employment elasticity implies jobless growth.
    • India is said to have been going through the phenomenon of jobless growth. Employment elasticity has consistently fallen between 1983 and 2017 in India. This is because the workers who have been released from agriculture sector have not been absorbed by other sectors of the economy.
    • Construction sector, the second largest employer after agriculture sector, has been a shock absorber as it employs many unskilled and low-skilled workers.
    • India has not produced enough good quality jobs commensurate with its GDP growth.

  5. Question 5 of 5
    5. Question

    5. With reference to financial instruments, consider the following statements:
    1. Both Treasury Bills and Cash Management Bills (CMBs) are capital market instruments.
    2. While Treasury Bills are issued by the RBI to meet Union government’s temporary cash flow mismatches, CMBs are issued by banks or eligible Financial Institutions to raise short-term funds.
    Which of the above given statements is/are correct.

    Correct

    Answer: D
    Explanation:
    Statement 1 is incorrect: Both Treasury Bills and Cash Management Bills (CMBs) are money market instruments.
    Statement 2 is incorrect.
    Cash Management Bills:
    • These are short-term security sold by the Reserve Bank of India on behalf of the Union government to meet the temporary cash flow mismatches of the government.
    • These are similar to T-Bills, but the tenure is less than 91 days.
    • These are issued at discount to the face value through auctions.
    • State governments cannot issue CMBs.
    Treasury Bills
    • These are short-term debt instruments issued by the government of India to meet the short-term requirements.
    • They are issued in three tenors: 91-day, 182-day and 364-day.
    • These are zero-coupon securities and pay no interest.
    • Instead, they are issued at a discount and redeemed at the face value at maturity.
    • Treasury bills are not issued by the state governments.

    Incorrect

    Answer: D
    Explanation:
    Statement 1 is incorrect: Both Treasury Bills and Cash Management Bills (CMBs) are money market instruments.
    Statement 2 is incorrect.
    Cash Management Bills:
    • These are short-term security sold by the Reserve Bank of India on behalf of the Union government to meet the temporary cash flow mismatches of the government.
    • These are similar to T-Bills, but the tenure is less than 91 days.
    • These are issued at discount to the face value through auctions.
    • State governments cannot issue CMBs.
    Treasury Bills
    • These are short-term debt instruments issued by the government of India to meet the short-term requirements.
    • They are issued in three tenors: 91-day, 182-day and 364-day.
    • These are zero-coupon securities and pay no interest.
    • Instead, they are issued at a discount and redeemed at the face value at maturity.
    • Treasury bills are not issued by the state governments.

window.wpAdvQuizInitList = window.wpAdvQuizInitList || []; window.wpAdvQuizInitList.push({ id: '#wpAdvQuiz_608', init: { quizId: 608, mode: 0, globalPoints: 10, timelimit: 0, resultsGrade: [0], bo: 0, qpp: 0, catPoints: [10], formPos: 0, lbn: "Finish quiz", json: {"2829":{"type":"single","id":2829,"catId":0,"points":2,"correct":[0,1,0,0]},"2830":{"type":"single","id":2830,"catId":0,"points":2,"correct":[0,0,1,0]},"2831":{"type":"single","id":2831,"catId":0,"points":2,"correct":[1,0,0,0]},"2832":{"type":"single","id":2832,"catId":0,"points":2,"correct":[0,1,0,0]},"2833":{"type":"single","id":2833,"catId":0,"points":2,"correct":[0,0,0,1]}} } });




Day-567 | Daily MCQs | UPSC Prelims | GEOGRAPHY

Day-567

Time limit: 0

Quiz-summary

0 of 5 questions completed

Questions:

  1. 1
  2. 2
  3. 3
  4. 4
  5. 5

Information

DAILY MCQ

You have already completed the quiz before. Hence you can not start it again.

Quiz is loading...

You must sign in or sign up to start the quiz.

You have to finish following quiz, to start this quiz:

Results

0 of 5 questions answered correctly

Your time:

Time has elapsed

You have reached 0 of 0 points, (0)

Categories

  1. Not categorized 0%
  1. 1
  2. 2
  3. 3
  4. 4
  5. 5
  1. Answered
  2. Review
  1. Question 1 of 5
    1. Question

    1. Consider the following statements with respect to earthquake waves:
    1. S-waves cannot pass through fluids.
    2. P-waves travel about 10 times faster than S-waves.
    3. Rayleigh waves cause most of the damage from an earthquake.
    How many of the above statements are correct?

    Correct

    Answer: B
    Context:
    • Statement 1 is correct: P- Waves can pass through air, solid and liquid while S-waves can pass through the solids only.
    • Statement 2 is incorrect: P-waves are 1.7 times faster than S-waves.
    • Statement 3 is correct: Rayleigh waves cause the maximum damage from an earthquake.
    Additional Information
    Earthquake Waves-
    • Two main terms: Focus and Epicentre
    • The rock rupture or sudden breakage of rocks deep within the crust at a point or place is known as focus or hypocenter, and the point on the Earth’s surface located vertically above the focus is termed as epicentre.
    • The depth of focus from the epicentre, called as focal depth, is an important parameter in determining the potential damage of an Earthquake.
    • The vibrations or seismic waves radiate from this point of rupture or focus, and are propagated up to the Earth’s surface.
    There are two types of waves: Body waves and Surface waves released during an Earthquake.
    Primary Waves:
    • P wave is the fastest kind of seismic wave and first to arrive at a seismic station that can be recorded on the Seismograph.
    • This can move through solid rock, liquids and gases.
    • P waves are “push and pull” waves like sound waves, which pushes and pulls the air in each particle vibrating to and fro in the direction of propagation.
    • These are also known as compressional or longitudinal waves.
    • P-wave velocity in earthquakes is in the range 5 to 8 km/s.
    • The precise speed varies according to the region of the Earth’s interior, from less than 6 km/s in the Earth’s crust to 13.5 km/s in the lower mantle, and 11 km/s through the inner core.
    • P-waves are about 1.7 times faster than the S-waves.
    Secondary Waves:
    • The second type of body waves is the S wave or secondary wave which appears only after the P waves have arrived.
    • These are transverse or shear waves, in which the motion of each particle is at right angle to the direction of propagation.
    • S waves can pass only through solid rock and move rock particles up and down, or side-to-side-perpendicular to the direction of the wave travels.
    • Both P and S waves travel through the body of the Earth hence, they are called as body waves.
    • S-waves cannot pass through fluids (liquids and gases) as fluids do not support shear stresses.
    Surface waves are two types: Love waves and Rayleigh waves.
    Love wave:
    It is the fastest surface wave and moves the ground from side-to-side in horizontal motion.
    Rayleigh waves:
    • They are much larger than the other waves move the ground up and down and side-to-side in the same direction of wave.
    • It rolls along the ground just like a wave rolls across the ocean or lake.
    • Most of the shaking and damage from an earthquake is due to the Rayleigh wave.

    Incorrect

    Answer: B
    Context:
    • Statement 1 is correct: P- Waves can pass through air, solid and liquid while S-waves can pass through the solids only.
    • Statement 2 is incorrect: P-waves are 1.7 times faster than S-waves.
    • Statement 3 is correct: Rayleigh waves cause the maximum damage from an earthquake.
    Additional Information
    Earthquake Waves-
    • Two main terms: Focus and Epicentre
    • The rock rupture or sudden breakage of rocks deep within the crust at a point or place is known as focus or hypocenter, and the point on the Earth’s surface located vertically above the focus is termed as epicentre.
    • The depth of focus from the epicentre, called as focal depth, is an important parameter in determining the potential damage of an Earthquake.
    • The vibrations or seismic waves radiate from this point of rupture or focus, and are propagated up to the Earth’s surface.
    There are two types of waves: Body waves and Surface waves released during an Earthquake.
    Primary Waves:
    • P wave is the fastest kind of seismic wave and first to arrive at a seismic station that can be recorded on the Seismograph.
    • This can move through solid rock, liquids and gases.
    • P waves are “push and pull” waves like sound waves, which pushes and pulls the air in each particle vibrating to and fro in the direction of propagation.
    • These are also known as compressional or longitudinal waves.
    • P-wave velocity in earthquakes is in the range 5 to 8 km/s.
    • The precise speed varies according to the region of the Earth’s interior, from less than 6 km/s in the Earth’s crust to 13.5 km/s in the lower mantle, and 11 km/s through the inner core.
    • P-waves are about 1.7 times faster than the S-waves.
    Secondary Waves:
    • The second type of body waves is the S wave or secondary wave which appears only after the P waves have arrived.
    • These are transverse or shear waves, in which the motion of each particle is at right angle to the direction of propagation.
    • S waves can pass only through solid rock and move rock particles up and down, or side-to-side-perpendicular to the direction of the wave travels.
    • Both P and S waves travel through the body of the Earth hence, they are called as body waves.
    • S-waves cannot pass through fluids (liquids and gases) as fluids do not support shear stresses.
    Surface waves are two types: Love waves and Rayleigh waves.
    Love wave:
    It is the fastest surface wave and moves the ground from side-to-side in horizontal motion.
    Rayleigh waves:
    • They are much larger than the other waves move the ground up and down and side-to-side in the same direction of wave.
    • It rolls along the ground just like a wave rolls across the ocean or lake.
    • Most of the shaking and damage from an earthquake is due to the Rayleigh wave.

  2. Question 2 of 5
    2. Question

    2. Consider the following statements:
    1. Oases are depressions of varying sizes in desert regions.
    2. Rainwater is the main source of water in oases.
    3. Oases help in supporting a settled life in desert regions.
    How many of the statements given above are correct?

    Correct

    Answer: B
    Explanation:
    • Statement 1 is correct: In the desert, oases are the natural depressions of varying sizes. In these depressions, the underground water reaches the surface. So, statement 2 is incorrect.
    • They are typically supplied by groundwater or springs and temporary rivers can also sustain some oases to desert depressions.
    • Statement 3 is correct: Oases act as natural spring in the hostile desert and thus support a settled life. The oasis people live in mud-brick houses with flat roofs, closely packed together. A planned settlement with a suq (central market place), the mosque, school and shopping blocks. eg. Tafilalet Oasis in Morocco.

    Incorrect

    Answer: B
    Explanation:
    • Statement 1 is correct: In the desert, oases are the natural depressions of varying sizes. In these depressions, the underground water reaches the surface. So, statement 2 is incorrect.
    • They are typically supplied by groundwater or springs and temporary rivers can also sustain some oases to desert depressions.
    • Statement 3 is correct: Oases act as natural spring in the hostile desert and thus support a settled life. The oasis people live in mud-brick houses with flat roofs, closely packed together. A planned settlement with a suq (central market place), the mosque, school and shopping blocks. eg. Tafilalet Oasis in Morocco.

  3. Question 3 of 5
    3. Question

    3. Consider the following pairs:
    Places in news – Countries involved
    1. Rafah crossing – Armenia-Azerbaijan border
    2. Zangezur corridor – Columbia-Panama border
    3. Darien gap – Gaza-Egypt border
    How many of the above given pairs are correctly matched?

    Correct

    Answer: D
    Explanation:
    Pair 1 is matched incorrectly:
    Rafah crossing:
    • Rafah crossing is in news due to ongoing Israel-Hamas conflict.
    • In the wake of Israel’s blockade of the Gaza Strip following the October 7 attack by Hamas, the Rafah border crossing with Egypt has become a crucial point for civilians seeking to enter or leave Gaza.
    • This crossing, unlike others, is not controlled by Israel.
    Pair 2 is matched incorrectly:
    Zangezur corridor: In news due to ongoing Armenia-Azerbaijan conflict.
    • Zangezur, which is currently part of southern Armenia, has been a disputed territory since World War I.
    • Under the communist Soviet Union, the area, which is located between Baku’s Nakhchivan autonomous region and Azerbaijan, became part of the Armenian Soviet Socialist Republic (SSR). Today, Armenia considers the area its Syunik province.
    • The Zangezur Corridor runs through Azerbaijan and Armenia territories reaching Nakhchivan, Azerbaijan’s autonomous region, which neighbours Türkiye.
    • The corridor is part of a strategic transportation route extending from the Azerbaijani capital, Baku, to Kars, Türkiye’s eastern province, passing through Armenian territory near the country’s border with Iran.
    Pair 3 is matched incorrectly:
    • Darien gap is a geographic region in the Isthmus of Panama connecting the American continents within Central America.
    • It is consisting of a large watershed, forest, and mountains in Panama’s Darién Province and the northern portion of Colombia’s Chocó Department.

    Incorrect

    Answer: D
    Explanation:
    Pair 1 is matched incorrectly:
    Rafah crossing:
    • Rafah crossing is in news due to ongoing Israel-Hamas conflict.
    • In the wake of Israel’s blockade of the Gaza Strip following the October 7 attack by Hamas, the Rafah border crossing with Egypt has become a crucial point for civilians seeking to enter or leave Gaza.
    • This crossing, unlike others, is not controlled by Israel.
    Pair 2 is matched incorrectly:
    Zangezur corridor: In news due to ongoing Armenia-Azerbaijan conflict.
    • Zangezur, which is currently part of southern Armenia, has been a disputed territory since World War I.
    • Under the communist Soviet Union, the area, which is located between Baku’s Nakhchivan autonomous region and Azerbaijan, became part of the Armenian Soviet Socialist Republic (SSR). Today, Armenia considers the area its Syunik province.
    • The Zangezur Corridor runs through Azerbaijan and Armenia territories reaching Nakhchivan, Azerbaijan’s autonomous region, which neighbours Türkiye.
    • The corridor is part of a strategic transportation route extending from the Azerbaijani capital, Baku, to Kars, Türkiye’s eastern province, passing through Armenian territory near the country’s border with Iran.
    Pair 3 is matched incorrectly:
    • Darien gap is a geographic region in the Isthmus of Panama connecting the American continents within Central America.
    • It is consisting of a large watershed, forest, and mountains in Panama’s Darién Province and the northern portion of Colombia’s Chocó Department.

  4. Question 4 of 5
    4. Question

    4. With reference to the Shaligram Stones, a sacred stone worshipped by Hindus, consider the following statements:
    1. These stones are fossils of ammonite, which is a type of mollusc that lived millions years ago.
    2. These are mostly found in the rivers flowing in the Vindhya Range.
    Which of the statements given above is/are correct?

    Correct

    Answer: A
    Explanation:
    Statement 1 is correct:
    • The Ram Temple in Ayodhya is a much-awaited landmark in Hindu religion, with the idol of Lord Ram and Sita to be carved out of a special stone known as the Shaligram Stone.
    • Anthropologist Holly Walters In her book, ‘Shaligram Pilgrimage in the Nepal Himalayas’,has written that shaligram stones are fossils of ammonite, which is a type of mollusc that lived between 400 million and 65 million years ago.
    • Referring to a Geological Survey of India publication from 1904, Walters wrote that shaligram stones “date specifically from the Early Oxfordian to the Late Tithonian Age near the end of the Jurassic Period some 165-140 million years ago”.
    Statement 2 is incorrect:
    • These stones are mostly found in riverbeds or banks of the Kali Gandaki, a tributary of the Gandaki River in Nepal.
    • These stones are revered by Hindus who believe it to be a representation of Lord Vishnu.
    • According to Hindu mythology, Lord Vishnu was cursed to become the shaligram stone for “betraying the chastity of the goddess Tulsi”.

    Incorrect

    Answer: A
    Explanation:
    Statement 1 is correct:
    • The Ram Temple in Ayodhya is a much-awaited landmark in Hindu religion, with the idol of Lord Ram and Sita to be carved out of a special stone known as the Shaligram Stone.
    • Anthropologist Holly Walters In her book, ‘Shaligram Pilgrimage in the Nepal Himalayas’,has written that shaligram stones are fossils of ammonite, which is a type of mollusc that lived between 400 million and 65 million years ago.
    • Referring to a Geological Survey of India publication from 1904, Walters wrote that shaligram stones “date specifically from the Early Oxfordian to the Late Tithonian Age near the end of the Jurassic Period some 165-140 million years ago”.
    Statement 2 is incorrect:
    • These stones are mostly found in riverbeds or banks of the Kali Gandaki, a tributary of the Gandaki River in Nepal.
    • These stones are revered by Hindus who believe it to be a representation of Lord Vishnu.
    • According to Hindu mythology, Lord Vishnu was cursed to become the shaligram stone for “betraying the chastity of the goddess Tulsi”.

  5. Question 5 of 5
    5. Question

    5. Consider the following statements regarding the ‘Theri’ desert:
    1. It is a part of the larger Thar Desert.
    2. Southwest monsoon play an important role in the formation of red dunes in this desert.
    Which of the statements given above is/are correct?

    Correct

    Answer: B
    Explanation:
    Statement 1 is incorrect:
    Theri Desert:
    • It is a small desert situated in the state of Tamil Nadu.
    • It consists of red sand dunes and is confined to the Thoothukudi district.
    • The red dunes are called theri in Tamil.
    • They consist of sediments dating back to the Quaternary Period (which began 2.6 million years ago) and are made of marine deposits.
    • They have very low water and nutrient retention capacity.
    • The dunes are susceptible to aerodynamic lift. This is the push that lets something move up. It is the force that is the opposite of weight.
    Statement 2 is correct:
    Formation of Theri desert:
    • Theory 1: Theri appear as gentle, undulating terrain. The lithology (the study of general physical characteristics of rocks) of the area shows that the area might have been a paleo (ancient) coast in the past. The presence of limestone in many places indicates marine transgression.
    • Theory 2: The present-day theris might have been formed by the confinement of beach sand locally, after regression of the sea. When high velocity winds from the Western Ghats blew east, they induced migration of sand grains and accumulation of dunes.
    • Theory 3: Another view is that these are geological formations that appeared in a period of a few hundred years.
    • There is a lot of red sand spread over these theris.
    • The red sand is brought from the surface of a broad belt of red loam in the plains of the Nanguneri region (about 57 kilometres from this area in Tirunelveli district) by south west monsoon winds during May-September.
    Additional Information
    Mineral Composition of Theri:
    • The petrographical study (petrography is the study of composition and properties of rocks) and X-ray diffraction analysis (a method used to determine a material’s crystallographic structure) of the red sand dunes reveal the presence of heavy and light minerals.
    • These include Ilmenite, Magnetite, Rutile, Garnet, Zircon, Diopside, Tourmaline, Hematite, Goethite, Kyanite, Quartz, Feldspar, and Biotite.
    • The iron-rich heavy minerals like ilmenite, magnetite, garnet, hypersthene, and rutile present in the soil had undergone leaching by surface water and were then oxidized because of the favorable semi- arid climatic conditions.
    • It was due to these processes that the dunes near Tiruchendur, a coastal town of the Thoothukudi district are red-colored.

    Incorrect

    Answer: B
    Explanation:
    Statement 1 is incorrect:
    Theri Desert:
    • It is a small desert situated in the state of Tamil Nadu.
    • It consists of red sand dunes and is confined to the Thoothukudi district.
    • The red dunes are called theri in Tamil.
    • They consist of sediments dating back to the Quaternary Period (which began 2.6 million years ago) and are made of marine deposits.
    • They have very low water and nutrient retention capacity.
    • The dunes are susceptible to aerodynamic lift. This is the push that lets something move up. It is the force that is the opposite of weight.
    Statement 2 is correct:
    Formation of Theri desert:
    • Theory 1: Theri appear as gentle, undulating terrain. The lithology (the study of general physical characteristics of rocks) of the area shows that the area might have been a paleo (ancient) coast in the past. The presence of limestone in many places indicates marine transgression.
    • Theory 2: The present-day theris might have been formed by the confinement of beach sand locally, after regression of the sea. When high velocity winds from the Western Ghats blew east, they induced migration of sand grains and accumulation of dunes.
    • Theory 3: Another view is that these are geological formations that appeared in a period of a few hundred years.
    • There is a lot of red sand spread over these theris.
    • The red sand is brought from the surface of a broad belt of red loam in the plains of the Nanguneri region (about 57 kilometres from this area in Tirunelveli district) by south west monsoon winds during May-September.
    Additional Information
    Mineral Composition of Theri:
    • The petrographical study (petrography is the study of composition and properties of rocks) and X-ray diffraction analysis (a method used to determine a material’s crystallographic structure) of the red sand dunes reveal the presence of heavy and light minerals.
    • These include Ilmenite, Magnetite, Rutile, Garnet, Zircon, Diopside, Tourmaline, Hematite, Goethite, Kyanite, Quartz, Feldspar, and Biotite.
    • The iron-rich heavy minerals like ilmenite, magnetite, garnet, hypersthene, and rutile present in the soil had undergone leaching by surface water and were then oxidized because of the favorable semi- arid climatic conditions.
    • It was due to these processes that the dunes near Tiruchendur, a coastal town of the Thoothukudi district are red-colored.

window.wpAdvQuizInitList = window.wpAdvQuizInitList || []; window.wpAdvQuizInitList.push({ id: '#wpAdvQuiz_607', init: { quizId: 607, mode: 0, globalPoints: 10, timelimit: 0, resultsGrade: [0], bo: 0, qpp: 0, catPoints: [10], formPos: 0, lbn: "Finish quiz", json: {"2824":{"type":"single","id":2824,"catId":0,"points":2,"correct":[0,1,0,0]},"2825":{"type":"single","id":2825,"catId":0,"points":2,"correct":[0,1,0,0]},"2826":{"type":"single","id":2826,"catId":0,"points":2,"correct":[0,0,0,1]},"2827":{"type":"single","id":2827,"catId":0,"points":2,"correct":[1,0,0,0]},"2828":{"type":"single","id":2828,"catId":0,"points":2,"correct":[0,1,0,0]}} } });




Day-566 | Daily MCQs | UPSC Prelims | POLITY

Day-566

Time limit: 0

Quiz-summary

0 of 5 questions completed

Questions:

  1. 1
  2. 2
  3. 3
  4. 4
  5. 5

Information

DAILY MCQ

You have already completed the quiz before. Hence you can not start it again.

Quiz is loading...

You must sign in or sign up to start the quiz.

You have to finish following quiz, to start this quiz:

Results

0 of 5 questions answered correctly

Your time:

Time has elapsed

You have reached 0 of 0 points, (0)

Categories

  1. Not categorized 0%
  1. 1
  2. 2
  3. 3
  4. 4
  5. 5
  1. Answered
  2. Review
  1. Question 1 of 5
    1. Question

    1. Consider the following statements:
    1. The President of India makes rules for the more convenient transaction of the business of the Government of India.
    2. All executive actions of the Government of India shall be expressed to be taken in the name of the Prime Minister.
    3. The Constitution authorizes the President to seek the opinion of the Supreme Court only in matters related to a fact of public importance.
    How many of the above given statements are correct?

    Correct

    Answer: A
    Explanation:
    Statement 1 is correct: Article 77(3) of the Constitution of India empowers the President to make rules for two crucial aspects of government functioning:
    1. Transaction of Business: The President can determine how the government conducts its internal operations. This includes aspects like:
    • Meetings and procedures of the Cabinet: This could involve setting the frequency of meetings, agenda format, voting procedures, and decision-making processes within the Cabinet.
    • Communication and coordination between ministries: The President can establish channels for information exchange and collaboration among different government departments.
    • Delegation of powers: The President can define the extent to which ministers can make decisions on behalf of the government without requiring Cabinet approval.
    2. Allocation of Business: The President can assign specific areas of responsibility to individual ministers. This involves:
    • Dividing ministerial portfolios: The President can determine which ministries will handle specific sectors like finance, defense, education, or health.
    • Redistribution of portfolios: The President can adjust ministerial responsibilities based on changing priorities or government reorganization.
    • Coordination among ministries: The President can establish mechanisms for inter-ministerial collaboration on issues that span multiple portfolios.
    Significance of this Article-77 (3): It grants the President significant influence in shaping the government’s internal structure and functioning.
    • It ensures efficient and coordinated handling of government business by establishing clear lines of authority and communication.
    • It provides flexibility for the government to adapt its structure and responsibilities to changing circumstances.
    Statement 2 is incorrect: Article 77(1) of the Constitution states that “All executive action of the Government of India shall be expressed to be taken in the name of the President.” This means that while the Prime Minister and cabinet lead the government and make decisions, all official actions and documents are issued and executed in the President’s name.
    Statement 3 is incorrect: The statement is partially correct; while Article 143 of the Indian Constitution empowers the President to seek the Supreme Court’s opinion, it encompasses more than just matters related to a fact of public importance. It actually covers two categories i.e. question of law or fact of public importance: This includes any legal or factual issue that has arisen or is likely to arise and significantly impacts the public interest. It could involve interpretations of the Constitution, disputes regarding treaties, or complex policy matters. However, it’s important to note that the Supreme Court’s opinion under Article 143 is purely advisory, meaning it’s not legally binding on the President.

    Incorrect

    Answer: A
    Explanation:
    Statement 1 is correct: Article 77(3) of the Constitution of India empowers the President to make rules for two crucial aspects of government functioning:
    1. Transaction of Business: The President can determine how the government conducts its internal operations. This includes aspects like:
    • Meetings and procedures of the Cabinet: This could involve setting the frequency of meetings, agenda format, voting procedures, and decision-making processes within the Cabinet.
    • Communication and coordination between ministries: The President can establish channels for information exchange and collaboration among different government departments.
    • Delegation of powers: The President can define the extent to which ministers can make decisions on behalf of the government without requiring Cabinet approval.
    2. Allocation of Business: The President can assign specific areas of responsibility to individual ministers. This involves:
    • Dividing ministerial portfolios: The President can determine which ministries will handle specific sectors like finance, defense, education, or health.
    • Redistribution of portfolios: The President can adjust ministerial responsibilities based on changing priorities or government reorganization.
    • Coordination among ministries: The President can establish mechanisms for inter-ministerial collaboration on issues that span multiple portfolios.
    Significance of this Article-77 (3): It grants the President significant influence in shaping the government’s internal structure and functioning.
    • It ensures efficient and coordinated handling of government business by establishing clear lines of authority and communication.
    • It provides flexibility for the government to adapt its structure and responsibilities to changing circumstances.
    Statement 2 is incorrect: Article 77(1) of the Constitution states that “All executive action of the Government of India shall be expressed to be taken in the name of the President.” This means that while the Prime Minister and cabinet lead the government and make decisions, all official actions and documents are issued and executed in the President’s name.
    Statement 3 is incorrect: The statement is partially correct; while Article 143 of the Indian Constitution empowers the President to seek the Supreme Court’s opinion, it encompasses more than just matters related to a fact of public importance. It actually covers two categories i.e. question of law or fact of public importance: This includes any legal or factual issue that has arisen or is likely to arise and significantly impacts the public interest. It could involve interpretations of the Constitution, disputes regarding treaties, or complex policy matters. However, it’s important to note that the Supreme Court’s opinion under Article 143 is purely advisory, meaning it’s not legally binding on the President.

  2. Question 2 of 5
    2. Question

    2. Consider the following statements:
    1. Right to constitutional remedies are fundamental rights under the constitution.
    2. Right to education is included as a fundamental right under the Indian Constitution.
    3. Fundamental Rights in India are absolute and cannot be restricted under any circumstances.
    How many of the above given statements are correct?

    Correct

    Answer: B
    Explanation:
    Statement 1 is correct: Article 32 of the Constitution of India guarantees the right to constitutional remedies, empowering individuals to approach the Supreme Court for the enforcement of their fundamental rights. Often referred to as the “heart and soul” of the Indian Constitution. It grants citizens the right to move the Supreme Court directly if they believe their fundamental rights have been violated by any executive action or legislative provision. This right plays a crucial role in upholding the rule of law and ensuring justice for all.
    Statement 2 is correct: The Indian Constitution recognizes the importance of education through Article 21A (Right to Education) and Article 45 (Directive Principles). The Right to Education is explicitly listed as a fundamental right. Also, the Right to Education Act, 2009, enacted by the Parliament, makes elementary education (6-14 years) a fundamental right for all children.
    Statement 3 is incorrect: While fundamental rights are essential safeguards, they are not absolute. The Constitution empowers the government to impose reasonable restrictions on certain rights under specific circumstances for the greater good of society.
    For example, freedom of speech and expression can be limited in the interest of national security, public order, or morality (Article 19(2) of the Constitution of India).

    Incorrect

    Answer: B
    Explanation:
    Statement 1 is correct: Article 32 of the Constitution of India guarantees the right to constitutional remedies, empowering individuals to approach the Supreme Court for the enforcement of their fundamental rights. Often referred to as the “heart and soul” of the Indian Constitution. It grants citizens the right to move the Supreme Court directly if they believe their fundamental rights have been violated by any executive action or legislative provision. This right plays a crucial role in upholding the rule of law and ensuring justice for all.
    Statement 2 is correct: The Indian Constitution recognizes the importance of education through Article 21A (Right to Education) and Article 45 (Directive Principles). The Right to Education is explicitly listed as a fundamental right. Also, the Right to Education Act, 2009, enacted by the Parliament, makes elementary education (6-14 years) a fundamental right for all children.
    Statement 3 is incorrect: While fundamental rights are essential safeguards, they are not absolute. The Constitution empowers the government to impose reasonable restrictions on certain rights under specific circumstances for the greater good of society.
    For example, freedom of speech and expression can be limited in the interest of national security, public order, or morality (Article 19(2) of the Constitution of India).

  3. Question 3 of 5
    3. Question

    3. Consider the following:
    1. The Constitution of India
    2. The Supreme Court Rules
    3. Conventions
    The authority of the Chief Justice of India (CJI) to allocate cases to benches in the Supreme Court, stems from how many of the above mentioned sources?

    Correct

    Answer: C
    Explanation: While there’s no single specific law defining the Chief Justice of India’s (CJI) power to allocate cases to benches in the Supreme Court, their authority stems from a combination of factors:
    1. The Constitution of India:
    • Article 145 of the Constitution empowers the Supreme Court to make rules for regulating its practice and procedure. This broad power lays the foundation for the Court’s own internal rules guiding case allocation.
    2. The Supreme Court Rules, 2013:
    • These rules, formulated by the Court itself under Article 145 of the Constitution of India, explicitly recognize the CJI as the “Master of the Roster.” They establish the framework for bench formation, case allocation, and listing orders, but leave some discretion to the CJI.
    3. Conventions and Established Practices:
    • Over time, the Court has developed specific conventions and practices regarding case allocation, often based on past precedents and judges’ expertise.
    4. Judicial Independence:
    • The independence of the judiciary, a fundamental principle in India, grants the CJI significant authority regarding internal court matters. This includes freedom to determine case allocation procedures as long as they adhere to the broader constitutional and legal framework.
    Therefore, the CJI’s power to allocate cases arises from a confluence of constitutional provision, self-made rules,established practices, and judicial independence. This framework balances flexibility with accountability, allowing the CJI to tailor case allocation while ensuring adherence to broader legal principles.

    Incorrect

    Answer: C
    Explanation: While there’s no single specific law defining the Chief Justice of India’s (CJI) power to allocate cases to benches in the Supreme Court, their authority stems from a combination of factors:
    1. The Constitution of India:
    • Article 145 of the Constitution empowers the Supreme Court to make rules for regulating its practice and procedure. This broad power lays the foundation for the Court’s own internal rules guiding case allocation.
    2. The Supreme Court Rules, 2013:
    • These rules, formulated by the Court itself under Article 145 of the Constitution of India, explicitly recognize the CJI as the “Master of the Roster.” They establish the framework for bench formation, case allocation, and listing orders, but leave some discretion to the CJI.
    3. Conventions and Established Practices:
    • Over time, the Court has developed specific conventions and practices regarding case allocation, often based on past precedents and judges’ expertise.
    4. Judicial Independence:
    • The independence of the judiciary, a fundamental principle in India, grants the CJI significant authority regarding internal court matters. This includes freedom to determine case allocation procedures as long as they adhere to the broader constitutional and legal framework.
    Therefore, the CJI’s power to allocate cases arises from a confluence of constitutional provision, self-made rules,established practices, and judicial independence. This framework balances flexibility with accountability, allowing the CJI to tailor case allocation while ensuring adherence to broader legal principles.

  4. Question 4 of 5
    4. Question

    4. Consider the following statements:
    1. The Seventh Schedule provides the legislative competency in residual matters to the Union legislature except in those related to the state list.
    2. Amending the Seventh Schedule requires a simple majority vote in both the houses of Parliament.
    3. The Sarkaria Commission recommended that the residuary powers of legislation with regard to taxation matters should remain with the parliament.
    How many of the above given statements are correct?

    Correct

    Answer: A
    Explanation:
    Statement 1 is incorrect: As per Entry 97 of the Union list, Seventh Schedule, the residual power of legislation lies with the Union legislature and the state legislatures have no power over them.
    Entry 97: Any other matter not enumerated in List II or List III including any tax not mentioned in either of those Lists.
    The Seventh Schedule is constituted under Article 246 of the Constitution of India:
    • Article 246 deals with the distribution of legislative powers between the Union (central government) and the States.It divides subjects into three lists:Union List, State List and Concurrent List.
    • The Seventh Schedule provides a detailed breakdown of specific areas under each list, defining the scope of legislative powers for both the Union and the States.
    • This division enables both levels of government to function effectively within their respective jurisdictions.
    Statement 2 is incorrect: Amending the Seventh Schedule requires a special majority in both houses of Parliament, and consent of atleast half of the states by simple majority.
    Statement 3 is correct: Recommendations of the Sarkaria Commission:
    • It recommended that the residuary powers of legislation in regard to taxation matters should remain exclusively in the competence of Parliament while the residuary field other than that of taxation should be placed on the concurrent list.
    • To ensure uniformity on the basic issues of national policy, with respect to the subject of a proposed legislation, consultations may be carried out with the state governments individually and collectively at the forum of the proposed Inter-Governmental Council. It was not recommended that the consultation be a constitutional obligation.
    • Ordinarily, the Union should occupy only that much field of a concurrent subject on which uniformity of policy and action is essential in the larger interest of the nation, leaving the rest and details for state action.
    • On administrative relations, Sarkaria made the following observation: “Federalism is more a functional arrangement for cooperative action, than a static institutional concept. Article 258 (power of the Union to confer powers etc on states in certain cases) provides a tool by the liberal use of which cooperative federalism can be substantially realised in the working of the system. A more generous use of this tool should be made than has hitherto been done, for progressive decentralisation of powers to the governments of the states.”
    • On Article 356, it was recommended that it be used “very sparingly, in extreme cases, as a measure of last resort, when all other alternatives fail to prevent or rectify a breakdown of constitutional machinery in the state.

    Incorrect

    Answer: A
    Explanation:
    Statement 1 is incorrect: As per Entry 97 of the Union list, Seventh Schedule, the residual power of legislation lies with the Union legislature and the state legislatures have no power over them.
    Entry 97: Any other matter not enumerated in List II or List III including any tax not mentioned in either of those Lists.
    The Seventh Schedule is constituted under Article 246 of the Constitution of India:
    • Article 246 deals with the distribution of legislative powers between the Union (central government) and the States.It divides subjects into three lists:Union List, State List and Concurrent List.
    • The Seventh Schedule provides a detailed breakdown of specific areas under each list, defining the scope of legislative powers for both the Union and the States.
    • This division enables both levels of government to function effectively within their respective jurisdictions.
    Statement 2 is incorrect: Amending the Seventh Schedule requires a special majority in both houses of Parliament, and consent of atleast half of the states by simple majority.
    Statement 3 is correct: Recommendations of the Sarkaria Commission:
    • It recommended that the residuary powers of legislation in regard to taxation matters should remain exclusively in the competence of Parliament while the residuary field other than that of taxation should be placed on the concurrent list.
    • To ensure uniformity on the basic issues of national policy, with respect to the subject of a proposed legislation, consultations may be carried out with the state governments individually and collectively at the forum of the proposed Inter-Governmental Council. It was not recommended that the consultation be a constitutional obligation.
    • Ordinarily, the Union should occupy only that much field of a concurrent subject on which uniformity of policy and action is essential in the larger interest of the nation, leaving the rest and details for state action.
    • On administrative relations, Sarkaria made the following observation: “Federalism is more a functional arrangement for cooperative action, than a static institutional concept. Article 258 (power of the Union to confer powers etc on states in certain cases) provides a tool by the liberal use of which cooperative federalism can be substantially realised in the working of the system. A more generous use of this tool should be made than has hitherto been done, for progressive decentralisation of powers to the governments of the states.”
    • On Article 356, it was recommended that it be used “very sparingly, in extreme cases, as a measure of last resort, when all other alternatives fail to prevent or rectify a breakdown of constitutional machinery in the state.

  5. Question 5 of 5
    5. Question

    5. Consider the following statements about the Lokpal:
    1. The Lokpal can investigate an offence alleged to be committed by a public servant under the Prevention of Corruption Act, 1988.
    2. A member of the Lokpal can be appointed as its Chairperson for a period of five years or until he attains the age of Seventy years.
    3. The Lokpal can investigate private individuals under certain circumstances with the consent of the full bench of the Lokpal.
    How many of the above given statements are correct?

    Correct

    Answer: A
    Explanation:
    Statement 1 is correct: The Lokpal’s jurisdiction in investigating corruption matters is primarily defined by the Lokpal and Lokayuktas Act, 2013, and indeed, its core authority focuses on offenses covered by the Prevention of Corruption Act, 1988 (PCA). This means the Lokpal can investigate public servants accused of offenses like bribery, taking undue advantage to obtain gratification, and misconduct in public office as outlined in the PCA.
    Statement 2 is incorrect: A per Section 8 of the Act, a member can be appointed as the Chairperson. But he or she shall be eligible to be appointed as the Chairperson, if his total tenure as Member and Chairperson does not exceed five years. It means where amember is appointed as the Chairperson, his/her term of office will not be more than five years in aggregate as the Member and the Chairperson.
    Statement 3 is incorrect: Although the Lokpal can investigate the corruption complaints against private individuals, the consent of the full bench of the Lokpal is not needed. In certain circumstances, even the private persons will be deemed to be pubic servants for the purposes of initiating inquiry and investigation. For example, as per Section 14 of the Lokpal and Lokayuktas Act, 2013, a person who is a director or other office bearer of a trust/society etc will be deemed to be a public servant. This is based on the condition that if such an organisation receives a donation of 10 Lakh in a year from a foreign source under FCRA, 2010.
    Section 14:any person who is or has been a director, manager, secretary or other officer of every other society or association of persons or trust (whether registered under any law for the time being in force or not) in receipt of any donation from any foreign source under the Foreign Contribution (Regulation) Act, 2010 in excess of ten lakh rupees in a year or such higher amount as the Central Government may specify; shall be deemed to be a public servant under clause (c) of section 2 of the Prevention of Corruption Act, 1988.

    Incorrect

    Answer: A
    Explanation:
    Statement 1 is correct: The Lokpal’s jurisdiction in investigating corruption matters is primarily defined by the Lokpal and Lokayuktas Act, 2013, and indeed, its core authority focuses on offenses covered by the Prevention of Corruption Act, 1988 (PCA). This means the Lokpal can investigate public servants accused of offenses like bribery, taking undue advantage to obtain gratification, and misconduct in public office as outlined in the PCA.
    Statement 2 is incorrect: A per Section 8 of the Act, a member can be appointed as the Chairperson. But he or she shall be eligible to be appointed as the Chairperson, if his total tenure as Member and Chairperson does not exceed five years. It means where amember is appointed as the Chairperson, his/her term of office will not be more than five years in aggregate as the Member and the Chairperson.
    Statement 3 is incorrect: Although the Lokpal can investigate the corruption complaints against private individuals, the consent of the full bench of the Lokpal is not needed. In certain circumstances, even the private persons will be deemed to be pubic servants for the purposes of initiating inquiry and investigation. For example, as per Section 14 of the Lokpal and Lokayuktas Act, 2013, a person who is a director or other office bearer of a trust/society etc will be deemed to be a public servant. This is based on the condition that if such an organisation receives a donation of 10 Lakh in a year from a foreign source under FCRA, 2010.
    Section 14:any person who is or has been a director, manager, secretary or other officer of every other society or association of persons or trust (whether registered under any law for the time being in force or not) in receipt of any donation from any foreign source under the Foreign Contribution (Regulation) Act, 2010 in excess of ten lakh rupees in a year or such higher amount as the Central Government may specify; shall be deemed to be a public servant under clause (c) of section 2 of the Prevention of Corruption Act, 1988.

window.wpAdvQuizInitList = window.wpAdvQuizInitList || []; window.wpAdvQuizInitList.push({ id: '#wpAdvQuiz_606', init: { quizId: 606, mode: 0, globalPoints: 10, timelimit: 0, resultsGrade: [0], bo: 0, qpp: 0, catPoints: [10], formPos: 0, lbn: "Finish quiz", json: {"2819":{"type":"single","id":2819,"catId":0,"points":2,"correct":[1,0,0,0]},"2820":{"type":"single","id":2820,"catId":0,"points":2,"correct":[0,1,0,0]},"2821":{"type":"single","id":2821,"catId":0,"points":2,"correct":[0,0,1,0]},"2822":{"type":"single","id":2822,"catId":0,"points":2,"correct":[1,0,0,0]},"2823":{"type":"single","id":2823,"catId":0,"points":2,"correct":[1,0,0,0]}} } });




Day-565 | Daily MCQs | UPSC Prelims | HISTORY

Day-565

Time limit: 0

Quiz-summary

0 of 5 questions completed

Questions:

  1. 1
  2. 2
  3. 3
  4. 4
  5. 5

Information

DAILY MCQ

You have already completed the quiz before. Hence you can not start it again.

Quiz is loading...

You must sign in or sign up to start the quiz.

You have to finish following quiz, to start this quiz:

Results

0 of 5 questions answered correctly

Your time:

Time has elapsed

You have reached 0 of 0 points, (0)

Categories

  1. Not categorized 0%
  1. 1
  2. 2
  3. 3
  4. 4
  5. 5
  1. Answered
  2. Review
  1. Question 1 of 5
    1. Question

    1. Which of the following statements are correct in relation to the copper image of a dancing girl found from Mohenjo-Daro?
    1. Her long hairs are tied in a bun.
    2. Bangles cover her right arm.
    3. She has large eyes and a sharp nose.
    4. She is wearing a necklace made from shells.
    Select the correct answer using the code given below:

    Correct

    Answer: D
    Explanation:
    The copper/bronze image of Mohenjodaro dancing girl was found in the remains of a small house by the Indian archaeologist D. R. Sahni in 1926-1927. It is one of the finest artefacts from the Indus Valley Civilization (2500 BCE). At present, this naturalistic free-standing sculpture of a woman is on display at the IVC gallery of the National Museum, New Delhi. She has very long legs and arms compared to her torso; her head is tilted slightly backward and her left leg is bent at the knee. The key features of this four inches statuette are as follows:
    • The image, full of expression and bodily vigour, is wearing a cowry shell necklace.
    • Her long curly hairs are plaited, coiled and neatly tied into a bun. She has large eyes & a flat nose.
    • Her left arm is covered with 24-25 bangles, and in the right arm bent at the elbow she is wearing 4 bangles, two at the wrist, and two above the elbow.
    • Her right hand rests on her hip, and the left hand appears to be clasped in a traditional Indian dance gesture resting on her left thigh.

    Incorrect

    Answer: D
    Explanation:
    The copper/bronze image of Mohenjodaro dancing girl was found in the remains of a small house by the Indian archaeologist D. R. Sahni in 1926-1927. It is one of the finest artefacts from the Indus Valley Civilization (2500 BCE). At present, this naturalistic free-standing sculpture of a woman is on display at the IVC gallery of the National Museum, New Delhi. She has very long legs and arms compared to her torso; her head is tilted slightly backward and her left leg is bent at the knee. The key features of this four inches statuette are as follows:
    • The image, full of expression and bodily vigour, is wearing a cowry shell necklace.
    • Her long curly hairs are plaited, coiled and neatly tied into a bun. She has large eyes & a flat nose.
    • Her left arm is covered with 24-25 bangles, and in the right arm bent at the elbow she is wearing 4 bangles, two at the wrist, and two above the elbow.
    • Her right hand rests on her hip, and the left hand appears to be clasped in a traditional Indian dance gesture resting on her left thigh.

  2. Question 2 of 5
    2. Question

    2. Consider the following statements:
    Statement I: Some of the most experimental hybrid styles of vesara architecture are found the region of Karnataka.
    Statement II: The hybridization and incorporation of several styles was the hallmark of Chalukyas, Hoysalas and Vijayanagar.
    Which one of the following is correct in respect of the above statements?

    Correct

    Answer: A
    Explanation:
    Vesara, a hybrid form of Indian temple architecture, has a South Indian plan and shapes that features North Indian details. This fusion style originated in Karnataka and evolved under the patronage of the Chalukyas, Rashtrakutas, Hoysalas, and Vijayanagar rulers. There is a lot of variation in the designs and styles of vesara temples as no set guidelines are followed in their plan and superstructure. The important features of Vesara style of temple architecture are as follows:
    • Generally, the temples have Nagara style Shikar and Dravida style Mandapas.
    • Shikar, the temple’s top, and Mandapa, the temple’s main shrine are connected by the Antarala.
    • Elaborate carvings are found on the pillars, doorframes, and ceilings of the Deccan temples.
    • The Vesara style is a wonderful example of using the knowledge of art, architecture, mathematics, religion, philosophy, and engineering.
    Some examples of the Vesara style are Virupaksha temple at Hampi, and the Chennakesava temple at Belur. The Hoysala temples at Belur, Halebidu and Somanathpura are not only the leading examples of the Vesara style. They are now also included in list of UNESCO world heritage sites.

    Incorrect

    Answer: A
    Explanation:
    Vesara, a hybrid form of Indian temple architecture, has a South Indian plan and shapes that features North Indian details. This fusion style originated in Karnataka and evolved under the patronage of the Chalukyas, Rashtrakutas, Hoysalas, and Vijayanagar rulers. There is a lot of variation in the designs and styles of vesara temples as no set guidelines are followed in their plan and superstructure. The important features of Vesara style of temple architecture are as follows:
    • Generally, the temples have Nagara style Shikar and Dravida style Mandapas.
    • Shikar, the temple’s top, and Mandapa, the temple’s main shrine are connected by the Antarala.
    • Elaborate carvings are found on the pillars, doorframes, and ceilings of the Deccan temples.
    • The Vesara style is a wonderful example of using the knowledge of art, architecture, mathematics, religion, philosophy, and engineering.
    Some examples of the Vesara style are Virupaksha temple at Hampi, and the Chennakesava temple at Belur. The Hoysala temples at Belur, Halebidu and Somanathpura are not only the leading examples of the Vesara style. They are now also included in list of UNESCO world heritage sites.

  3. Question 3 of 5
    3. Question

    3. Consider the following pairs:
    Authors – Texts
    1. Sarangadev – Sangeetratnakara
    2. Lochana – Raagtarangini
    3. Ahobala – Sangeeta Parijata
    4. Srinivasa – Ragatattvabodha
    How many pairs given above are correctly matched?

    Correct

    Answer: D
    Explanation: All the four pairs are correctly matched. The given texts are famous works of music written by the mentioned authors.
    1. Sangeetratnakara is a 13th century text of music authored by Sarangadev in Sanskrit. It is considered a definitive source by both the traditions of Indian classic music i.e. Hindustani and Carnatic music.
    2. Raagtarangini written by Lochana Kavi in Maithili set several songs to contemporary ragas and raginis.
    3. Ahobala Pandit’s Sangeeta Parijata is a 17th century work that introduces the elements of South Indian music in the North. And, it is regarded by some as the earliest text of the Hindustani music.
    4. Ragatattvabodha is a rare book on music written in the 17th century.

    Incorrect

    Answer: D
    Explanation: All the four pairs are correctly matched. The given texts are famous works of music written by the mentioned authors.
    1. Sangeetratnakara is a 13th century text of music authored by Sarangadev in Sanskrit. It is considered a definitive source by both the traditions of Indian classic music i.e. Hindustani and Carnatic music.
    2. Raagtarangini written by Lochana Kavi in Maithili set several songs to contemporary ragas and raginis.
    3. Ahobala Pandit’s Sangeeta Parijata is a 17th century work that introduces the elements of South Indian music in the North. And, it is regarded by some as the earliest text of the Hindustani music.
    4. Ragatattvabodha is a rare book on music written in the 17th century.

  4. Question 4 of 5
    4. Question

    4. Regarding observations made by the Moroccan traveller Ibn Battuta, How many of the following are correct about medieval India?
    1. The land was very fertile.
    2. The cities in the Indian subcontinent were sparsely populated.
    3. The sultan of Delhi was unpredictable.
    Select the correct answer using the codes given below:

    Correct

    Answer: B
    Explanation:
    Ibn Battuta recorded his observations about the lands he visited in his travelogue Rehla. He was appointed as the Qazi of Delhi by Mohammed bin Tughluq (1325-51) whom he found quite eccentric in his behavior. He also served as the Sultan’s envoy to the Chinese emperor in 1342.
    He described the Indian cities and towns as highly populous and wealthy. He was quite impressed with the life and colours of Delhi and Daulatabad. According to him, the Indian cities were full of opportunities for the ambitious, resourceful and skilful people. And the markets were the hubs of socio-cultural activities and economic transactions.
    He also observed that the Indian soils were very fertile allowing farmers to grow two crops i.e. Rabi and Kharif, annually. He identified rice, sugarcane, wheat, oilseeds, sesame, cotton and barley as the important crops.

    Incorrect

    Answer: B
    Explanation:
    Ibn Battuta recorded his observations about the lands he visited in his travelogue Rehla. He was appointed as the Qazi of Delhi by Mohammed bin Tughluq (1325-51) whom he found quite eccentric in his behavior. He also served as the Sultan’s envoy to the Chinese emperor in 1342.
    He described the Indian cities and towns as highly populous and wealthy. He was quite impressed with the life and colours of Delhi and Daulatabad. According to him, the Indian cities were full of opportunities for the ambitious, resourceful and skilful people. And the markets were the hubs of socio-cultural activities and economic transactions.
    He also observed that the Indian soils were very fertile allowing farmers to grow two crops i.e. Rabi and Kharif, annually. He identified rice, sugarcane, wheat, oilseeds, sesame, cotton and barley as the important crops.

  5. Question 5 of 5
    5. Question

    5. Who was the founder of the Lokayata school of Indian philosophy?

    Correct

    Answer: C
    Explanation:
    Lokayata literally means the philosophy of the people or the masses. It is one of the earliest schools which have been mentioned in the Vedas and Brihadaranyaka Upanishad. It is the ancient philosophy of Indian materialism supposed to be founded by Brihaspati. Charvaka is the most powerful proponent of this philosophy who recognizes only four out of the five elements.
    Charvakas or Lokayatas does not recognise ether as it is not known through perception. For them, the whole universe is made of four elements, and knowledge is the product of these four elements which leaves no trace after death. There is no other world, hence death is the end of humans and pleasure is the ultimate object of life. It rejects ritualism and recognises no existence other than this material world. Since God, soul and heaven cannot be perceived, they are not recognised by this philosophy.
    Badarayana well-known as Vyasa Muni composed the Brahma Sutras, the source of the orthodox Hindu philosophy called the uttar-mimamsa or Vedanta.
    Charaka was an ancient physician and scholar. He is recognized as the father of Ayurveda. He described various diseases and their treatments in his book Charak-samhita.

    Incorrect

    Answer: C
    Explanation:
    Lokayata literally means the philosophy of the people or the masses. It is one of the earliest schools which have been mentioned in the Vedas and Brihadaranyaka Upanishad. It is the ancient philosophy of Indian materialism supposed to be founded by Brihaspati. Charvaka is the most powerful proponent of this philosophy who recognizes only four out of the five elements.
    Charvakas or Lokayatas does not recognise ether as it is not known through perception. For them, the whole universe is made of four elements, and knowledge is the product of these four elements which leaves no trace after death. There is no other world, hence death is the end of humans and pleasure is the ultimate object of life. It rejects ritualism and recognises no existence other than this material world. Since God, soul and heaven cannot be perceived, they are not recognised by this philosophy.
    Badarayana well-known as Vyasa Muni composed the Brahma Sutras, the source of the orthodox Hindu philosophy called the uttar-mimamsa or Vedanta.
    Charaka was an ancient physician and scholar. He is recognized as the father of Ayurveda. He described various diseases and their treatments in his book Charak-samhita.

window.wpAdvQuizInitList = window.wpAdvQuizInitList || []; window.wpAdvQuizInitList.push({ id: '#wpAdvQuiz_604', init: { quizId: 604, mode: 0, globalPoints: 10, timelimit: 0, resultsGrade: [0], bo: 0, qpp: 0, catPoints: [10], formPos: 0, lbn: "Finish quiz", json: {"2814":{"type":"single","id":2814,"catId":0,"points":2,"correct":[0,0,0,1]},"2815":{"type":"single","id":2815,"catId":0,"points":2,"correct":[1,0,0,0]},"2816":{"type":"single","id":2816,"catId":0,"points":2,"correct":[0,0,0,1]},"2817":{"type":"single","id":2817,"catId":0,"points":2,"correct":[0,1,0,0]},"2818":{"type":"single","id":2818,"catId":0,"points":2,"correct":[0,0,1,0]}} } });




Day-564 | Daily MCQs | UPSC Prelims | ENVIRONMENT

Day-564

Time limit: 0

Quiz-summary

0 of 5 questions completed

Questions:

  1. 1
  2. 2
  3. 3
  4. 4
  5. 5

Information

DAILY MCQ

You have already completed the quiz before. Hence you can not start it again.

Quiz is loading...

You must sign in or sign up to start the quiz.

You have to finish following quiz, to start this quiz:

Results

0 of 5 questions answered correctly

Your time:

Time has elapsed

You have reached 0 of 0 points, (0)

Categories

  1. Not categorized 0%
  1. 1
  2. 2
  3. 3
  4. 4
  5. 5
  1. Answered
  2. Review
  1. Question 1 of 5
    1. Question

    1. Consider the following statements:
    Statement I: Wild pigs and nilgai enjoy the same level of protection in India.
    Statement II: Both of these species are listed under the Schedule III of the Wildlife (Protection) Act, 1972.
    Which one of the following is correct in respect of the above statements?

    Correct

    Answer: C
    Explanation:
    Statement 1 is correct but statement 2 is incorrect: Wild pigs and nilgai, among others, enjoy the same level of protection in the country. Wild pigs and nilgai are placed under Schedule II of the Wildlife Protection Act after the recent amendments made to the list in 2023.
    ● The hunting of these animals by any individual is prohibited as per the provisions of the Wildlife Protection Act.
    ● If the Chief Wildlife Warden or the authorised officer is satisfied that any wild animal specified in Schedule II has become dangerous to human life or to property (including standing crops on any land) or is disabled or diseased beyond recovery, only then the hunting of such animals is permitted by written order of the authority.
    ● The provisions of the amended act also mention that the Central Government through a notification can declare any wild animal specified in Schedule II to be vermin for any area. The period for which it is declared as vermin should also be specified in the notification.

    Incorrect

    Answer: C
    Explanation:
    Statement 1 is correct but statement 2 is incorrect: Wild pigs and nilgai, among others, enjoy the same level of protection in the country. Wild pigs and nilgai are placed under Schedule II of the Wildlife Protection Act after the recent amendments made to the list in 2023.
    ● The hunting of these animals by any individual is prohibited as per the provisions of the Wildlife Protection Act.
    ● If the Chief Wildlife Warden or the authorised officer is satisfied that any wild animal specified in Schedule II has become dangerous to human life or to property (including standing crops on any land) or is disabled or diseased beyond recovery, only then the hunting of such animals is permitted by written order of the authority.
    ● The provisions of the amended act also mention that the Central Government through a notification can declare any wild animal specified in Schedule II to be vermin for any area. The period for which it is declared as vermin should also be specified in the notification.

  2. Question 2 of 5
    2. Question

    2. The term ‘ Ampelognathus coheni ‘ mentioned in the media was in reference to:

    Correct

    Answer. A
    Explanation: An extraordinary discovery has been made in the Dallas-Fort Worth area of Texas in the USA, as palaeontologists have identified a new dinosaur species that lived approximately 96 million years ago. The species, named Ampelognathus coheni, is particularly significant because herbivorous dinosaur fossils are seldom found in North Texas.
    It is the first small-bodied ornithopod dinosaur discovered from the Lewisville Formation.
    Dinosaurs roamed Earth roughly 175 million years ago, and most were wiped out by an extinction event roughly 65 million years ago.
    Additional information: Similar discovery from India
    Indian palaeontologists have unearthed one of the world’s oldest fossils of a long-neck, plant-eating dinosaur from Thar desert in Rajasthan.
    The dinosaur has been named Tharosaurus indicus after the desert and is the oldest known member of a family of the ancient creatures called Diplodocoidea. A more famous member of the same family is Diplodocus, made popular by blockbuster movies like Jurassic Park.
    Tharosaurus is half of a size of a diplodocus. Its neck-to-tail length is about 10-13 mt while that of a diplodocus was around 26 mt.

    Incorrect

    Answer. A
    Explanation: An extraordinary discovery has been made in the Dallas-Fort Worth area of Texas in the USA, as palaeontologists have identified a new dinosaur species that lived approximately 96 million years ago. The species, named Ampelognathus coheni, is particularly significant because herbivorous dinosaur fossils are seldom found in North Texas.
    It is the first small-bodied ornithopod dinosaur discovered from the Lewisville Formation.
    Dinosaurs roamed Earth roughly 175 million years ago, and most were wiped out by an extinction event roughly 65 million years ago.
    Additional information: Similar discovery from India
    Indian palaeontologists have unearthed one of the world’s oldest fossils of a long-neck, plant-eating dinosaur from Thar desert in Rajasthan.
    The dinosaur has been named Tharosaurus indicus after the desert and is the oldest known member of a family of the ancient creatures called Diplodocoidea. A more famous member of the same family is Diplodocus, made popular by blockbuster movies like Jurassic Park.
    Tharosaurus is half of a size of a diplodocus. Its neck-to-tail length is about 10-13 mt while that of a diplodocus was around 26 mt.

  3. Question 3 of 5
    3. Question

    3. Consider the following:
    1. Killer whales
    2. Chimpanzees
    3. Humans
    4. Dolphins
    5. Orangutan
    Menopause as a characteristic feature, has been witnessed in how many of the above-mentioned species?

    Correct

    Answer. C
    Explanation: Options 1, 2, 3 and 5 are correct.
    While humans, short-finned pilot whales, killer whales are among the only long-lived mammals that undergo menopause, dolphins continue to reproduce throughout their life. Dolphins don’t experience menopause.
    ● After decades of observations in a rainforest in Uganda, the scientists discovered that some chimpanzees go through menopause, too. Without much disease, Ngogo chimpanzees may live long enough to experience menopause.
    ● A female orangutan named ‘Gina’ at Durrell wildlife park (Jerzey Island) is believed to have gone through the menopause, the first time this is known to have been medically recorded. Orangutans and humans share 97.6% of their DNA. This research has established that orangutans experience menopause too.

    Incorrect

    Answer. C
    Explanation: Options 1, 2, 3 and 5 are correct.
    While humans, short-finned pilot whales, killer whales are among the only long-lived mammals that undergo menopause, dolphins continue to reproduce throughout their life. Dolphins don’t experience menopause.
    ● After decades of observations in a rainforest in Uganda, the scientists discovered that some chimpanzees go through menopause, too. Without much disease, Ngogo chimpanzees may live long enough to experience menopause.
    ● A female orangutan named ‘Gina’ at Durrell wildlife park (Jerzey Island) is believed to have gone through the menopause, the first time this is known to have been medically recorded. Orangutans and humans share 97.6% of their DNA. This research has established that orangutans experience menopause too.

  4. Question 4 of 5
    4. Question

    4. Consider the following statements:
    1. The term ‘Critical Tiger Habitat’ is mentioned in the Wildlife (Protection) Act, 1972.
    2. The term ‘Critical Wildlife Habitat’ is defined in the Forest Rights Act, 2006.
    3. A national park or a wildlife sanctuary constituting the core area of a tiger reserve is included in ‘Critical Tiger Habitat’.
    How many of the above statements are correct?

    Correct

    Answer. C
    Explanation:
    Statements 1 and 3 are correct: “Section 38V” of the Wildlife (Protection) Act, 1972 (as amended in 2006) explains the core or critical tiger habitat as well as the buffer or peripheral area of a tiger reserve.
    A tiger reserve includes two parts:
    A. Core area or critical tiger habitat (National Park or Sanctuary status).
    B. Buffer or peripheral area
    The phrase ‘core or critical tiger habitat’ is mentioned only in the Wildlife (Protection) Act, 1972, as a sequel to the amendment made to the said Act in 2006. It is NOT defined in the Scheduled Tribes and Other Traditional Forest Dwellers (Recognition of Forest Rights) Act, 2006.
    Critical tiger habitats are considered as inviolate areas in tiger reserves. ‘Core or critical tiger habitat’ is different from ‘critical wildlife habitat’. Since tigers are territorial big cats, hence considering their social land tenure dynamics, the ‘core / critical tiger habitat’ has been viewed separately from the ‘critical wildlife habitat’, which is applicable to other wild animal species.
    Statement 2 is correct: The phrase ‘critical wildlife habitat’ is defined only in the Scheduled Tribes and Other Traditional Forest Dwellers (Recognition of Forest Rights) Act, 2006, and NOT in the Wildlife (Protection) Act, 1972. Critical Wildlife Habitat (CWH) is notified within national parks and wildlife sanctuaries.
    FRA defines CWHs as ‘areas of national parks and sanctuaries where it has been specifically and clearly established, case by case, on the basis of scientific and objective criteria, that such areas are required to be kept as inviolate for the purposes of wildlife conservation’. In order to notify a CWH, the Act requires state governments to establish that the presence of right-holders is causing irreversible damage to wildlife and their habitats, and that co-existence between rights holders and wildlife was not a reasonable option.
    While the Ministry of Tribal Affairs is the nodal authority for the Forest Rights Act, the law identifies the Ministry of Environment, Forest & Climate Change (MoEF&CC) as the agency to notify the guidelines regarding the notification and declaration of critical wildlife habitats.
    MoEF&CC finally issued CWH guidelines in January 2018. Key features of guidelines:
    ● The Chief Wildlife Warden of a state will notify an Expert Committee for the purpose of identification of critical wildlife habitats (CWH) in a national park or sanctuary.
    ● The Expert Committee will identify areas within national parks and sanctuaries, based on scientific and objective criteria relevant to the protected area, required to be kept inviolate for the purpose of wildlife conservation.
    ● The Expert Committee shall issue a public notice on the intention to notify CWH. The public notice shall include details of areas required to be kept inviolate, criteria adopted for CWH identification, implication of the notification on existing rights, and all options of resettlement and rehabilitation schemes, if applicable.
    ● The Expert Committee shall carry out open consultations with all stakeholders, and the proceedings of the consultations, especially the objections, will be documented appropriately.
    ● The committee will submit the CWH proposal to the Chief Wildlife Warden. The decision on the proposal will be taken by the Standing Committee of the National Board for Wildlife. A Ministry of Tribal Affairs representative would be invited during the deliberation of the proposal by the standing committee. Following the committee’s recommendation, the notification of CWH will be published in the official gazette.

    Incorrect

    Answer. C
    Explanation:
    Statements 1 and 3 are correct: “Section 38V” of the Wildlife (Protection) Act, 1972 (as amended in 2006) explains the core or critical tiger habitat as well as the buffer or peripheral area of a tiger reserve.
    A tiger reserve includes two parts:
    A. Core area or critical tiger habitat (National Park or Sanctuary status).
    B. Buffer or peripheral area
    The phrase ‘core or critical tiger habitat’ is mentioned only in the Wildlife (Protection) Act, 1972, as a sequel to the amendment made to the said Act in 2006. It is NOT defined in the Scheduled Tribes and Other Traditional Forest Dwellers (Recognition of Forest Rights) Act, 2006.
    Critical tiger habitats are considered as inviolate areas in tiger reserves. ‘Core or critical tiger habitat’ is different from ‘critical wildlife habitat’. Since tigers are territorial big cats, hence considering their social land tenure dynamics, the ‘core / critical tiger habitat’ has been viewed separately from the ‘critical wildlife habitat’, which is applicable to other wild animal species.
    Statement 2 is correct: The phrase ‘critical wildlife habitat’ is defined only in the Scheduled Tribes and Other Traditional Forest Dwellers (Recognition of Forest Rights) Act, 2006, and NOT in the Wildlife (Protection) Act, 1972. Critical Wildlife Habitat (CWH) is notified within national parks and wildlife sanctuaries.
    FRA defines CWHs as ‘areas of national parks and sanctuaries where it has been specifically and clearly established, case by case, on the basis of scientific and objective criteria, that such areas are required to be kept as inviolate for the purposes of wildlife conservation’. In order to notify a CWH, the Act requires state governments to establish that the presence of right-holders is causing irreversible damage to wildlife and their habitats, and that co-existence between rights holders and wildlife was not a reasonable option.
    While the Ministry of Tribal Affairs is the nodal authority for the Forest Rights Act, the law identifies the Ministry of Environment, Forest & Climate Change (MoEF&CC) as the agency to notify the guidelines regarding the notification and declaration of critical wildlife habitats.
    MoEF&CC finally issued CWH guidelines in January 2018. Key features of guidelines:
    ● The Chief Wildlife Warden of a state will notify an Expert Committee for the purpose of identification of critical wildlife habitats (CWH) in a national park or sanctuary.
    ● The Expert Committee will identify areas within national parks and sanctuaries, based on scientific and objective criteria relevant to the protected area, required to be kept inviolate for the purpose of wildlife conservation.
    ● The Expert Committee shall issue a public notice on the intention to notify CWH. The public notice shall include details of areas required to be kept inviolate, criteria adopted for CWH identification, implication of the notification on existing rights, and all options of resettlement and rehabilitation schemes, if applicable.
    ● The Expert Committee shall carry out open consultations with all stakeholders, and the proceedings of the consultations, especially the objections, will be documented appropriately.
    ● The committee will submit the CWH proposal to the Chief Wildlife Warden. The decision on the proposal will be taken by the Standing Committee of the National Board for Wildlife. A Ministry of Tribal Affairs representative would be invited during the deliberation of the proposal by the standing committee. Following the committee’s recommendation, the notification of CWH will be published in the official gazette.

  5. Question 5 of 5
    5. Question

    5. With reference to environment, the term ‘eDNA’, recently seen in the news, can be best understood as:

    Correct

    Answer. C
    Explanation: The ability to identify certain animals living in an environment through environmental DNA, or ‘eDNA.’ has revolutionized conservation and wildlife monitoring.
    Environmental DNA or eDNA refers to eDNA is DNA shed by all organisms into their surroundings through natural processes during their lifetime or after death.
    A new method has been developed by the researchers at the Laboratory for the Conservation of Endangered Species (LaCONES) which is cheaper, faster, and highly scalable to large freshwater and marine ecosystems which can help in monitoring and conserving the rich biodiversity of our country.
    LaCONES is a CSIR-Centre for Cellular and Molecular Biology (CCMB) laboratory headquartered in Hyderabad.
    The new non-invasive method can assess the total biodiversity of any ecosystem by sequencing the DNA fragments found in the environmental samples such as water, soil or air. This method can detect all kinds of organisms, including viruses, bacteria, archaea, and eukaryotes such as fungi, plants, insects, birds, fish and other animals from just a few litres of water sample without any direct capture or counting of species.
    The scientists tested their method in the highly biodiverse wetland ecosystem of Chilika Lagoon in Odisha, the country’s largest brackish water lagoon. By comparing over 10 billion sequences of eDNA fragments from multiple seasonal samples with a large database of reference sequences from all the known species, the researchers were able to detect organisms across the tree of life.

    Incorrect

    Answer. C
    Explanation: The ability to identify certain animals living in an environment through environmental DNA, or ‘eDNA.’ has revolutionized conservation and wildlife monitoring.
    Environmental DNA or eDNA refers to eDNA is DNA shed by all organisms into their surroundings through natural processes during their lifetime or after death.
    A new method has been developed by the researchers at the Laboratory for the Conservation of Endangered Species (LaCONES) which is cheaper, faster, and highly scalable to large freshwater and marine ecosystems which can help in monitoring and conserving the rich biodiversity of our country.
    LaCONES is a CSIR-Centre for Cellular and Molecular Biology (CCMB) laboratory headquartered in Hyderabad.
    The new non-invasive method can assess the total biodiversity of any ecosystem by sequencing the DNA fragments found in the environmental samples such as water, soil or air. This method can detect all kinds of organisms, including viruses, bacteria, archaea, and eukaryotes such as fungi, plants, insects, birds, fish and other animals from just a few litres of water sample without any direct capture or counting of species.
    The scientists tested their method in the highly biodiverse wetland ecosystem of Chilika Lagoon in Odisha, the country’s largest brackish water lagoon. By comparing over 10 billion sequences of eDNA fragments from multiple seasonal samples with a large database of reference sequences from all the known species, the researchers were able to detect organisms across the tree of life.

window.wpAdvQuizInitList = window.wpAdvQuizInitList || []; window.wpAdvQuizInitList.push({ id: '#wpAdvQuiz_603', init: { quizId: 603, mode: 0, globalPoints: 10, timelimit: 0, resultsGrade: [0], bo: 0, qpp: 0, catPoints: [10], formPos: 0, lbn: "Finish quiz", json: {"2809":{"type":"single","id":2809,"catId":0,"points":2,"correct":[0,0,1,0]},"2810":{"type":"single","id":2810,"catId":0,"points":2,"correct":[1,0,0,0]},"2811":{"type":"single","id":2811,"catId":0,"points":2,"correct":[0,0,1,0]},"2812":{"type":"single","id":2812,"catId":0,"points":2,"correct":[0,0,1,0]},"2813":{"type":"single","id":2813,"catId":0,"points":2,"correct":[0,0,1,0]}} } });




Day-563 | Daily MCQs | UPSC Prelims | ENVIRONMENT AND ECOLOGY

Day-563

Time limit: 0

Quiz-summary

0 of 5 questions completed

Questions:

  1. 1
  2. 2
  3. 3
  4. 4
  5. 5

Information

DAILY MCQ

You have already completed the quiz before. Hence you can not start it again.

Quiz is loading...

You must sign in or sign up to start the quiz.

You have to finish following quiz, to start this quiz:

Results

0 of 5 questions answered correctly

Your time:

Time has elapsed

You have reached 0 of 0 points, (0)

Categories

  1. Not categorized 0%
  1. 1
  2. 2
  3. 3
  4. 4
  5. 5
  1. Answered
  2. Review
  1. Question 1 of 5
    1. Question

    1. The term ‘Title 42’, often seen in the news, is related to which of the following?

    Correct

    Answer: B
    Context: The United States lifted Title 42 that has blocked migrants caught at the U.S.-Mexico border from seeking asylum since 2020 in May 2023.
    Explanation:
    ● The COVID restrictions, known as Title 42, were first implemented under Republican then-President Donald Trump in March 2020 at the beginning of the pandemic.
    ● At the time, the U.S. Centers for Disease Control and Prevention (CDC) said the order was needed to stem the spread of the coronavirus in crowded detention settings.
    ● Title 42 allowed border agents to rapidly expel many migrants to Mexico, but some public health experts, Democrats and advocates criticized its health justification, saying it was part of Trump’s goal of curbing legal and illegal immigration.

    Incorrect

    Answer: B
    Context: The United States lifted Title 42 that has blocked migrants caught at the U.S.-Mexico border from seeking asylum since 2020 in May 2023.
    Explanation:
    ● The COVID restrictions, known as Title 42, were first implemented under Republican then-President Donald Trump in March 2020 at the beginning of the pandemic.
    ● At the time, the U.S. Centers for Disease Control and Prevention (CDC) said the order was needed to stem the spread of the coronavirus in crowded detention settings.
    ● Title 42 allowed border agents to rapidly expel many migrants to Mexico, but some public health experts, Democrats and advocates criticized its health justification, saying it was part of Trump’s goal of curbing legal and illegal immigration.

  2. Question 2 of 5
    2. Question

    2. Consider the following statements regarding the Arab League:
    1. It is a regional organization of Arab states which are geographically located only in the Middle East, aiming to strengthen ties among member states.
    2. It was formed in Cairo, Egypt, following the adoption of the Alexandria Protocol in 1944.
    3. It currently consists of 22 members including Palestine which is one of its founding members.
    How many of the statements given above are correct?

    Correct

    Answer: A
    Context: The Arab League in May 2023 voted to reinstate Syria’s membership after its suspension more than 10 years ago, underlining the thawing relations between Damascus and other Arab countries. Syria’s participation has been suspended since November 2011 as a consequence of government repression during the Syrian Civil War.
    Explanation:
    Statement 1 is incorrect: It is a regional organization of Arab states in the Middle East and parts of Africa to strengthen ties among member states.
    Statement 2 is correct: It was formed in Cairo, Egypt, following the adoption of the Alexandria Protocol in 1944.
    Statement 3 is incorrect: It currently consists of 22 members, but Palestine is not one of its founding members. Members: Algeria, Bahrain, Comoros, Djibouti, Egypt, Iraq, Jordan, Kuwait, Lebanon, Libya, Mauritania, Morocco, Oman, Palestine, Qatar, Saudi Arabia, Somalia, Sudan, Syria, Tunisia, the United Arab Emirates, and Yemen.
    Founding members: Egypt, Iraq, Jordan, Lebanon, Saudi Arabia, and Syria.

    Additional information:
    ● It aims to strengthen and coordinate the political, cultural, economic, and social programs of its members and to mediate disputes among them or between them and third parties.
    ● The signing on 13th April 1950, of an agreement on joint defense and economic cooperation also committed the signatories to coordination of military defense measures.
    ● According to the Council on Foreign Relations, an American think tank, “The League makes decisions on a majority basis, but there is no mechanism to compel members to comply with resolutions. “It has been criticised for its internal conflicts and collective inaction on important international issues.”

    Incorrect

    Answer: A
    Context: The Arab League in May 2023 voted to reinstate Syria’s membership after its suspension more than 10 years ago, underlining the thawing relations between Damascus and other Arab countries. Syria’s participation has been suspended since November 2011 as a consequence of government repression during the Syrian Civil War.
    Explanation:
    Statement 1 is incorrect: It is a regional organization of Arab states in the Middle East and parts of Africa to strengthen ties among member states.
    Statement 2 is correct: It was formed in Cairo, Egypt, following the adoption of the Alexandria Protocol in 1944.
    Statement 3 is incorrect: It currently consists of 22 members, but Palestine is not one of its founding members. Members: Algeria, Bahrain, Comoros, Djibouti, Egypt, Iraq, Jordan, Kuwait, Lebanon, Libya, Mauritania, Morocco, Oman, Palestine, Qatar, Saudi Arabia, Somalia, Sudan, Syria, Tunisia, the United Arab Emirates, and Yemen.
    Founding members: Egypt, Iraq, Jordan, Lebanon, Saudi Arabia, and Syria.

    Additional information:
    ● It aims to strengthen and coordinate the political, cultural, economic, and social programs of its members and to mediate disputes among them or between them and third parties.
    ● The signing on 13th April 1950, of an agreement on joint defense and economic cooperation also committed the signatories to coordination of military defense measures.
    ● According to the Council on Foreign Relations, an American think tank, “The League makes decisions on a majority basis, but there is no mechanism to compel members to comply with resolutions. “It has been criticised for its internal conflicts and collective inaction on important international issues.”

  3. Question 3 of 5
    3. Question

    3. The 108-foot-tall ‘Statue of Oneness’ of Adi Shankaracharya was unveiled at Omkareshwar town, Madhya Pradesh recently. Which one of the following statements correctly represents the teachings of Adi Shankaracharya?

    Correct

    Answer: C
    Context: Madhya Pradesh Chief Minister has unveiled a colossal 108-foot statue of the 8th-century Hindu philosopher and saint, Adi Shankaracharya in September 2023 at Omkareshwar town, in Khandwa district, Madhya Pradesh
    Explanation:
    ● According to Sri Shankaracharya, knowledge is the only way to achieve salvation and to attain liberating enlightenment.
    ● He mentions that Sri Krishna in Srimad Bhagavadgita compares knowledge to a bright lamp that destroys the darkness of ignorance.
    Additional information:
    Shankaracharya (c. 788 – 820 CE)
    ● One of the mystic Bhakti poet-saint leaders who gave a new orientation to Hinduism.
    ● He was born in Kaladi in Kerala. He propounded the Advaita (Monism) philosophy and the idea of Nirgunabrahman (god without attributes).
    ● In Advaita, the reality of the world is denied, and Brahman is considered the only reality. It is only Brahman at its base that gives it its reality.
    ● His famous quotes include, ‘Brahma Satyam Jagat Mithya Jivo Brahmatra Naparaha’ meaning, “The Absolute Spirit is the reality, the world of appearance is Maya” and ‘Ekameva Adviteeyam Brahma’ meaning, “The absolute is one alone, not two”.
    ● Upadesasahasri, Vivekachudamani, Bhaja Govindum Stotra are some of the works authored by Shankaracharya. He also wrote commentaries on the Bhagavad Gita, the Brahma Sutra and the Upanishads.
    ● He set up mathas at Dwarka, Puri, Sringeri and Badrinath.

    Incorrect

    Answer: C
    Context: Madhya Pradesh Chief Minister has unveiled a colossal 108-foot statue of the 8th-century Hindu philosopher and saint, Adi Shankaracharya in September 2023 at Omkareshwar town, in Khandwa district, Madhya Pradesh
    Explanation:
    ● According to Sri Shankaracharya, knowledge is the only way to achieve salvation and to attain liberating enlightenment.
    ● He mentions that Sri Krishna in Srimad Bhagavadgita compares knowledge to a bright lamp that destroys the darkness of ignorance.
    Additional information:
    Shankaracharya (c. 788 – 820 CE)
    ● One of the mystic Bhakti poet-saint leaders who gave a new orientation to Hinduism.
    ● He was born in Kaladi in Kerala. He propounded the Advaita (Monism) philosophy and the idea of Nirgunabrahman (god without attributes).
    ● In Advaita, the reality of the world is denied, and Brahman is considered the only reality. It is only Brahman at its base that gives it its reality.
    ● His famous quotes include, ‘Brahma Satyam Jagat Mithya Jivo Brahmatra Naparaha’ meaning, “The Absolute Spirit is the reality, the world of appearance is Maya” and ‘Ekameva Adviteeyam Brahma’ meaning, “The absolute is one alone, not two”.
    ● Upadesasahasri, Vivekachudamani, Bhaja Govindum Stotra are some of the works authored by Shankaracharya. He also wrote commentaries on the Bhagavad Gita, the Brahma Sutra and the Upanishads.
    ● He set up mathas at Dwarka, Puri, Sringeri and Badrinath.

  4. Question 4 of 5
    4. Question

    4. Which of the following is correct regarding the objective of Tele MANAS?

    Correct

    Answer: D
    Context: The Tele Mental Health Assistance and Networking across States (Tele MANAS), helpline has reached a significant milestone. The toll-free service has received over 2,00,000 calls from various parts of the nation since the launch in 2022, portraying a consistently progressive trend.
    Explanation:
    ● The Tele (Mental Health Assistance and Networking Across States) MANAS, the digital arm of the ‘District Mental Health Programme’) launched by the Government of India in October 2022 to strengthen mental health service delivery in the country.
    ● Ministry: Ministry of Health & Family Welfare.
    ● It aims to establish a digital mental health network that will withstand the challenges amplified by the pandemic.
    Objectives of Tele MANAS: –
    ● To enable people to seek support for their mental health issues while maintaining the anonymity of the callers, thereby reducing the stigma generally surrounding mental health issues.
    ● To exponentially scale up the reach of mental health services to anybody who reaches out, across India, any time, by setting up a 24×7 tele-mental health facility in each of the States and UTs of the country.
    ● To implement a full-fledged mental health-service network that, in addition to counselling, provides integrated medical and psychosocial interventions including video consultations with mental health specialists, e-prescriptions, follow-up services and linkages to in-person services.
    ● To extend services to vulnerable groups of the population and difficult-to-reach populations.
    Salient Features of Tele MANAS: –
    ● Tele MANAS will be organized as a two-tier system. (Tele-MANAS)
    ● Tier 1: comprise the State Tele MANAS cells, which include trained counsellors and mental health specialists.
    ● Tier 2: comprise specialists at District Mental Health Programme (DMHP)/Medical College resources for physical consultation and/or e Sanjeevani for audio-visual consultation.
    ● Toll-free helpline numbers: 14416 or 1-800-891-4416 with multi-language provision allows callers to select the language of their choice for availing the services. (Address Stigma and Discrimination in Mental Health)
    Eligibility:
    ● Any individual with mental health issues can reach out to Tele MANAS services for help.
    ● Family members of persons with mental health issues can reach out for help.
    ● Grass-root health care providers/community health providers., Accredited Social Health Activists (ASHAs), and community volunteers from the community can reach out to Tele MANAS on behalf of an individual or individuals in that community with mental health issues.

    Incorrect

    Answer: D
    Context: The Tele Mental Health Assistance and Networking across States (Tele MANAS), helpline has reached a significant milestone. The toll-free service has received over 2,00,000 calls from various parts of the nation since the launch in 2022, portraying a consistently progressive trend.
    Explanation:
    ● The Tele (Mental Health Assistance and Networking Across States) MANAS, the digital arm of the ‘District Mental Health Programme’) launched by the Government of India in October 2022 to strengthen mental health service delivery in the country.
    ● Ministry: Ministry of Health & Family Welfare.
    ● It aims to establish a digital mental health network that will withstand the challenges amplified by the pandemic.
    Objectives of Tele MANAS: –
    ● To enable people to seek support for their mental health issues while maintaining the anonymity of the callers, thereby reducing the stigma generally surrounding mental health issues.
    ● To exponentially scale up the reach of mental health services to anybody who reaches out, across India, any time, by setting up a 24×7 tele-mental health facility in each of the States and UTs of the country.
    ● To implement a full-fledged mental health-service network that, in addition to counselling, provides integrated medical and psychosocial interventions including video consultations with mental health specialists, e-prescriptions, follow-up services and linkages to in-person services.
    ● To extend services to vulnerable groups of the population and difficult-to-reach populations.
    Salient Features of Tele MANAS: –
    ● Tele MANAS will be organized as a two-tier system. (Tele-MANAS)
    ● Tier 1: comprise the State Tele MANAS cells, which include trained counsellors and mental health specialists.
    ● Tier 2: comprise specialists at District Mental Health Programme (DMHP)/Medical College resources for physical consultation and/or e Sanjeevani for audio-visual consultation.
    ● Toll-free helpline numbers: 14416 or 1-800-891-4416 with multi-language provision allows callers to select the language of their choice for availing the services. (Address Stigma and Discrimination in Mental Health)
    Eligibility:
    ● Any individual with mental health issues can reach out to Tele MANAS services for help.
    ● Family members of persons with mental health issues can reach out for help.
    ● Grass-root health care providers/community health providers., Accredited Social Health Activists (ASHAs), and community volunteers from the community can reach out to Tele MANAS on behalf of an individual or individuals in that community with mental health issues.

  5. Question 5 of 5
    5. Question

    5. It is a robotic spacecraft that has been on a seven-year mission to collect and return samples from an asteroid called Bennu which is millions of kilometers from Earth. The mission in the paragraph refers to-

    Correct

    Answer: C
    Context: After a seven-year-round trip, NASA’s OSIRIS-REx brought asteroid samples to Earth in September, 2023.
    Explanation:
    ● NASA’s OSIRIS-REx is a robotic spacecraft that has been on a seven-year mission to collect and return samples from an asteroid called Bennu which is billions of kilometers from Earth.
    ● It took more than two years to reach Bennu after OSIRIS-REx launched in 2016. It then spent almost as much time mapping the asteroid, finding a site to scrape and collecting about 250 grams (8.8 ounces) of the rock. Then it began its journey back toward Earth.
    Additional information:
    ● It is the United States’ first asteroid sample return mission, aiming to collect and carry a pristine, unaltered sample from an asteroid back to earth for scientific study.
    ● The OSIRIS-REx (Origins, Spectral Interpretation, Resource Identification, Security, Regolith Explorer) spacecraft was launched in 2016 for the journey to Bennu.
    ● The mission is essentially a seven-year-long voyage and will conclude when at least 60 grams of samples are delivered back to the Earth (in 2023).
    ● As per NASA , the mission promises to bring the largest amount of extraterrestrial material back to the Earth since the Apollo era.
    ● Apollo was the NASA program that resulted in American astronauts’ making a total of 11 space flights and walking on the moon (1968-72).
    ● The spacecraft contains five instruments meant to explore Bennu, including cameras, a spectrometer and a laser altimeter.

    Incorrect

    Answer: C
    Context: After a seven-year-round trip, NASA’s OSIRIS-REx brought asteroid samples to Earth in September, 2023.
    Explanation:
    ● NASA’s OSIRIS-REx is a robotic spacecraft that has been on a seven-year mission to collect and return samples from an asteroid called Bennu which is billions of kilometers from Earth.
    ● It took more than two years to reach Bennu after OSIRIS-REx launched in 2016. It then spent almost as much time mapping the asteroid, finding a site to scrape and collecting about 250 grams (8.8 ounces) of the rock. Then it began its journey back toward Earth.
    Additional information:
    ● It is the United States’ first asteroid sample return mission, aiming to collect and carry a pristine, unaltered sample from an asteroid back to earth for scientific study.
    ● The OSIRIS-REx (Origins, Spectral Interpretation, Resource Identification, Security, Regolith Explorer) spacecraft was launched in 2016 for the journey to Bennu.
    ● The mission is essentially a seven-year-long voyage and will conclude when at least 60 grams of samples are delivered back to the Earth (in 2023).
    ● As per NASA , the mission promises to bring the largest amount of extraterrestrial material back to the Earth since the Apollo era.
    ● Apollo was the NASA program that resulted in American astronauts’ making a total of 11 space flights and walking on the moon (1968-72).
    ● The spacecraft contains five instruments meant to explore Bennu, including cameras, a spectrometer and a laser altimeter.

window.wpAdvQuizInitList = window.wpAdvQuizInitList || []; window.wpAdvQuizInitList.push({ id: '#wpAdvQuiz_602', init: { quizId: 602, mode: 0, globalPoints: 10, timelimit: 0, resultsGrade: [0], bo: 0, qpp: 0, catPoints: [10], formPos: 0, lbn: "Finish quiz", json: {"2804":{"type":"single","id":2804,"catId":0,"points":2,"correct":[0,1,0,0]},"2805":{"type":"single","id":2805,"catId":0,"points":2,"correct":[1,0,0,0]},"2806":{"type":"single","id":2806,"catId":0,"points":2,"correct":[0,0,1,0]},"2807":{"type":"single","id":2807,"catId":0,"points":2,"correct":[0,0,0,1]},"2808":{"type":"single","id":2808,"catId":0,"points":2,"correct":[0,0,1,0]}} } });




Day-562 | Daily MCQs | UPSC Prelims | POLITY

Day-562

Time limit: 0

Quiz-summary

0 of 5 questions completed

Questions:

  1. 1
  2. 2
  3. 3
  4. 4
  5. 5

Information

DAILY MCQ

You have already completed the quiz before. Hence you can not start it again.

Quiz is loading...

You must sign in or sign up to start the quiz.

You have to finish following quiz, to start this quiz:

Results

0 of 5 questions answered correctly

Your time:

Time has elapsed

You have reached 0 of 0 points, (0)

Categories

  1. Not categorized 0%
  1. 1
  2. 2
  3. 3
  4. 4
  5. 5
  1. Answered
  2. Review
  1. Question 1 of 5
    1. Question

    1. Consider the following statements about the Electoral Bonds:
    1. Electoral bonds are bearer instruments.
    2. Electoral bonds can be used to fund independent candidates.
    3. They can only be purchased from designated branches of the State Bank of India.
    How many of the above given statements are correct?

    Correct

    Answer: B
    Explanation:
    Statement 1 is correct: Electoral bonds are bearer instruments, meaning that they do not carry the name of the donor and their identity remains anonymous.Electoral bonds are designed to protect the anonymity of donors, allowing them to contribute to political parties without fear of reprisal or undue influence. This anonymity aims to encourage transparency and prevent corruption in political funding.
    Statement 2 is incorrect: Electoral bonds are strictly intended for funding registered political parties and cannot be used to directly finance individual and independent candidates. This restriction aims to prevent the use of electoral bonds for personal gain or undue influence over candidates.
    Statement 3 is correct: The government has designated specific branches of the SBI as authorized sellers of electoral bonds. This centralized approach helps to monitor and regulate the issuance of electoral bonds and maintain a record of transactions.
    Additional Information:
    ● The electoral bonds system was introduced in 2017 by way of a Finance bill and it was implemented in 2018.
    ● State Bank of India (SBI) issues the bonds in denominations of Rs 1,000, Rs 10,000, Rs 1 lakh, Rs 10 lakh, and Rs 1 crore which are payable to the bearer on demand and interest-free.
    ● Can be purchased by Indian citizens or entities established in India.
    ● They are valid for 15 calendar days from the date of issue.
    ● SBI is the authorized issuer.
    ● Only the political parties registered under Section 29A of the Representation of the People Act, 1951 and have secured not less than 1% of the votes polled in the last general election to the House of the People or the Legislative Assembly, are eligible to receive electoral bonds.

    Incorrect

    Answer: B
    Explanation:
    Statement 1 is correct: Electoral bonds are bearer instruments, meaning that they do not carry the name of the donor and their identity remains anonymous.Electoral bonds are designed to protect the anonymity of donors, allowing them to contribute to political parties without fear of reprisal or undue influence. This anonymity aims to encourage transparency and prevent corruption in political funding.
    Statement 2 is incorrect: Electoral bonds are strictly intended for funding registered political parties and cannot be used to directly finance individual and independent candidates. This restriction aims to prevent the use of electoral bonds for personal gain or undue influence over candidates.
    Statement 3 is correct: The government has designated specific branches of the SBI as authorized sellers of electoral bonds. This centralized approach helps to monitor and regulate the issuance of electoral bonds and maintain a record of transactions.
    Additional Information:
    ● The electoral bonds system was introduced in 2017 by way of a Finance bill and it was implemented in 2018.
    ● State Bank of India (SBI) issues the bonds in denominations of Rs 1,000, Rs 10,000, Rs 1 lakh, Rs 10 lakh, and Rs 1 crore which are payable to the bearer on demand and interest-free.
    ● Can be purchased by Indian citizens or entities established in India.
    ● They are valid for 15 calendar days from the date of issue.
    ● SBI is the authorized issuer.
    ● Only the political parties registered under Section 29A of the Representation of the People Act, 1951 and have secured not less than 1% of the votes polled in the last general election to the House of the People or the Legislative Assembly, are eligible to receive electoral bonds.

  2. Question 2 of 5
    2. Question

    2. Which of the following are the purposes of including the Directive Principles of State Policy (DPSP) in the Constitution of India?
    1. Socio-economic development
    2. Establishing political democracy
    3. Creation of a welfare state
    4. Serve as a moral compass for governance
    Select the correct answer from the code given below:

    Correct

    Answer: D
    Explanation:
    Statement 1 is correct: To guide the government’s approach to socio-economic development and welfare: The DPSP outlines a vision for a just and equitable society, providing direction for the government’s policies and programs aimed at promoting social and economic well-being. These principles advocate for measures such as providing adequate means of livelihood, securing the right to work and education, and promoting social justice and equal opportunities.
    Statement 2 is incorrect: While the Directive Principles of State Policy (DPSP) do have some implications for political democracy, their primary focus is on establishing social and economic democracy.
    Political democracy refers to a system of government in which power is held by the people, either directly through voting or indirectly through elected representatives. It is characterized by features such as free and fair elections, freedom of speech and assembly, and the rule of law.
    Social and economic democracy, on the other hand, is concerned with creating a society in which all citizens have equal opportunities and a fair share of the nation’s resources. It encompasses concepts such as social justice, equitable distribution of wealth, and ensuring the basic needs of all citizens.
    The DPSP, enshrined in Part IV of the Indian Constitution, primarily focuses on promoting social and economic democracy. They outline principles such as providing adequate means of livelihood, securing the right to work and education, and promoting social justice and equal opportunities. These principles aim to create a society where all citizens have equal access to opportunities and resources, regardless of their social background or economic status.
    Statement 3 is correct: To promote a welfare state: The DPSP embodies the concept of a welfare state, emphasizing the government’s responsibility to ensure the well-being of its citizens. These principles encourage the government to take an active role in providing social services, such as healthcare, education, and social security, to uplift the lives of its citizens.
    Statement 4 is correct: To serve as a moral compass for governance: The DPSP serves as guiding principles for the government’s actions, providing a moral framework for policymaking and decision-making. These principles encourage the government to consider not only immediate economic and political considerations but also the long-term social and moral implications of its actions.
    Basics of Directive Principles of State Policy:
    ● Part IV of the Indian Constitution: The DPSP are enshrined in Part IV of the Indian Constitution, along with the Fundamental Rights. These principles are considered to be “fundamental in the governance of the country” and “it shall be the duty of the State to apply these principles in making laws.”
    ● Non-justiciable nature: While the DPSP are considered important guiding principles, they are not justiciable. This means that individuals cannot directly approach courts to seek enforcement of these principles. However, the DPSP can be used by courts as a source of inspiration and interpretation when adjudicating cases related to Fundamental Rights.
    ● Role in policymaking: The DPSP serve as a moral compass for the government’s policies and programs. While not directly enforceable, these principles provide a framework for the government to strive towards achieving a just, equitable, and welfare-oriented society.

    Incorrect

    Answer: D
    Explanation:
    Statement 1 is correct: To guide the government’s approach to socio-economic development and welfare: The DPSP outlines a vision for a just and equitable society, providing direction for the government’s policies and programs aimed at promoting social and economic well-being. These principles advocate for measures such as providing adequate means of livelihood, securing the right to work and education, and promoting social justice and equal opportunities.
    Statement 2 is incorrect: While the Directive Principles of State Policy (DPSP) do have some implications for political democracy, their primary focus is on establishing social and economic democracy.
    Political democracy refers to a system of government in which power is held by the people, either directly through voting or indirectly through elected representatives. It is characterized by features such as free and fair elections, freedom of speech and assembly, and the rule of law.
    Social and economic democracy, on the other hand, is concerned with creating a society in which all citizens have equal opportunities and a fair share of the nation’s resources. It encompasses concepts such as social justice, equitable distribution of wealth, and ensuring the basic needs of all citizens.
    The DPSP, enshrined in Part IV of the Indian Constitution, primarily focuses on promoting social and economic democracy. They outline principles such as providing adequate means of livelihood, securing the right to work and education, and promoting social justice and equal opportunities. These principles aim to create a society where all citizens have equal access to opportunities and resources, regardless of their social background or economic status.
    Statement 3 is correct: To promote a welfare state: The DPSP embodies the concept of a welfare state, emphasizing the government’s responsibility to ensure the well-being of its citizens. These principles encourage the government to take an active role in providing social services, such as healthcare, education, and social security, to uplift the lives of its citizens.
    Statement 4 is correct: To serve as a moral compass for governance: The DPSP serves as guiding principles for the government’s actions, providing a moral framework for policymaking and decision-making. These principles encourage the government to consider not only immediate economic and political considerations but also the long-term social and moral implications of its actions.
    Basics of Directive Principles of State Policy:
    ● Part IV of the Indian Constitution: The DPSP are enshrined in Part IV of the Indian Constitution, along with the Fundamental Rights. These principles are considered to be “fundamental in the governance of the country” and “it shall be the duty of the State to apply these principles in making laws.”
    ● Non-justiciable nature: While the DPSP are considered important guiding principles, they are not justiciable. This means that individuals cannot directly approach courts to seek enforcement of these principles. However, the DPSP can be used by courts as a source of inspiration and interpretation when adjudicating cases related to Fundamental Rights.
    ● Role in policymaking: The DPSP serve as a moral compass for the government’s policies and programs. While not directly enforceable, these principles provide a framework for the government to strive towards achieving a just, equitable, and welfare-oriented society.

  3. Question 3 of 5
    3. Question

    3. Tenth schedule to the Constitution of India was added by:

    Correct

    Answer: B
    Explanation: Option B is correct.
    The Tenth Schedule to the Constitution of India was added by the Constitution (Fifty-second Amendment) Act, 1985 which introduced the concept of anti-defection. The aim of the Tenth Schedule is to prevent members of legislatures from switching parties for personal gain or political expediency, thereby promoting stability and coherence in the legislative process.
    Prior to the Fifty-second Amendment, there was no constitutional provision against defection. This led to frequent instances of members of legislatures switching parties, often in exchange for personal benefits or ministerial positions. This instability and lack of loyalty to party principles undermined the integrity of the legislative process and raised concerns about the functioning of democracy.
    45th Amendment Act, 1980:extended the period of reservation of seats for the Scheduled Castes and Scheduled Tribes
    54th Amendment Act, 1986:enhanced the salaries of Judges of the High Courts and Supreme Court of India.
    59th Amendment Act, 1988:It empowered the President of India to impose President’s rule in Punjab for a period of up to three years, instead of the usual one year.

    Incorrect

    Answer: B
    Explanation: Option B is correct.
    The Tenth Schedule to the Constitution of India was added by the Constitution (Fifty-second Amendment) Act, 1985 which introduced the concept of anti-defection. The aim of the Tenth Schedule is to prevent members of legislatures from switching parties for personal gain or political expediency, thereby promoting stability and coherence in the legislative process.
    Prior to the Fifty-second Amendment, there was no constitutional provision against defection. This led to frequent instances of members of legislatures switching parties, often in exchange for personal benefits or ministerial positions. This instability and lack of loyalty to party principles undermined the integrity of the legislative process and raised concerns about the functioning of democracy.
    45th Amendment Act, 1980:extended the period of reservation of seats for the Scheduled Castes and Scheduled Tribes
    54th Amendment Act, 1986:enhanced the salaries of Judges of the High Courts and Supreme Court of India.
    59th Amendment Act, 1988:It empowered the President of India to impose President’s rule in Punjab for a period of up to three years, instead of the usual one year.

  4. Question 4 of 5
    4. Question

    4. Consider the following statements:
    Statement-I: Article 300 of the Indian Constitution embodies the concept of constitutional tort.
    Statement-II: Article 300 permits the Union and the State Governments to be sued as juristic persons.
    Which one of the following is correct in respect of the above statements?

    Correct

    Answer: A
    Explanation:
    A “constitutional tort” refers to the violation of an individual’s constitutional rights by a state actor or government official. It’s a situation where a person’s rights, as granted by a nation’s constitution or other foundational legal document, are infringed upon by those acting on behalf of the government.
    In such situations, the aggrieved individual may have the right to sue the state actor or the government itself for damages.
    The concept of constitutional tort aims to provide a remedy for violations of constitutional rights and to hold state actors accountable for their actions.
    Statement-I is correct: In the context of India, the term “constitutional tort” isn’t as commonly used as in some other jurisdictions, but the underlying idea is present within the Indian Constitution and legal system.
    Constitutional tort in India refers to the breach of an individual’s fundamental rights enshrined in the Constitution by a state actor or a public official. When the government or its agents act in a way that infringes upon an individual’s rights, the affected person can seek redressal through the judiciary.
    The Indian Constitution, under Article 32 and Article 226, empowers individuals to move the Supreme Court and High Courts respectively for the enforcement of their fundamental rights. In doing so, the courts often provide remedies and compensations, holding public officials accountable for their actions. Article 300 of the Constitution of India also embodies the concept of Constitutional tort.
    Article 300 of the Indian Constitution ensures that the government is subject to the rule of law and can be involved in legal proceedings to resolve disputes or address claims related to its rights and liabilities. It reinforces the principle that no one, including the government, is above the law.
    Statement-II is correct: Under Article 300(1) of the Constitution of India the Government of India may sue or be sued by the name of the Union of India and the Government of a State may sue or be sued by the name of the State. This is a fundamental principle of Indian law that ensures that the government is accountable to the people.
    Both Statement-I and Statement-II are correct and Statement-II is the correct explanation for Statement-I

    Incorrect

    Answer: A
    Explanation:
    A “constitutional tort” refers to the violation of an individual’s constitutional rights by a state actor or government official. It’s a situation where a person’s rights, as granted by a nation’s constitution or other foundational legal document, are infringed upon by those acting on behalf of the government.
    In such situations, the aggrieved individual may have the right to sue the state actor or the government itself for damages.
    The concept of constitutional tort aims to provide a remedy for violations of constitutional rights and to hold state actors accountable for their actions.
    Statement-I is correct: In the context of India, the term “constitutional tort” isn’t as commonly used as in some other jurisdictions, but the underlying idea is present within the Indian Constitution and legal system.
    Constitutional tort in India refers to the breach of an individual’s fundamental rights enshrined in the Constitution by a state actor or a public official. When the government or its agents act in a way that infringes upon an individual’s rights, the affected person can seek redressal through the judiciary.
    The Indian Constitution, under Article 32 and Article 226, empowers individuals to move the Supreme Court and High Courts respectively for the enforcement of their fundamental rights. In doing so, the courts often provide remedies and compensations, holding public officials accountable for their actions. Article 300 of the Constitution of India also embodies the concept of Constitutional tort.
    Article 300 of the Indian Constitution ensures that the government is subject to the rule of law and can be involved in legal proceedings to resolve disputes or address claims related to its rights and liabilities. It reinforces the principle that no one, including the government, is above the law.
    Statement-II is correct: Under Article 300(1) of the Constitution of India the Government of India may sue or be sued by the name of the Union of India and the Government of a State may sue or be sued by the name of the State. This is a fundamental principle of Indian law that ensures that the government is accountable to the people.
    Both Statement-I and Statement-II are correct and Statement-II is the correct explanation for Statement-I

  5. Question 5 of 5
    5. Question

    5. Consider the following:
    1. India Innovation Index (III)
    2. School Education Quality Index (SEQI)
    3. State Health Index (SHI)
    How many of the above given indices are released by the NITI Aayog?

    Correct

    Answer: C
    Explanation: All the above mentioned Indices are released by the NITI Aayog.
    NITI Aayog, the Government of India’s think tank, releases a variety of indexes to assess and track the progress of various sectors and aspects of development in India.
    India Innovation Index (III): The India Innovation Index, released annually, measures the innovation ecosystem across states and Union Territories in India. It assesses various aspects of innovation, including finance, human capital, research and development, and infrastructure.
    The India Innovation Index (III) is based on a comprehensive framework that encompasses seven pillars of innovation:
    1. Human Capital: This pillar assesses the skills and capabilities of the workforce, including education, research and development, and creative talent.
    2. Infrastructure: This pillar evaluates the availability and quality of physical and digital infrastructure, including transportation, communication, and IT infrastructure.
    3. Finance: This pillar assesses the access to capital for innovation activities, including venture capital, angel investments, and government funding.
    4. Market Access: This pillar evaluates the ease of doing business, market size, and regulatory environment for innovative products and services.
    5. Knowledge Creation: This pillar assesses the output of innovation activities, including research and development, publications, and patents.
    6. Business Environment: This pillar evaluates the overall business environment for innovation, including factors such as intellectual property protection, regulatory environment, and government support for innovation.
    7. Performance: This pillar assesses the commercialization of innovation, including venture capital deals, startups, and global rankings.
    Key Findings of the India Innovation Index (III) 2022:
    ● Karnataka topped the index for the third consecutive year.
    ● Karnataka was followed by Tamil Nadu, Maharashtra, Telangana, and Gujarat.
    ● Uttarakhand, Bihar, and Jharkhand were among the bottom-ranked states.
    ● The gap between the top-performing and bottom-performing states has narrowed in recent years.

    School Education Quality Index (SEQI): The School Education Quality Index, released every two years, measures the quality of school education across states and Union Territories in India. It assesses parameters such as learning outcomes, infrastructure, teacher qualifications, and school management.The SEQI is based on a comprehensive framework that encompasses 70 indicators across five key dimensions:
    1. Outcomes: This dimension assesses the learning outcomes of students, including their performance in national and international assessments.
    2. Infrastructure and Facilities: This dimension evaluates the availability and quality of physical infrastructure, such as classrooms, libraries, and laboratories, as well as teacher-student ratios.
    3. Teacher Availability and Quality: This dimension assesses the number, qualifications, and professional development of teachers.
    4. Processes and Governance: This dimension evaluates the effectiveness of school management, school governance, and teacher management practices.
    5. Equity and Access: This dimension assesses the accessibility of education for all students, including those from marginalized communities and disadvantaged backgrounds.

    State Health Index (SHI): The State Health Index, released every three years, evaluates the performance of public health systems across states and Union Territories in India. It assesses factors such as health outcomes, infrastructure, access to services, and financial resources to provide a holistic evaluation of the healthcare delivery system and identify areas for improvement.
    The SHI is based on a comprehensive framework that encompasses 24 indicators across three key dimensions:
    1. Health Outcomes: This dimension assesses the overall health status of the population, including indicators such as infant mortality rate, maternal mortality rate, and life expectancy.
    2. Governance and Information: This dimension evaluates the effectiveness of the healthcare system’s governance, including factors such as health budget allocation, utilization of health services, and availability of health information.
    3. Key Inputs and Processes: This dimension assesses the availability and quality of key health inputs, such as healthcare infrastructure, human resources, and access to essential medicines.

    Incorrect

    Answer: C
    Explanation: All the above mentioned Indices are released by the NITI Aayog.
    NITI Aayog, the Government of India’s think tank, releases a variety of indexes to assess and track the progress of various sectors and aspects of development in India.
    India Innovation Index (III): The India Innovation Index, released annually, measures the innovation ecosystem across states and Union Territories in India. It assesses various aspects of innovation, including finance, human capital, research and development, and infrastructure.
    The India Innovation Index (III) is based on a comprehensive framework that encompasses seven pillars of innovation:
    1. Human Capital: This pillar assesses the skills and capabilities of the workforce, including education, research and development, and creative talent.
    2. Infrastructure: This pillar evaluates the availability and quality of physical and digital infrastructure, including transportation, communication, and IT infrastructure.
    3. Finance: This pillar assesses the access to capital for innovation activities, including venture capital, angel investments, and government funding.
    4. Market Access: This pillar evaluates the ease of doing business, market size, and regulatory environment for innovative products and services.
    5. Knowledge Creation: This pillar assesses the output of innovation activities, including research and development, publications, and patents.
    6. Business Environment: This pillar evaluates the overall business environment for innovation, including factors such as intellectual property protection, regulatory environment, and government support for innovation.
    7. Performance: This pillar assesses the commercialization of innovation, including venture capital deals, startups, and global rankings.
    Key Findings of the India Innovation Index (III) 2022:
    ● Karnataka topped the index for the third consecutive year.
    ● Karnataka was followed by Tamil Nadu, Maharashtra, Telangana, and Gujarat.
    ● Uttarakhand, Bihar, and Jharkhand were among the bottom-ranked states.
    ● The gap between the top-performing and bottom-performing states has narrowed in recent years.

    School Education Quality Index (SEQI): The School Education Quality Index, released every two years, measures the quality of school education across states and Union Territories in India. It assesses parameters such as learning outcomes, infrastructure, teacher qualifications, and school management.The SEQI is based on a comprehensive framework that encompasses 70 indicators across five key dimensions:
    1. Outcomes: This dimension assesses the learning outcomes of students, including their performance in national and international assessments.
    2. Infrastructure and Facilities: This dimension evaluates the availability and quality of physical infrastructure, such as classrooms, libraries, and laboratories, as well as teacher-student ratios.
    3. Teacher Availability and Quality: This dimension assesses the number, qualifications, and professional development of teachers.
    4. Processes and Governance: This dimension evaluates the effectiveness of school management, school governance, and teacher management practices.
    5. Equity and Access: This dimension assesses the accessibility of education for all students, including those from marginalized communities and disadvantaged backgrounds.

    State Health Index (SHI): The State Health Index, released every three years, evaluates the performance of public health systems across states and Union Territories in India. It assesses factors such as health outcomes, infrastructure, access to services, and financial resources to provide a holistic evaluation of the healthcare delivery system and identify areas for improvement.
    The SHI is based on a comprehensive framework that encompasses 24 indicators across three key dimensions:
    1. Health Outcomes: This dimension assesses the overall health status of the population, including indicators such as infant mortality rate, maternal mortality rate, and life expectancy.
    2. Governance and Information: This dimension evaluates the effectiveness of the healthcare system’s governance, including factors such as health budget allocation, utilization of health services, and availability of health information.
    3. Key Inputs and Processes: This dimension assesses the availability and quality of key health inputs, such as healthcare infrastructure, human resources, and access to essential medicines.

window.wpAdvQuizInitList = window.wpAdvQuizInitList || []; window.wpAdvQuizInitList.push({ id: '#wpAdvQuiz_601', init: { quizId: 601, mode: 0, globalPoints: 10, timelimit: 0, resultsGrade: [0], bo: 0, qpp: 0, catPoints: [10], formPos: 0, lbn: "Finish quiz", json: {"2799":{"type":"single","id":2799,"catId":0,"points":2,"correct":[0,1,0,0]},"2800":{"type":"single","id":2800,"catId":0,"points":2,"correct":[0,0,0,1]},"2801":{"type":"single","id":2801,"catId":0,"points":2,"correct":[0,1,0,0]},"2802":{"type":"single","id":2802,"catId":0,"points":2,"correct":[1,0,0,0]},"2803":{"type":"single","id":2803,"catId":0,"points":2,"correct":[0,0,1,0]}} } });




Day-561 | Daily MCQs | UPSC Prelims | GEOGRAPHY

Day-561

Time limit: 0

Quiz-summary

0 of 5 questions completed

Questions:

  1. 1
  2. 2
  3. 3
  4. 4
  5. 5

Information

DAILY MCQ

You have already completed the quiz before. Hence you can not start it again.

Quiz is loading...

You must sign in or sign up to start the quiz.

You have to finish following quiz, to start this quiz:

Results

0 of 5 questions answered correctly

Your time:

Time has elapsed

You have reached 0 of 0 points, (0)

Categories

  1. Not categorized 0%
  1. 1
  2. 2
  3. 3
  4. 4
  5. 5
  1. Answered
  2. Review
  1. Question 1 of 5
    1. Question

    1. Consider the following pairs:
    Shifting cultivation – Practised in the region
    1. Milpa – Indonesia
    2. Ladang – Central America
    3. Jhuming – North Eastern India
    4. Taungya – Sri Lanka
    How many pairs given above are correctly matched?

    Correct

    Answer: A
    Explanation:
    Primitive subsistence agriculture or shifting cultivation is widely practised by many tribes in the tropics, especially in Africa, south and Central America and South East Asia.

    The vegetation is usually cleared by fire, and the ashes add to the fertility of the soil. Shifting cultivation is thus, also called slash and burn agriculture.
    The cultivated patches are very small and cultivation is done with very primitive tools such as sticks and hoes.
    After sometime (3 to 5 years) the soil looses its fertility and the farmer shifts to another parts and clears other patch of the forest for cultivation.
    The farmer may return to the earlier patch after sometime.
    One of the major problems of shifting cultivation is that the cycle of jhum becomes less and less due to loss of fertility in different parcels.
    It is prevalent in tropical region in different names:
    Jhuming in North eastern states of India
    Milpa in central America and Mexico
    Ladang in Indonesia and Malaysia
    Taungya in Myanmar
    Chena in Sri Lanka
    Caigin in the Philippines

    Incorrect

    Answer: A
    Explanation:
    Primitive subsistence agriculture or shifting cultivation is widely practised by many tribes in the tropics, especially in Africa, south and Central America and South East Asia.

    The vegetation is usually cleared by fire, and the ashes add to the fertility of the soil. Shifting cultivation is thus, also called slash and burn agriculture.
    The cultivated patches are very small and cultivation is done with very primitive tools such as sticks and hoes.
    After sometime (3 to 5 years) the soil looses its fertility and the farmer shifts to another parts and clears other patch of the forest for cultivation.
    The farmer may return to the earlier patch after sometime.
    One of the major problems of shifting cultivation is that the cycle of jhum becomes less and less due to loss of fertility in different parcels.
    It is prevalent in tropical region in different names:
    Jhuming in North eastern states of India
    Milpa in central America and Mexico
    Ladang in Indonesia and Malaysia
    Taungya in Myanmar
    Chena in Sri Lanka
    Caigin in the Philippines

  2. Question 2 of 5
    2. Question

    2. In the context of mining activities in India, consider the following pairs:
    Iron ore mines – States
    1. Gurumahisani – West Bengal
    2. Noamundi – Jharkhand
    3. Badampahar – Odisha
    4. Dalli-Rajhara – Rajasthan
    How many pairs given above are correctly matched?

    Correct

    Answer: B
    Explanation: India is endowed with fairly abundant resources of iron ore. It has the largest reserve of iron ore in Asia. The two main types of ore found in our country are haematite and magnetite. The iron ore mines occur in close proximity to the coal fields in the northeastern plateau region of the country which adds to their advantage.
    About 95 per cent of total reserves of iron ore are located in the States of Odisha, Jharkhand, Chhattisgarh, Karnataka, Goa, Telangana, Andhra Pradesh and Tamil Nadu.
    Pair 1 is matched incorrectly and Pair 3 is matched correctly: In Odisha, iron ore occurs in a series of hill ranges in Sundergarh, Mayurbhanj and Jhar.
    The important mines are Gurumahisani, Sulaipet, Badampahar (Mayurbhaj), Kiruburu (Kendujhar) and Bonai (Sundergarh).
    Pair 2 is matched correctly: Jharkhand has some of the oldest iron ore mines and most of the iron and steel plants are located around them. Most of the important mines such as Noamundi and Gua are located in Poorbi and Pashchimi Singhbhum districts.
    Pair 4 is matched incorrectly: Dalli-Rajhara mines are in Chhattisgarh.

    Additional Information
    Apart from these sources, iron ore is extracted from following sources also:
    In Chattisgarh major iron ore mining areas are Durg, Dantewara and Bailadila. Dalli, and Rajhara in Durg are the important mines of iron ore in the country.
    In Karnataka, iron ore deposits occur in Sandur -Hospet area of Ballari district, Baba Budan hills and Kudremukh in Chikkamagaluru district and parts of Shivamogga, Chitradurg and Tumakuru districts.
    The districts of Chandrapur, Bhandara and Ratnagiri in Maharashtra, Karimnagar and Warangal district of Telangana, Kurnool, Cuddapah and Anantapur districts of Andhra Pradesh, Salem and Nilgiris districts of Tamil Nadu are other iron mining regions.
    Goa has also emerged as an important producer of iron ore.

    Incorrect

    Answer: B
    Explanation: India is endowed with fairly abundant resources of iron ore. It has the largest reserve of iron ore in Asia. The two main types of ore found in our country are haematite and magnetite. The iron ore mines occur in close proximity to the coal fields in the northeastern plateau region of the country which adds to their advantage.
    About 95 per cent of total reserves of iron ore are located in the States of Odisha, Jharkhand, Chhattisgarh, Karnataka, Goa, Telangana, Andhra Pradesh and Tamil Nadu.
    Pair 1 is matched incorrectly and Pair 3 is matched correctly: In Odisha, iron ore occurs in a series of hill ranges in Sundergarh, Mayurbhanj and Jhar.
    The important mines are Gurumahisani, Sulaipet, Badampahar (Mayurbhaj), Kiruburu (Kendujhar) and Bonai (Sundergarh).
    Pair 2 is matched correctly: Jharkhand has some of the oldest iron ore mines and most of the iron and steel plants are located around them. Most of the important mines such as Noamundi and Gua are located in Poorbi and Pashchimi Singhbhum districts.
    Pair 4 is matched incorrectly: Dalli-Rajhara mines are in Chhattisgarh.

    Additional Information
    Apart from these sources, iron ore is extracted from following sources also:
    In Chattisgarh major iron ore mining areas are Durg, Dantewara and Bailadila. Dalli, and Rajhara in Durg are the important mines of iron ore in the country.
    In Karnataka, iron ore deposits occur in Sandur -Hospet area of Ballari district, Baba Budan hills and Kudremukh in Chikkamagaluru district and parts of Shivamogga, Chitradurg and Tumakuru districts.
    The districts of Chandrapur, Bhandara and Ratnagiri in Maharashtra, Karimnagar and Warangal district of Telangana, Kurnool, Cuddapah and Anantapur districts of Andhra Pradesh, Salem and Nilgiris districts of Tamil Nadu are other iron mining regions.
    Goa has also emerged as an important producer of iron ore.

  3. Question 3 of 5
    3. Question

    3. Lachin Corridor, seen in the news recently, is related to which one of the following?

    Correct

    Answer: B
    Explanation:
    The Lachin corridor is a mountain road in Azerbaijan that links Armenia and Nagorno-Karabakh.
    Being the only road between these two territories, it is considered a humanitarian corridor or “lifeline” to the Armenian population of Nagorno-Karabakh.
    The corridor is in the Lachin District of Azerbaijan, but is ostensibly under the control of a Russian peacekeeping force as provided for in the 2020 Nagorno-Karabakh armistice agreement.

    Incorrect

    Answer: B
    Explanation:
    The Lachin corridor is a mountain road in Azerbaijan that links Armenia and Nagorno-Karabakh.
    Being the only road between these two territories, it is considered a humanitarian corridor or “lifeline” to the Armenian population of Nagorno-Karabakh.
    The corridor is in the Lachin District of Azerbaijan, but is ostensibly under the control of a Russian peacekeeping force as provided for in the 2020 Nagorno-Karabakh armistice agreement.

  4. Question 4 of 5
    4. Question

    4. Consider the following peninsular rivers:
    1. Subarnarekha
    2. Sharavati
    3. Brahmani
    4. Pennar
    How many of the rivers given above flow from east to west?

    Correct

    Answer: A
    Explanation: Only Sharavati River flows towards the west. All other are east flowing rivers given here.
    The Western Ghats, which run close to the western coast, serve as a water divide between the major Peninsular Rivers, which discharge their water into the Bay of Bengal, and the small rivulets joining the Arabian Sea.
    Most of the major Peninsular Rivers except Narmada and Tapi flow from west to east.

    The rivers flowing towards the Arabian Sea have short courses.
    The Sharavati is one such river, which originates in the Shimoga district of Karnataka and drains a catchment area of 2,209 sq. km. The total length of the river is around 128 km, and it joins the Arabian Sea at Honnavar in Uttara Kannada district. On its way, the Sharavati forms the Jog Falls, where the river falls from a height of 253 metres.
    There are a number of small rivers that join the Bay of Bengal. The Subarnrekha, the Baitarni, the Brahmani, the Vamsadhara, the Pennar, the Palar, and the Vaigai are important east flowing rivers.

    Incorrect

    Answer: A
    Explanation: Only Sharavati River flows towards the west. All other are east flowing rivers given here.
    The Western Ghats, which run close to the western coast, serve as a water divide between the major Peninsular Rivers, which discharge their water into the Bay of Bengal, and the small rivulets joining the Arabian Sea.
    Most of the major Peninsular Rivers except Narmada and Tapi flow from west to east.

    The rivers flowing towards the Arabian Sea have short courses.
    The Sharavati is one such river, which originates in the Shimoga district of Karnataka and drains a catchment area of 2,209 sq. km. The total length of the river is around 128 km, and it joins the Arabian Sea at Honnavar in Uttara Kannada district. On its way, the Sharavati forms the Jog Falls, where the river falls from a height of 253 metres.
    There are a number of small rivers that join the Bay of Bengal. The Subarnrekha, the Baitarni, the Brahmani, the Vamsadhara, the Pennar, the Palar, and the Vaigai are important east flowing rivers.

  5. Question 5 of 5
    5. Question

    5. Consider the following regions:
    1. Mumbai-Pune region
    2. Vishakhapatnam-Guntur region
    3. Chhotanagpur region
    4. Ambala-Amritsar region
    How many of the above regions are the major industrial regions in India?

    Correct

    Answer: C
    Explanation:
    Industries are not evenly distributed in the country. They tend to concentrate on certain locations because of the favorable locational factors.

    Major industrial regions of the country are given below (8):
    Mumbai-Pune Region
    Hugli Region
    Bengaluru-Tamil Nadu Region
    Gujarat Region
    Chotanagpur Region
    Vishakhapatnam-Guntur Region
    Gurugram-Delhi-Meerut Region, and
    Kollam-Thiruvananthapuram Region
    Minor Industrial Regions (13):
    Ambala-Amritsar
    Saharanpur-Muzzaffarnagar-Bijnor
    Indore-Dewas-Ujjain
    Jaipur-Ajmer
    Kolhapur-South Kannada
    Northern Malabar
    Middle Malabar
    Adilabad-Nizamabad
    Allahabad-Varanasi-Mirzapur
    Bhojpur-Munger
    Bilaspur-Korba
    Durg-Raipur
    Brahmaputra Valley

    Incorrect

    Answer: C
    Explanation:
    Industries are not evenly distributed in the country. They tend to concentrate on certain locations because of the favorable locational factors.

    Major industrial regions of the country are given below (8):
    Mumbai-Pune Region
    Hugli Region
    Bengaluru-Tamil Nadu Region
    Gujarat Region
    Chotanagpur Region
    Vishakhapatnam-Guntur Region
    Gurugram-Delhi-Meerut Region, and
    Kollam-Thiruvananthapuram Region
    Minor Industrial Regions (13):
    Ambala-Amritsar
    Saharanpur-Muzzaffarnagar-Bijnor
    Indore-Dewas-Ujjain
    Jaipur-Ajmer
    Kolhapur-South Kannada
    Northern Malabar
    Middle Malabar
    Adilabad-Nizamabad
    Allahabad-Varanasi-Mirzapur
    Bhojpur-Munger
    Bilaspur-Korba
    Durg-Raipur
    Brahmaputra Valley

window.wpAdvQuizInitList = window.wpAdvQuizInitList || []; window.wpAdvQuizInitList.push({ id: '#wpAdvQuiz_600', init: { quizId: 600, mode: 0, globalPoints: 10, timelimit: 0, resultsGrade: [0], bo: 0, qpp: 0, catPoints: [10], formPos: 0, lbn: "Finish quiz", json: {"2794":{"type":"single","id":2794,"catId":0,"points":2,"correct":[1,0,0,0]},"2795":{"type":"single","id":2795,"catId":0,"points":2,"correct":[0,1,0,0]},"2796":{"type":"single","id":2796,"catId":0,"points":2,"correct":[0,1,0,0]},"2797":{"type":"single","id":2797,"catId":0,"points":2,"correct":[1,0,0,0]},"2798":{"type":"single","id":2798,"catId":0,"points":2,"correct":[0,0,1,0]}} } });




Day-560 | Daily MCQs | UPSC Prelims | ECONOMICS

Day-560

Time limit: 0

Quiz-summary

0 of 5 questions completed

Questions:

  1. 1
  2. 2
  3. 3
  4. 4
  5. 5

Information

DAILY MCQ

You have already completed the quiz before. Hence you can not start it again.

Quiz is loading...

You must sign in or sign up to start the quiz.

You have to finish following quiz, to start this quiz:

Results

0 of 5 questions answered correctly

Your time:

Time has elapsed

You have reached 0 of 0 points, (0)

Categories

  1. Not categorized 0%
  1. 1
  2. 2
  3. 3
  4. 4
  5. 5
  1. Answered
  2. Review
  1. Question 1 of 5
    1. Question

    1. With reference to the Open Market Sale Scheme (OMSS), consider the following statements:
    1. The purpose of OMSS is to moderate the prices of foodgrains in the open market.
    2. It is administered by the Ministry of Agriculture & Farmers Welfare.
    3. Weekly e-auctions are conducted by the Food Corporation of India (FCI).
    4. State Governments are also allowed to participate in the auction.
    How many of the statements given above are correct?

    Correct

    Answer: B
    Explanation:
    Statement 1 is correct: The purpose of OMSS is to dispose of surplus stocks of wheat and rice held by FCI. This moderates the prices in the open market
    Statement 2 is incorrect: It is administered by the Ministry of Consumer Affairs, Food and Public Distribution.
    Statement 3 is correct: FCI conducts weekly auctions for the OMSS for wheat on the platform of the National Commodity and Derivatives Exchange Limited (NCDEX).
    Statement 4 is incorrect: States were previously allowed to procure food grains through the OMSS without participating in the auctions, for their needs.
    However, the Centre has recently discontinued the OMSS for state governments. It has given three reasons for the discontinuation of OMSS for State governments:
    ● It claims that the OMSS can be more successful in curbing the inflationary trend in food grains if the grain is released through the market rather than through States/the PDS.
    ● The government argued that it needs to maintain adequate food grain stock in the central pool as it expects lower procurement this year. This is because the output of some crops has been hit due to adverse weather events like untimely rains and high temperatures.
    ● The central government has cited its obligation towards consumers who are not covered by the NFSA but are affected by fluctuations in retail prices of food grains. It argued that while the state governments will allocate food grains to the NFSA beneficiaries, as well as for beneficiaries of state-specific schemes, the interest of general consumers has been ignored.
    Additional information:
    Open Market Sale Scheme (OMSS):-
    It was launched to enhance the supply of food grains (ensuring food security) during the lean season and thereby moderate the open market prices (controlling inflation), especially in the deficit regions.
    Salient Features of OMSS:-
    ● Under the OMSS, the FCI from time to time sells surplus food grains from the central pool,
    especially wheat and rice in the open market to traders, bulk consumers, retail chains, etc., at predetermined prices.
    ● The FCI does this through e-auctions where open market bidders can buy specified quantities.
    ● FCI conducts weekly auctions for the OMSS for wheat on the platform of the National Commodity and Derivatives Exchange Limited (NCDEX).
    ● NCDEX: a commodity exchange platform in India that provides a platform for trading in various agricultural and other commodities.
    ● States are also allowed to procure food grains through the OMSS without participating in the auctions, for their needs.

    Incorrect

    Answer: B
    Explanation:
    Statement 1 is correct: The purpose of OMSS is to dispose of surplus stocks of wheat and rice held by FCI. This moderates the prices in the open market
    Statement 2 is incorrect: It is administered by the Ministry of Consumer Affairs, Food and Public Distribution.
    Statement 3 is correct: FCI conducts weekly auctions for the OMSS for wheat on the platform of the National Commodity and Derivatives Exchange Limited (NCDEX).
    Statement 4 is incorrect: States were previously allowed to procure food grains through the OMSS without participating in the auctions, for their needs.
    However, the Centre has recently discontinued the OMSS for state governments. It has given three reasons for the discontinuation of OMSS for State governments:
    ● It claims that the OMSS can be more successful in curbing the inflationary trend in food grains if the grain is released through the market rather than through States/the PDS.
    ● The government argued that it needs to maintain adequate food grain stock in the central pool as it expects lower procurement this year. This is because the output of some crops has been hit due to adverse weather events like untimely rains and high temperatures.
    ● The central government has cited its obligation towards consumers who are not covered by the NFSA but are affected by fluctuations in retail prices of food grains. It argued that while the state governments will allocate food grains to the NFSA beneficiaries, as well as for beneficiaries of state-specific schemes, the interest of general consumers has been ignored.
    Additional information:
    Open Market Sale Scheme (OMSS):-
    It was launched to enhance the supply of food grains (ensuring food security) during the lean season and thereby moderate the open market prices (controlling inflation), especially in the deficit regions.
    Salient Features of OMSS:-
    ● Under the OMSS, the FCI from time to time sells surplus food grains from the central pool,
    especially wheat and rice in the open market to traders, bulk consumers, retail chains, etc., at predetermined prices.
    ● The FCI does this through e-auctions where open market bidders can buy specified quantities.
    ● FCI conducts weekly auctions for the OMSS for wheat on the platform of the National Commodity and Derivatives Exchange Limited (NCDEX).
    ● NCDEX: a commodity exchange platform in India that provides a platform for trading in various agricultural and other commodities.
    ● States are also allowed to procure food grains through the OMSS without participating in the auctions, for their needs.

  2. Question 2 of 5
    2. Question

    2. Consider the following statements:
    1. High levels of Current Account Deficit can lead to inflation in the economy.
    2. High surpluses in the Capital Account can depreciate the domestic currency.
    Which of the statements given above are correct?

    Correct

    Answer: A
    Explanation:
    Statement 1 is correct: A large and persistent Current Account Deficit (CAD) can make a country vulnerable to economic instability due to uncomfortable values of imports This can result in a sharp drop in the value of the country’s currency, making imports more expensive and leading to inflation. For example, Pakistan and Sri Lanka are experiencing high inflation in the domestic economy due to high levels of Current Account Deficit.
    Statement 2 is incorrect: High surpluses in the Capital Account means more foreign exchange reserves are coming into the country in the form of foreign investments and borrowings by the government. This increases the demand for local currency which leads to its appreciation.

    Incorrect

    Answer: A
    Explanation:
    Statement 1 is correct: A large and persistent Current Account Deficit (CAD) can make a country vulnerable to economic instability due to uncomfortable values of imports This can result in a sharp drop in the value of the country’s currency, making imports more expensive and leading to inflation. For example, Pakistan and Sri Lanka are experiencing high inflation in the domestic economy due to high levels of Current Account Deficit.
    Statement 2 is incorrect: High surpluses in the Capital Account means more foreign exchange reserves are coming into the country in the form of foreign investments and borrowings by the government. This increases the demand for local currency which leads to its appreciation.

  3. Question 3 of 5
    3. Question

    3. Consider the following statements:
    Statement I: Headline inflation is more volatile than core inflation.
    Statement II: Core inflation excludes inflation in food and fuel.
    Which one of the following is correct in respect of the above statements?

    Correct

    Answer: A
    Explanation:
    Statement 1 is correct: Headline Inflation is the measure of total inflation within an economy. It includes price rise in food, fuel and all other commodities. But Core inflation does not consider the inflation in food and fuel which are more volatile components. Thus, Headline inflation is more volatile than core inflation.
    Statement 2 is correct and is the correct explanation of statement 1: Since inflation in food and fuel is excluded in the core inflation. This makes it more stable as compared to headline inflation. Food and fuel prices are governed by international as well as domestic production capacities which can fluctuate more frequently.
    Additional information:
    ● Headline inflation is not adjusted to remove highly volatile figures, including those that can shift regardless of economic conditions.
    ● Headline inflation is often closely related to shifts in the cost of living, which provides useful information to consumers within the marketplace.
    ● The headline figure is not adjusted for seasonality or for the often-volatile elements of food and energy prices, which are removed in the core Consumer Price Index (CPI).
    Core Inflation
    ● Core inflation is the change in the costs of goods and services but does not include those from the food and energy sectors.
    ● This measure of inflation excludes these items because their prices are much more volatile.

    Incorrect

    Answer: A
    Explanation:
    Statement 1 is correct: Headline Inflation is the measure of total inflation within an economy. It includes price rise in food, fuel and all other commodities. But Core inflation does not consider the inflation in food and fuel which are more volatile components. Thus, Headline inflation is more volatile than core inflation.
    Statement 2 is correct and is the correct explanation of statement 1: Since inflation in food and fuel is excluded in the core inflation. This makes it more stable as compared to headline inflation. Food and fuel prices are governed by international as well as domestic production capacities which can fluctuate more frequently.
    Additional information:
    ● Headline inflation is not adjusted to remove highly volatile figures, including those that can shift regardless of economic conditions.
    ● Headline inflation is often closely related to shifts in the cost of living, which provides useful information to consumers within the marketplace.
    ● The headline figure is not adjusted for seasonality or for the often-volatile elements of food and energy prices, which are removed in the core Consumer Price Index (CPI).
    Core Inflation
    ● Core inflation is the change in the costs of goods and services but does not include those from the food and energy sectors.
    ● This measure of inflation excludes these items because their prices are much more volatile.

  4. Question 4 of 5
    4. Question

    4. Consider the following:
    1. Individual farmers who are owners/cultivators
    2. Tenant farmers
    3. Self-Help Groups of sharecroppers
    4. Farmers involved in animal husbandry
    5. Fish farmers
    How many of the above are eligible for credit support from the banking system under the Kisan Credit Card scheme?

    Correct

    Answer: D
    Explanation:
    Eligible beneficiaries under the scheme for credit support from the banking system:
    ● Individual farmers who are owners/cultivators
    ● Sharecroppers, tenant farmers
    ● Self-Help Groups of sharecroppers, farmers, tenant farmers etc
    ● Farmers involved in the production of crops or activities such as animal husbandry.
    ● Fish farmers, fishers, SHGs, JLGs, and women groups
    ● Fishermen who own a registered boat or any other type of fishing vessel and possess the necessary license or permissions for fishing in estuaries or the sea.
    ● Poultry farmers and even those who raise sheep, rabbits, goats pigs etc.
    ● Dairy: Farmers, dairy farmers, SHGs, JLGs, and tenant farmers who own, lease, or rent sheds.
    Additional information:
    ● The scheme was introduced in 1998 for providing adequate and timely credit support from the banking system, under a single window with flexible and simplified procedures to the farmers for their cultivation and other needs like purchase of agriculture inputs such as seeds, fertilizers, pesticides etc. and draw cash for their production needs.
    ● The scheme was further extended for the investment credit requirement of farmers viz. allied and non-farm activities in the year 2004.
    ● In the Budget-2018-19, government announced the extension of the facility of Kisan Credit Card (KCC) to fisheries and animal husbandry farmers to help them to meet their working capital needs.
    Implementing Agencies: Commercial Banks; Regional Rural Banks (RRBs); Small Finance Banks; Cooperatives
    Objectives:
    ● To meet the short-term credit requirements for cultivation of crops.
    ● Post-harvest expenses.
    ● Produce marketing loan.
    ● Consumption requirements of farmer households.
    ● Working capital for maintenance of farm assets and activities allied to agriculture.
    ● Investment credit requirement for agriculture and allied activities.

    Incorrect

    Answer: D
    Explanation:
    Eligible beneficiaries under the scheme for credit support from the banking system:
    ● Individual farmers who are owners/cultivators
    ● Sharecroppers, tenant farmers
    ● Self-Help Groups of sharecroppers, farmers, tenant farmers etc
    ● Farmers involved in the production of crops or activities such as animal husbandry.
    ● Fish farmers, fishers, SHGs, JLGs, and women groups
    ● Fishermen who own a registered boat or any other type of fishing vessel and possess the necessary license or permissions for fishing in estuaries or the sea.
    ● Poultry farmers and even those who raise sheep, rabbits, goats pigs etc.
    ● Dairy: Farmers, dairy farmers, SHGs, JLGs, and tenant farmers who own, lease, or rent sheds.
    Additional information:
    ● The scheme was introduced in 1998 for providing adequate and timely credit support from the banking system, under a single window with flexible and simplified procedures to the farmers for their cultivation and other needs like purchase of agriculture inputs such as seeds, fertilizers, pesticides etc. and draw cash for their production needs.
    ● The scheme was further extended for the investment credit requirement of farmers viz. allied and non-farm activities in the year 2004.
    ● In the Budget-2018-19, government announced the extension of the facility of Kisan Credit Card (KCC) to fisheries and animal husbandry farmers to help them to meet their working capital needs.
    Implementing Agencies: Commercial Banks; Regional Rural Banks (RRBs); Small Finance Banks; Cooperatives
    Objectives:
    ● To meet the short-term credit requirements for cultivation of crops.
    ● Post-harvest expenses.
    ● Produce marketing loan.
    ● Consumption requirements of farmer households.
    ● Working capital for maintenance of farm assets and activities allied to agriculture.
    ● Investment credit requirement for agriculture and allied activities.

  5. Question 5 of 5
    5. Question

    5. Consider the following statements regarding the expansionary monetary policy:
    1. It can result in a decrease in bond prices.
    2. It can lead to the depreciation of local currency.
    3. It can boost employment in the economy.
    How many of the above statements are correct?

    Correct

    Answer: B
    Explanation:
    Statement 1 is incorrect: Expansionary monetary policy results in a reduction in the bank deposit interest rates. When the rate of interest provided by banks keeps falling, bonds which provide a fixed interest rate for a longer duration will become more attractive. This may drive up the demand for bonds and thus may result in an increase in bond prices.
    Statement 2 is correct: Expansionary monetary policy can result in depreciation of currency. This is because of more supply of local currency as compared to foreign currency.
    Statement 3 is correct: Since there is more liquidity in the market, this lowers the borrowing cost. Thus, it boosts consumer spending. As demand for goods and services rises, more goods and services are produced which boosts employment.
    Additional information:
    Expansionary monetary policy is implemented by lowering key interest rates thus increasing market liquidity (money supply). High market liquidity usually encourages more economic activity.
    ● Increase in Foreign bond prices: Even though the demands for bonds as such may increase, the lower interest rates may make domestic bonds less attractive. So, the demand for domestic bonds may fall and the demand for foreign bonds may rise.
    ● Increase in exports and BoP: A lower exchange rate may cause exports to increase, imports to decrease and the balance of trade to increase.
    ● Higher Capital Investment: Lower interest rates lead to higher levels of capital investment.
    When the RBI adopts the Expansionary Monetary Policy, the central bank.
    ● Decrease Policy Rates (Interest Rates) like Repo, Reverse Repo, MSF, Bank Rate etc.
    ● Decrease Reserve Ratios like Cash Reserve Ratio (CRR) and Statutory Liquidity Ratio (SLR)
    ● Buys government securities from the market as part of Open Market Operations (OMO) – providing liquidity in the market.

    Incorrect

    Answer: B
    Explanation:
    Statement 1 is incorrect: Expansionary monetary policy results in a reduction in the bank deposit interest rates. When the rate of interest provided by banks keeps falling, bonds which provide a fixed interest rate for a longer duration will become more attractive. This may drive up the demand for bonds and thus may result in an increase in bond prices.
    Statement 2 is correct: Expansionary monetary policy can result in depreciation of currency. This is because of more supply of local currency as compared to foreign currency.
    Statement 3 is correct: Since there is more liquidity in the market, this lowers the borrowing cost. Thus, it boosts consumer spending. As demand for goods and services rises, more goods and services are produced which boosts employment.
    Additional information:
    Expansionary monetary policy is implemented by lowering key interest rates thus increasing market liquidity (money supply). High market liquidity usually encourages more economic activity.
    ● Increase in Foreign bond prices: Even though the demands for bonds as such may increase, the lower interest rates may make domestic bonds less attractive. So, the demand for domestic bonds may fall and the demand for foreign bonds may rise.
    ● Increase in exports and BoP: A lower exchange rate may cause exports to increase, imports to decrease and the balance of trade to increase.
    ● Higher Capital Investment: Lower interest rates lead to higher levels of capital investment.
    When the RBI adopts the Expansionary Monetary Policy, the central bank.
    ● Decrease Policy Rates (Interest Rates) like Repo, Reverse Repo, MSF, Bank Rate etc.
    ● Decrease Reserve Ratios like Cash Reserve Ratio (CRR) and Statutory Liquidity Ratio (SLR)
    ● Buys government securities from the market as part of Open Market Operations (OMO) – providing liquidity in the market.

window.wpAdvQuizInitList = window.wpAdvQuizInitList || []; window.wpAdvQuizInitList.push({ id: '#wpAdvQuiz_599', init: { quizId: 599, mode: 0, globalPoints: 10, timelimit: 0, resultsGrade: [0], bo: 0, qpp: 0, catPoints: [10], formPos: 0, lbn: "Finish quiz", json: {"2789":{"type":"single","id":2789,"catId":0,"points":2,"correct":[0,1,0,0]},"2790":{"type":"single","id":2790,"catId":0,"points":2,"correct":[1,0,0,0]},"2791":{"type":"single","id":2791,"catId":0,"points":2,"correct":[1,0,0,0]},"2792":{"type":"single","id":2792,"catId":0,"points":2,"correct":[0,0,0,1]},"2793":{"type":"single","id":2793,"catId":0,"points":2,"correct":[0,1,0,0]}} } });




Day-559 | Daily MCQs | UPSC Prelims | HISTORY

Day-559

Time limit: 0

Quiz-summary

0 of 5 questions completed

Questions:

  1. 1
  2. 2
  3. 3
  4. 4
  5. 5

Information

DAILY MCQ

You have already completed the quiz before. Hence you can not start it again.

Quiz is loading...

You must sign in or sign up to start the quiz.

You have to finish following quiz, to start this quiz:

Results

0 of 5 questions answered correctly

Your time:

Time has elapsed

You have reached 0 of 0 points, (0)

Categories

  1. Not categorized 0%
  1. 1
  2. 2
  3. 3
  4. 4
  5. 5
  1. Answered
  2. Review
  1. Question 1 of 5
    1. Question

    1. Which of the following statements about the early kingdoms of Tamilakam is correct?

    Correct

    Answer: D
    Explanation:
    The advent of the early historical period in South India is generally dated to the 3rd century BCE. The three most important kingdoms of Tamilakam (the land between the Tirupati hills (Vengadam) and the southernmost tip of the peninsula) were the Cholas, Cheras and Pandayas.
    Option a is incorrect: The principality of the Cholas in the lower Kaveri valley corresponded roughly to modern Tanjore and Trichinopoly districts of Tamil Nadu, and had its capital at Uraiyur. The kingdom of the Pandyas in the valleys of the Tamraparni and Vaigai roughly correspond to modern Tirunelveli, Madurai, Ramnad districts, and south Travancore and had its capital at Madurai. The Cheras on the Kerala coast had their capital at Karuvur, also known as Vanji.
    Option b is incorrect: The premier Chola port was Puhar (also known as Kaveripumpattinam), the major Pandya port was Korkai, while Tondi and Muchiri were the important ports in the Chera kingdom.
    Option c is incorrect: The Chera, Chola, and Pandya kings were the vendar (crowned kings). These great kings had their special insignia of royalty such as the staff, drum, and umbrella. They also had specific emblems of power—the tiger, bow, and fish were the emblems of the Cholas, Cheras, and Pandyas respectively. Apart from the vendar, there were a number of chieftains known as velir. The Sangam poets also eulogize various chieftains such as Ay, Andiran, and Pari for their bravery and generosity.
    Option d is correct: The early Pandya kings included Nediyon, Palshalai Mudukudumi, and Nedunjeliyan. As per sources, king Nedunjeliyan is said to have defeated a confederacy of Cholas, Cheras, and five chieftains at a battle at Talaiyalanganam while he was still very young, in which the Chera king was taken prisoner.

    Incorrect

    Answer: D
    Explanation:
    The advent of the early historical period in South India is generally dated to the 3rd century BCE. The three most important kingdoms of Tamilakam (the land between the Tirupati hills (Vengadam) and the southernmost tip of the peninsula) were the Cholas, Cheras and Pandayas.
    Option a is incorrect: The principality of the Cholas in the lower Kaveri valley corresponded roughly to modern Tanjore and Trichinopoly districts of Tamil Nadu, and had its capital at Uraiyur. The kingdom of the Pandyas in the valleys of the Tamraparni and Vaigai roughly correspond to modern Tirunelveli, Madurai, Ramnad districts, and south Travancore and had its capital at Madurai. The Cheras on the Kerala coast had their capital at Karuvur, also known as Vanji.
    Option b is incorrect: The premier Chola port was Puhar (also known as Kaveripumpattinam), the major Pandya port was Korkai, while Tondi and Muchiri were the important ports in the Chera kingdom.
    Option c is incorrect: The Chera, Chola, and Pandya kings were the vendar (crowned kings). These great kings had their special insignia of royalty such as the staff, drum, and umbrella. They also had specific emblems of power—the tiger, bow, and fish were the emblems of the Cholas, Cheras, and Pandyas respectively. Apart from the vendar, there were a number of chieftains known as velir. The Sangam poets also eulogize various chieftains such as Ay, Andiran, and Pari for their bravery and generosity.
    Option d is correct: The early Pandya kings included Nediyon, Palshalai Mudukudumi, and Nedunjeliyan. As per sources, king Nedunjeliyan is said to have defeated a confederacy of Cholas, Cheras, and five chieftains at a battle at Talaiyalanganam while he was still very young, in which the Chera king was taken prisoner.

  2. Question 2 of 5
    2. Question

    2. Consider the following statements:
    Statement I: The consolidation of the Vijayanagara empire created a supra-local urban network that bolstered trade.
    Statement II: The increasing militarisation of the polity led to the rise of urban power groups.
    Which one of the following is correct in respect of the above statements?

    Correct

    Answer: A
    Explanation:
    The consolidation of the Vijayanagara empire integrated the entire Peninsula which led to migration of traders and artisans across the regions and created a supra-local urban network that bolstered trade. This fostered urban developments in Vijayanagara which were linked to several factors like militarisation, migration and trade and commerce. Hence, statements I and II are correct.
    Due to increasing militarisation of the polity, numerous fortified settlements emerged. Fortifications of mercantile towns established by powerful merchant organisations and armed presence within the temple precincts increased. The towns emerged as trading centres and often eroded the power of the older agrarian elites, leading to the rise of urban power groups that entered into alliances with new political forces. So, statement II is the correct explanation of I.

    Incorrect

    Answer: A
    Explanation:
    The consolidation of the Vijayanagara empire integrated the entire Peninsula which led to migration of traders and artisans across the regions and created a supra-local urban network that bolstered trade. This fostered urban developments in Vijayanagara which were linked to several factors like militarisation, migration and trade and commerce. Hence, statements I and II are correct.
    Due to increasing militarisation of the polity, numerous fortified settlements emerged. Fortifications of mercantile towns established by powerful merchant organisations and armed presence within the temple precincts increased. The towns emerged as trading centres and often eroded the power of the older agrarian elites, leading to the rise of urban power groups that entered into alliances with new political forces. So, statement II is the correct explanation of I.

  3. Question 3 of 5
    3. Question

    3. With reference to the history of art forms in India, consider the following statements:
    1. Nāṭyaśāstra is the first text which talks about the various Svaras (notes) used in singing.
    2. Vātsyāyana is the first author to talk about classification of Indian musical instruments.
    3. The Gāndharva saṅgita, one of the streams of music, was practised for salvation.
    How many statements given above are correct?

    Correct

    Answer: A
    Explanation:
    The origin of Indian music can be traced back to evidence from the Vedic period. During this period, the Vedic hymns were chanted and some of them were also set to tune and rhythm. The rhythmic recitations of Ṛigveda were known as Ṛcās.
    Sāmaveda is the compilation of these selected Ṛcās set to Svaras (notes). So, statement 1 is incorrect.
    Only three Svaras — Udātta, Anudātta and Svarita were used in Sāmagāna.
    Mantras chanted during religious activities with Svaras were known as Sāmagāna.
    Udātta was the sharp pitch, Anudātta was grave pitch and Svarita combined in itself the characteristics of both the pitches.
    Statement 2 is incorrect: Bharat’s Nāṭyaśāstra is the most important and pioneer work of music, dance and drama. This is the first text available on the classification of Indian Instruments.
    There are two streams of music known as Mārgi- and Deśi-.
    ● Mārgi- or Gāndharva saṅgita was practiced for salvation. So, statement 3 is correct.
    ● Deśi- Saṅgita which was further divided into classical, semi-classical, folk music, etc.

    Incorrect

    Answer: A
    Explanation:
    The origin of Indian music can be traced back to evidence from the Vedic period. During this period, the Vedic hymns were chanted and some of them were also set to tune and rhythm. The rhythmic recitations of Ṛigveda were known as Ṛcās.
    Sāmaveda is the compilation of these selected Ṛcās set to Svaras (notes). So, statement 1 is incorrect.
    Only three Svaras — Udātta, Anudātta and Svarita were used in Sāmagāna.
    Mantras chanted during religious activities with Svaras were known as Sāmagāna.
    Udātta was the sharp pitch, Anudātta was grave pitch and Svarita combined in itself the characteristics of both the pitches.
    Statement 2 is incorrect: Bharat’s Nāṭyaśāstra is the most important and pioneer work of music, dance and drama. This is the first text available on the classification of Indian Instruments.
    There are two streams of music known as Mārgi- and Deśi-.
    ● Mārgi- or Gāndharva saṅgita was practiced for salvation. So, statement 3 is correct.
    ● Deśi- Saṅgita which was further divided into classical, semi-classical, folk music, etc.

  4. Question 4 of 5
    4. Question

    4. Consider the following pairs:
    Communities associated
    with puppetries – States
    1. Are Kapu – Tamil Nadu
    2. Dayat – Kerala
    3. Ravanachhaya Bhat – Karnataka
    How many pairs given above are correctly matched?

    Correct

    Answer: D
    Explanation:
    The above mentioned communities are associated with Shadow Puppet Theatre Traditions of India.
    The communities which are distinctively associated with it are:
    ● Chamadyacha Bahulya Thakar, Maharashtra
    ● Tolu Bommalatta Killekyata or Are Kapu, Andhra Pradesh
    ● Togalu Gombeyatta Killekyata or Dayat, Karnataka
    ● Tolu Bommalattam Killekyata, Tamil Nadu
    ● Tolpava Kuthu Vellalachetti, Nair, Kerala
    ● Ravanachhaya Bhat, Odisha
    India has the richest variety of types and styles of shadow puppets. Shadow puppets are flat figures. They are cut out of leather, which has been treated to make it translucent. Shadow puppets are pressed against the screen with a strong source of light behind it. The manipulation between the light and the screen make silhouettes or colourful shadows, as the case may be, for the viewers who sit in front of the screen. This tradition of shadow puppets survives in Odisha, Kerala, Andhra Pradesh, Karnataka, Maharashtra and Tamil Nadu.

    Incorrect

    Answer: D
    Explanation:
    The above mentioned communities are associated with Shadow Puppet Theatre Traditions of India.
    The communities which are distinctively associated with it are:
    ● Chamadyacha Bahulya Thakar, Maharashtra
    ● Tolu Bommalatta Killekyata or Are Kapu, Andhra Pradesh
    ● Togalu Gombeyatta Killekyata or Dayat, Karnataka
    ● Tolu Bommalattam Killekyata, Tamil Nadu
    ● Tolpava Kuthu Vellalachetti, Nair, Kerala
    ● Ravanachhaya Bhat, Odisha
    India has the richest variety of types and styles of shadow puppets. Shadow puppets are flat figures. They are cut out of leather, which has been treated to make it translucent. Shadow puppets are pressed against the screen with a strong source of light behind it. The manipulation between the light and the screen make silhouettes or colourful shadows, as the case may be, for the viewers who sit in front of the screen. This tradition of shadow puppets survives in Odisha, Kerala, Andhra Pradesh, Karnataka, Maharashtra and Tamil Nadu.

  5. Question 5 of 5
    5. Question

    5. With reference to the cultural history of India, the ‘Hemadpanti style’ of temple architecture is associated with-

    Correct

    Answer: C
    Explanation:
    A number of temples were erected during Yadava period in Maharashtra up to 13-14th century C.E. There are twenty five Yadava temples in Dharashiv and Latur district. These temples are commonly known as ‘Hemadpanti temples’.
    Characteristic features of Yadava temple:
    ● Temples are not decorated on its exterior wall.
    ● Garbhagriha, antarala, sabhamandapa and mukhamandapa are the parts of temple.
    ● The sculptural ornamentation on the pillars and doorjamb is reduced.
    ● Sculpture of Ganesh was carved on Lalatbimba.
    ● These temples are commonly known as ‘Hemadpanti Temples’.
    ● Most temples are dedicated to Shiva.
    In the last quarter of the 12th century AD the Yadavas of Devagiri came into prominence. They had previously been ruling over Seunadesha (Khandesh) as feudatories of the Chalukyas of Kalyani. The founder of the family was Dridhaprahara, the son of Subahu. During their rule a peculiar style of architecture called Hemadpanti after Hemadri or Hemadpant, a minister of Mahadeva and Ramachandra came into vogue.

    Incorrect

    Answer: C
    Explanation:
    A number of temples were erected during Yadava period in Maharashtra up to 13-14th century C.E. There are twenty five Yadava temples in Dharashiv and Latur district. These temples are commonly known as ‘Hemadpanti temples’.
    Characteristic features of Yadava temple:
    ● Temples are not decorated on its exterior wall.
    ● Garbhagriha, antarala, sabhamandapa and mukhamandapa are the parts of temple.
    ● The sculptural ornamentation on the pillars and doorjamb is reduced.
    ● Sculpture of Ganesh was carved on Lalatbimba.
    ● These temples are commonly known as ‘Hemadpanti Temples’.
    ● Most temples are dedicated to Shiva.
    In the last quarter of the 12th century AD the Yadavas of Devagiri came into prominence. They had previously been ruling over Seunadesha (Khandesh) as feudatories of the Chalukyas of Kalyani. The founder of the family was Dridhaprahara, the son of Subahu. During their rule a peculiar style of architecture called Hemadpanti after Hemadri or Hemadpant, a minister of Mahadeva and Ramachandra came into vogue.

window.wpAdvQuizInitList = window.wpAdvQuizInitList || []; window.wpAdvQuizInitList.push({ id: '#wpAdvQuiz_598', init: { quizId: 598, mode: 0, globalPoints: 10, timelimit: 0, resultsGrade: [0], bo: 0, qpp: 0, catPoints: [10], formPos: 0, lbn: "Finish quiz", json: {"2784":{"type":"single","id":2784,"catId":0,"points":2,"correct":[0,0,0,1]},"2785":{"type":"single","id":2785,"catId":0,"points":2,"correct":[1,0,0,0]},"2786":{"type":"single","id":2786,"catId":0,"points":2,"correct":[1,0,0,0]},"2787":{"type":"single","id":2787,"catId":0,"points":2,"correct":[0,0,0,1]},"2788":{"type":"single","id":2788,"catId":0,"points":2,"correct":[0,0,1,0]}} } });




Day-558 | Daily MCQs | UPSC Prelims | ENVIRONMENT AND ECOLOGY

Day-558

Time limit: 0

Quiz-summary

0 of 5 questions completed

Questions:

  1. 1
  2. 2
  3. 3
  4. 4
  5. 5

Information

DAILY MCQ

You have already completed the quiz before. Hence you can not start it again.

Quiz is loading...

You must sign in or sign up to start the quiz.

You have to finish following quiz, to start this quiz:

Results

0 of 5 questions answered correctly

Your time:

Time has elapsed

You have reached 0 of 0 points, (0)

Categories

  1. Not categorized 0%
  1. 1
  2. 2
  3. 3
  4. 4
  5. 5
  1. Answered
  2. Review
  1. Question 1 of 5
    1. Question

    1. The ‘Central Asian Mammals Initiative’, recently seen in the news, is started by which one of the following organisations?

    Correct

    Answer: C
    Explanation:
    The Central Asian region harbours the largest intact and still interconnected grasslands worldwide. As such, it is of global importance for many migratory and nomadic mammals which rely on the vast steppe, desert and mountain ecosystems that enable the essential long-distance movements which ensure their survival. Mass migrants in this region include CMS-listed species such as the Saiga antelope, Mongolian gazelle and Khulan.
    The UN Convention on the Conservation of Migratory Species of Wild Animals (CMS), also known as the Bonn Convention, has strengthened its engagement in Central Asia to conserve migratory mammals, their habitats and the vital role they play in preserving intact ecosystems. The Central Asian Mammals Initiative (CAMI) (Resolution 11.24 (Rev.COP13)) and its associated Programme of Work were adopted by CMS Parties at the COP11 and revised at COP13 in Gandhinagar, India (February 2020).
    With an initiative to strengthen the conservation of Central Asian migratory mammals, CMS aims to provide a common framework to coordinate conservation activities in the region and to coherently address major threats to migratory species.
    Additional information about CMS
    • The UN Convention on the Conservation of Migratory Species of Wild Animals (CMS) was signed in 1979 and has been into force since 1983. It is an intergovernmental treaty under the UNEP- popularly known as Bonn Convention.
    • As the only global convention specializing in the conservation of (terrestrial, avian and marine) migratory species, their habitats and migration routes, CMS complements and co-operates with a number of other international organizations, NGOs and partners in the media as well as in the corporate sector.

    Incorrect

    Answer: C
    Explanation:
    The Central Asian region harbours the largest intact and still interconnected grasslands worldwide. As such, it is of global importance for many migratory and nomadic mammals which rely on the vast steppe, desert and mountain ecosystems that enable the essential long-distance movements which ensure their survival. Mass migrants in this region include CMS-listed species such as the Saiga antelope, Mongolian gazelle and Khulan.
    The UN Convention on the Conservation of Migratory Species of Wild Animals (CMS), also known as the Bonn Convention, has strengthened its engagement in Central Asia to conserve migratory mammals, their habitats and the vital role they play in preserving intact ecosystems. The Central Asian Mammals Initiative (CAMI) (Resolution 11.24 (Rev.COP13)) and its associated Programme of Work were adopted by CMS Parties at the COP11 and revised at COP13 in Gandhinagar, India (February 2020).
    With an initiative to strengthen the conservation of Central Asian migratory mammals, CMS aims to provide a common framework to coordinate conservation activities in the region and to coherently address major threats to migratory species.
    Additional information about CMS
    • The UN Convention on the Conservation of Migratory Species of Wild Animals (CMS) was signed in 1979 and has been into force since 1983. It is an intergovernmental treaty under the UNEP- popularly known as Bonn Convention.
    • As the only global convention specializing in the conservation of (terrestrial, avian and marine) migratory species, their habitats and migration routes, CMS complements and co-operates with a number of other international organizations, NGOs and partners in the media as well as in the corporate sector.

  2. Question 2 of 5
    2. Question

    2. Consider the following:
    1. Flapshell turtle
    2. Blackshell turtle
    3. Indian tent turtle
    How many of the above species of turtle are herbivores?

    Correct

    Answer: A
    Explanation:
    Option 3 is correct.
    Indian Tent Turtle: The species is native to India, Nepal and Bangladesh with three subspecies recorded from the region viz., P. t. tentoria, P. t. circumdata and P. t. flaviventer.
    ● P. t. tentoria occurs in peninsular India and is recorded from Orissa, Maharashtra, Andhra Pradesh, Assam and Madhya Pradesh.
    ● P. t. circumdata occurs in the western tributaries of Ganga and the rivers of Gujarat. It is found in Rajasthan, Madhya Pradesh, Uttar Pradesh and Gujarat.
    ● P. t. flaviventer occurs in the northern tributaries of Ganga and is recorded from Uttar Pradesh, Bihar, West Bengal and Assam.
    Its habitats include still water pools on river side and slow running water near the river banks. These are active swimmers and are mainly herbivorous.
    Due to the attractive appearance of the species, they are illegally traded in the pet market.

    Conservation Status:
    ● IUCN : Least concern
    ● Wildlife Protection Act, 1972 : Schedule I
    ● CITES : Appendix II
    Options 1 and 2 are incorrect: Both flapshell and black turtle are omnivores. They feed on aquatic plants and insects, frogs, shrimp, snails, fruits, flowers, grasses and seeds.
    Flapshell turtle
    • Flapshell turtle lives in shallow, stagnant waters in lakes, rivers, ponds and streams in India, Pakistan, Sri Lanka, Nepal, Bangladesh and Myanmar.
    • It is a freshwater species of turtle that is adapted to survive in drought conditions. It often burrows and moves from one water hole to another to avoid desiccation. It also undergoes aestivation to survive dry conditions.
    • It prefers swampy areas with soil and exposure to sunlight as common nesting sites. It is generally solitary and active during the day.
    • It is listed as Schedule I species in Wildlife (Protection) Act, 1972, thus it has been accorded the highest level of protection in India.
    Blackshell turtle
    • Black turtles (freshwater turtle species) are commonly found in the Indomalayan biogeographical realm in India, Nepal, Sri lanka, Myanmar and Bangladesh.
    • It is listed under Schedule II of Wildlife (Protection) Act, 1972.

    Incorrect

    Answer: A
    Explanation:
    Option 3 is correct.
    Indian Tent Turtle: The species is native to India, Nepal and Bangladesh with three subspecies recorded from the region viz., P. t. tentoria, P. t. circumdata and P. t. flaviventer.
    ● P. t. tentoria occurs in peninsular India and is recorded from Orissa, Maharashtra, Andhra Pradesh, Assam and Madhya Pradesh.
    ● P. t. circumdata occurs in the western tributaries of Ganga and the rivers of Gujarat. It is found in Rajasthan, Madhya Pradesh, Uttar Pradesh and Gujarat.
    ● P. t. flaviventer occurs in the northern tributaries of Ganga and is recorded from Uttar Pradesh, Bihar, West Bengal and Assam.
    Its habitats include still water pools on river side and slow running water near the river banks. These are active swimmers and are mainly herbivorous.
    Due to the attractive appearance of the species, they are illegally traded in the pet market.

    Conservation Status:
    ● IUCN : Least concern
    ● Wildlife Protection Act, 1972 : Schedule I
    ● CITES : Appendix II
    Options 1 and 2 are incorrect: Both flapshell and black turtle are omnivores. They feed on aquatic plants and insects, frogs, shrimp, snails, fruits, flowers, grasses and seeds.
    Flapshell turtle
    • Flapshell turtle lives in shallow, stagnant waters in lakes, rivers, ponds and streams in India, Pakistan, Sri Lanka, Nepal, Bangladesh and Myanmar.
    • It is a freshwater species of turtle that is adapted to survive in drought conditions. It often burrows and moves from one water hole to another to avoid desiccation. It also undergoes aestivation to survive dry conditions.
    • It prefers swampy areas with soil and exposure to sunlight as common nesting sites. It is generally solitary and active during the day.
    • It is listed as Schedule I species in Wildlife (Protection) Act, 1972, thus it has been accorded the highest level of protection in India.
    Blackshell turtle
    • Black turtles (freshwater turtle species) are commonly found in the Indomalayan biogeographical realm in India, Nepal, Sri lanka, Myanmar and Bangladesh.
    • It is listed under Schedule II of Wildlife (Protection) Act, 1972.

  3. Question 3 of 5
    3. Question

    3. The ‘Bridgetown Initiative’, often seen in the news, is concerned with:

    Correct

    Answer: A
    Explanation:
    The Bridgetown Initiative, led by Barbados, is a proposal to reform the world of development finance, particularly how rich countries help poor countries cope with and adapt to climate change.
    Barbados is one of the Caribbean’s most vulnerable nations to climate change. Hurricanes are more frequent and intense and the island is also at risk from rising sea levels, storm surges, coastal erosion, droughts and flooding. Now the government of Barbados, led by Mia Mottley, the island’s first woman prime minister, is championing a radical initiative that could transform how lending is made to developing nations in a climate crisis.
    The key demands of the Bridgetown Initiative are:
    1. LIQUIDITY SUPPORT
    ● U.N. member states should fast-track the transfer of $100 billion in so-called ‘Special Drawing Rights’, a monetary reserve currency, to programmes that support climate resilience and subsidise lending to low-income countries.
    ● The International Monetary Fund should also immediately suspend surcharges – additional interest payments imposed on heavily indebted borrowing countries – for two to three years.
    ● It should also restore “enhanced access limits” established during the COVID pandemic for two emergency financial support instruments, the Rapid Credit Facility (RCF) and Rapid Financing Instruments.
    2. DEBT SUSTAINABILITY
    ● G20 creditor countries should redesign their Common Framework for restructuring the debt of poor countries in default, notably by speeding up debt relief talks and allowing middle-income countries to access it.
    ● The IMF should encourage the restructuring of unsustainable debt in a way that is consistent across countries, and change the way it analyses the debt to incentivise investments that create future savings, such as those for climate adaptation.
    ● Public and private creditors should include disaster clauses in lending deals to allow countries to divert debt payments to disaster relief; and refinance high-interest and short-term debt with credit guarantees and longer maturities.
    ● U.N. member states should agree to raise $100 billion a year for a fund to help pay for the climate-related loss and damage suffered by developing countries.
    3. PRIVATE CAPITAL
    ● The IMF and multilateral development banks should offer $100 billion a year in currency risk guarantees to help drive private sector investment in projects that would help developing countries make the transition to a low-carbon economy.
    4. DEVELOPMENT LENDING
    ● The G20 and other shareholders of the World Bank, IMF and development institutions should fully implement the 2022 recommendations of a panel of experts aimed at boosting lending by the multilateral development banks.
    ● Increase the leveraging of the World Bank’s International Development Association, which provides concessional finance; fully fund its emergency support facility to $6 billion by end-2023; and scale up the IDA’s funding to $279 billion.
    ● Raise the access limits to concessional finance through the Poverty Reduction and Growth Trust and the Resilience & Sustainability Trust.
    ● Assess funding eligibility in light of a country’s vulnerability and provide low-cost, 50-year loans to help them invest in areas including climate resilience, water security, pandemic preparedness and access to renewable energy.
    ● Simplify and harmonise the way countries can apply to access loans across the world, and provide more support in the process. The international financial institutions should also finance development plans that help protect shared resources.

    Incorrect

    Answer: A
    Explanation:
    The Bridgetown Initiative, led by Barbados, is a proposal to reform the world of development finance, particularly how rich countries help poor countries cope with and adapt to climate change.
    Barbados is one of the Caribbean’s most vulnerable nations to climate change. Hurricanes are more frequent and intense and the island is also at risk from rising sea levels, storm surges, coastal erosion, droughts and flooding. Now the government of Barbados, led by Mia Mottley, the island’s first woman prime minister, is championing a radical initiative that could transform how lending is made to developing nations in a climate crisis.
    The key demands of the Bridgetown Initiative are:
    1. LIQUIDITY SUPPORT
    ● U.N. member states should fast-track the transfer of $100 billion in so-called ‘Special Drawing Rights’, a monetary reserve currency, to programmes that support climate resilience and subsidise lending to low-income countries.
    ● The International Monetary Fund should also immediately suspend surcharges – additional interest payments imposed on heavily indebted borrowing countries – for two to three years.
    ● It should also restore “enhanced access limits” established during the COVID pandemic for two emergency financial support instruments, the Rapid Credit Facility (RCF) and Rapid Financing Instruments.
    2. DEBT SUSTAINABILITY
    ● G20 creditor countries should redesign their Common Framework for restructuring the debt of poor countries in default, notably by speeding up debt relief talks and allowing middle-income countries to access it.
    ● The IMF should encourage the restructuring of unsustainable debt in a way that is consistent across countries, and change the way it analyses the debt to incentivise investments that create future savings, such as those for climate adaptation.
    ● Public and private creditors should include disaster clauses in lending deals to allow countries to divert debt payments to disaster relief; and refinance high-interest and short-term debt with credit guarantees and longer maturities.
    ● U.N. member states should agree to raise $100 billion a year for a fund to help pay for the climate-related loss and damage suffered by developing countries.
    3. PRIVATE CAPITAL
    ● The IMF and multilateral development banks should offer $100 billion a year in currency risk guarantees to help drive private sector investment in projects that would help developing countries make the transition to a low-carbon economy.
    4. DEVELOPMENT LENDING
    ● The G20 and other shareholders of the World Bank, IMF and development institutions should fully implement the 2022 recommendations of a panel of experts aimed at boosting lending by the multilateral development banks.
    ● Increase the leveraging of the World Bank’s International Development Association, which provides concessional finance; fully fund its emergency support facility to $6 billion by end-2023; and scale up the IDA’s funding to $279 billion.
    ● Raise the access limits to concessional finance through the Poverty Reduction and Growth Trust and the Resilience & Sustainability Trust.
    ● Assess funding eligibility in light of a country’s vulnerability and provide low-cost, 50-year loans to help them invest in areas including climate resilience, water security, pandemic preparedness and access to renewable energy.
    ● Simplify and harmonise the way countries can apply to access loans across the world, and provide more support in the process. The international financial institutions should also finance development plans that help protect shared resources.

  4. Question 4 of 5
    4. Question

    4. The buttonwood trees have drawn much attention in recent times in states like Goa. Which one of the following best explains the reason behind it?

    Correct

    Answer: D
    Explanation:
    Conocarpus, also known as buttonwood tree, is an invasive species like Prosopis juliflora and Lantana camara which raises much apprehension among environmentalists since the tree is used for urban greening in various parts of the country. It is renowned for its use in public spaces for its ornamental looks but is known to have negative effects on the local biodiversity.
    One of the major concerns is that Conocarpus trees have deep roots that can damage telecommunication lines, drainage systems, and freshwater systems.
    Secondly, the trees of this species flower in winter and spread pollen in nearby areas. It is learnt that this is causing diseases like cold, cough, asthma, allergy etc.
    With glossy, dark-green leaves, Conocarpus is an evergreen tree species. The animals do not find this fast-growing species as palatable enough.
    Additional information about this tree:
    ● It is a mangrove tree species that can grow well in acidic and alkaline soil conditions.
    ● It has high water-absorption capacity.
    ● There are two species of Conocarpus (buttonwood) trees, with several varieties and hybrids — Conocarpus erectus, which is more widely used in India, is native to South America, and C. lancifolius is native to East Africa.
    ● It is an evergreen tree species which is ideal for seashore plantation since it is highly tolerant of salt and drought conditions.
    ● The strong wood of buttonwood was used for firewood, cabinet-making (furniture) and charcoal-making.
    ● It propagates by seeds or stem cuttings.
    ● Serious pollen allergies and respiratory problems have been reported in the vicinity of their plantations, which has led to civic agencies taking action to ban them. The problems had already been noted in Kuwait and other parts of the Middle East, as well as in Karachi and Faizabad in Pakistan, when they started being used in greening projects in India.

    Incorrect

    Answer: D
    Explanation:
    Conocarpus, also known as buttonwood tree, is an invasive species like Prosopis juliflora and Lantana camara which raises much apprehension among environmentalists since the tree is used for urban greening in various parts of the country. It is renowned for its use in public spaces for its ornamental looks but is known to have negative effects on the local biodiversity.
    One of the major concerns is that Conocarpus trees have deep roots that can damage telecommunication lines, drainage systems, and freshwater systems.
    Secondly, the trees of this species flower in winter and spread pollen in nearby areas. It is learnt that this is causing diseases like cold, cough, asthma, allergy etc.
    With glossy, dark-green leaves, Conocarpus is an evergreen tree species. The animals do not find this fast-growing species as palatable enough.
    Additional information about this tree:
    ● It is a mangrove tree species that can grow well in acidic and alkaline soil conditions.
    ● It has high water-absorption capacity.
    ● There are two species of Conocarpus (buttonwood) trees, with several varieties and hybrids — Conocarpus erectus, which is more widely used in India, is native to South America, and C. lancifolius is native to East Africa.
    ● It is an evergreen tree species which is ideal for seashore plantation since it is highly tolerant of salt and drought conditions.
    ● The strong wood of buttonwood was used for firewood, cabinet-making (furniture) and charcoal-making.
    ● It propagates by seeds or stem cuttings.
    ● Serious pollen allergies and respiratory problems have been reported in the vicinity of their plantations, which has led to civic agencies taking action to ban them. The problems had already been noted in Kuwait and other parts of the Middle East, as well as in Karachi and Faizabad in Pakistan, when they started being used in greening projects in India.

  5. Question 5 of 5
    5. Question

    5. Consider the following:
    1. Squids
    2. Shrimps
    3. Saltwater crocodiles
    4. Porpoises
    5. Seals
    How many of the above are the examples of nekton species?

    Correct

    Answer: D
    Explanation:
    Options 1, 2, 3, 4 and 5 are correct.
    Animals that swim or move freely in the ocean are nektons. Nekton comes in all shapes and sizes. They live in shallow and deep ocean waters. Most nekton eat zooplankton, other nektons or they scavenge for waste.
    There are three types of nektons:
    The largest group of nektons are chordates and have bones or cartilage. This group includes:
    ● bony fish
    ● whales
    ● sharks
    ● turtles
    ● snakes
    ● eels
    ● porpoises
    ● dolphins
    ● seals
    Molluscan nekton are animals like octopus and squid.
    Arthropod nekton are animals like shrimp. Most arthropods are benthos or bottom dwelling organisms.

    Incorrect

    Answer: D
    Explanation:
    Options 1, 2, 3, 4 and 5 are correct.
    Animals that swim or move freely in the ocean are nektons. Nekton comes in all shapes and sizes. They live in shallow and deep ocean waters. Most nekton eat zooplankton, other nektons or they scavenge for waste.
    There are three types of nektons:
    The largest group of nektons are chordates and have bones or cartilage. This group includes:
    ● bony fish
    ● whales
    ● sharks
    ● turtles
    ● snakes
    ● eels
    ● porpoises
    ● dolphins
    ● seals
    Molluscan nekton are animals like octopus and squid.
    Arthropod nekton are animals like shrimp. Most arthropods are benthos or bottom dwelling organisms.

window.wpAdvQuizInitList = window.wpAdvQuizInitList || []; window.wpAdvQuizInitList.push({ id: '#wpAdvQuiz_597', init: { quizId: 597, mode: 0, globalPoints: 10, timelimit: 0, resultsGrade: [0], bo: 0, qpp: 0, catPoints: [10], formPos: 0, lbn: "Finish quiz", json: {"2779":{"type":"single","id":2779,"catId":0,"points":2,"correct":[0,0,1,0]},"2780":{"type":"single","id":2780,"catId":0,"points":2,"correct":[1,0,0,0]},"2781":{"type":"single","id":2781,"catId":0,"points":2,"correct":[1,0,0,0]},"2782":{"type":"single","id":2782,"catId":0,"points":2,"correct":[0,0,0,1]},"2783":{"type":"single","id":2783,"catId":0,"points":2,"correct":[0,0,0,1]}} } });




Day-557 | Daily MCQs | UPSC Prelims | CURRENT DEVELOPMENTS

Day-557

Time limit: 0

Quiz-summary

0 of 5 questions completed

Questions:

  1. 1
  2. 2
  3. 3
  4. 4
  5. 5

Information

DAILY MCQ

You have already completed the quiz before. Hence you can not start it again.

Quiz is loading...

You must sign in or sign up to start the quiz.

You have to finish following quiz, to start this quiz:

Results

0 of 5 questions answered correctly

Your time:

Time has elapsed

You have reached 0 of 0 points, (0)

Categories

  1. Not categorized 0%
  1. 1
  2. 2
  3. 3
  4. 4
  5. 5
  1. Answered
  2. Review
  1. Question 1 of 5
    1. Question

    1. With reference to the Kampala Declaration on climate change, consider the following statements:
    1. It is a pioneering framework adopted by 48 African nations to manage climate-induced migration.
    2. It was co-hosted by the Governments of Kenya and Uganda with support from the Office of the High Commissioner for Human Rights (OHCHR) and the United Nations Framework Convention on Climate Change (UNFCCC).
    Which of the statements given above is/are correct?

    Correct

    Answer: A
    Context: In August 2023, a total of 48 African countries have now agreed to adopt the Kampala Ministerial Declaration on Migration, Environment and Climate Change (KDMECC) to address the nexus of human mobility and climate change in the continent.
    Explanation:
    Statement 1 is correct: It is a pioneering framework adopted by 48 African nations to manage climate-induced migration.
    Statement 2 is incorrect: It was co-hosted by the Governments of Kenya and Uganda with support from the International Organization for Migration (IOM) and the United Nations Framework Convention on Climate Change (UNFCCC).
    Additional information:
    ● KDMECC was originally signed and agreed upon by 15 African states in Kampala, Uganda in July 2022.
    ● The Declaration is the first comprehensive, action-oriented framework led by Member States to address climate-induced mobility in a practical and effective manner.
    The KDMECC aims to address the challenges associated with climate-induced migration through several key objectives:
    ● Comprehensive Framework: Providing member states with a comprehensive framework to develop and implement policies and strategies addressing climate-induced migration.
    ● Leveraging Migration for Development: Encouraging the integration of migration into sustainable development strategies, ensuring it benefits both migrants and host communities.
    ● Inclusivity: Prioritizing the voices of vulnerable groups, including youth, women, and those in disadvantaged situations, to ensure their needs and concerns are central to the response.

    Incorrect

    Answer: A
    Context: In August 2023, a total of 48 African countries have now agreed to adopt the Kampala Ministerial Declaration on Migration, Environment and Climate Change (KDMECC) to address the nexus of human mobility and climate change in the continent.
    Explanation:
    Statement 1 is correct: It is a pioneering framework adopted by 48 African nations to manage climate-induced migration.
    Statement 2 is incorrect: It was co-hosted by the Governments of Kenya and Uganda with support from the International Organization for Migration (IOM) and the United Nations Framework Convention on Climate Change (UNFCCC).
    Additional information:
    ● KDMECC was originally signed and agreed upon by 15 African states in Kampala, Uganda in July 2022.
    ● The Declaration is the first comprehensive, action-oriented framework led by Member States to address climate-induced mobility in a practical and effective manner.
    The KDMECC aims to address the challenges associated with climate-induced migration through several key objectives:
    ● Comprehensive Framework: Providing member states with a comprehensive framework to develop and implement policies and strategies addressing climate-induced migration.
    ● Leveraging Migration for Development: Encouraging the integration of migration into sustainable development strategies, ensuring it benefits both migrants and host communities.
    ● Inclusivity: Prioritizing the voices of vulnerable groups, including youth, women, and those in disadvantaged situations, to ensure their needs and concerns are central to the response.

  2. Question 2 of 5
    2. Question

    2. With reference to the term ‘Medicane’, seen in news, consider the following statements:
    1. Medicanes are temperate cyclones originating in mediterranean region.
    2. Like tropical cyclones, Medicanes form in hot conditions at the end of summer.
    3. They are also formed over the Black Sea.
    How many of the statements given above are correct?

    Correct

    Answer: B
    Context: The flash flood that has killed thousands of people in Libya in September 2023 followed a “medicane”, a rare but destructive weather phenomenon that scientists believe will intensify in a warming world.
    Explanation:
    Statement 1 is incorrect: Medicanes are tropical cyclones originating in the Mediterranean region.
    Mediterranean cyclone or hurricane which is so-called Medicane are not rare phenomenon. Such large storms happen in the Mediterranean region every few years.
    Statement 2 is correct: Like tropical cyclones, medicanes form in hot conditions at the end of summer.
    Statement 3 is correct: Medicanes used to occur only in the Mediterranean Sea, but in the last two decades they have been formed over the Black Sea too.
    Additional information:
    ● Mediterranean tropical-like cyclones are commonly known as ‘Medicanes’ while a rapidly rotating storm system with a low-pressure centre, a closed low-level atmospheric circulation with strong winds and a spiral arrangement of thunderstorms are called ‘Hurricanes’.
    ● Medicanes are not formally classified as cyclones and are not monitored by any agency.
    ● National Oceanic and Atmospheric Administration (NOAA) and Hellenic National Meteorological Service are the governing agencies for Medicanes.
    ● The cyclones in the Mediterranean Sea do not form easily because it is dry but due to SST (sea surface temperature), it has caused floods in Italy, Spain and France.

    Incorrect

    Answer: B
    Context: The flash flood that has killed thousands of people in Libya in September 2023 followed a “medicane”, a rare but destructive weather phenomenon that scientists believe will intensify in a warming world.
    Explanation:
    Statement 1 is incorrect: Medicanes are tropical cyclones originating in the Mediterranean region.
    Mediterranean cyclone or hurricane which is so-called Medicane are not rare phenomenon. Such large storms happen in the Mediterranean region every few years.
    Statement 2 is correct: Like tropical cyclones, medicanes form in hot conditions at the end of summer.
    Statement 3 is correct: Medicanes used to occur only in the Mediterranean Sea, but in the last two decades they have been formed over the Black Sea too.
    Additional information:
    ● Mediterranean tropical-like cyclones are commonly known as ‘Medicanes’ while a rapidly rotating storm system with a low-pressure centre, a closed low-level atmospheric circulation with strong winds and a spiral arrangement of thunderstorms are called ‘Hurricanes’.
    ● Medicanes are not formally classified as cyclones and are not monitored by any agency.
    ● National Oceanic and Atmospheric Administration (NOAA) and Hellenic National Meteorological Service are the governing agencies for Medicanes.
    ● The cyclones in the Mediterranean Sea do not form easily because it is dry but due to SST (sea surface temperature), it has caused floods in Italy, Spain and France.

  3. Question 3 of 5
    3. Question

    3. Often seen in news, the term ‘Atlantic Declaration’ refers to:

    Correct

    Answer: C
    Context: The United Kingdom and the United States (US) President signed “The Atlantic Declaration: A Framework for a Twenty-First Century US-UK Economic Partnership” during the latter’s visit to Washington in June 2023.
    Explanation:
    ● Declared as the first of its kind, the declaration and its action plans are expected to promote a “new type of innovative partnership” to cover all areas of cooperation including technology, economy and trade.
    ● While deepening the trade and investment relations, the declaration is also expected to strengthen the UK-US cooperation in sectors like defence, health, space, and science.
    ● The new declaration highlights that the nature of national security is changing with economics, and it is becoming more intertwined.
    ● The US and the UK are facing new international challenges including from “authoritarian states” such as China and Russia, along with disruptive technologies, non-state actions and transnational issues like climate change.
    ● To deal with the emerging challenges, both countries have worked together to strengthen their resilience in trade and technology; and cooperated to expand the scope of their defence, security, and intelligence relations.

    Incorrect

    Answer: C
    Context: The United Kingdom and the United States (US) President signed “The Atlantic Declaration: A Framework for a Twenty-First Century US-UK Economic Partnership” during the latter’s visit to Washington in June 2023.
    Explanation:
    ● Declared as the first of its kind, the declaration and its action plans are expected to promote a “new type of innovative partnership” to cover all areas of cooperation including technology, economy and trade.
    ● While deepening the trade and investment relations, the declaration is also expected to strengthen the UK-US cooperation in sectors like defence, health, space, and science.
    ● The new declaration highlights that the nature of national security is changing with economics, and it is becoming more intertwined.
    ● The US and the UK are facing new international challenges including from “authoritarian states” such as China and Russia, along with disruptive technologies, non-state actions and transnational issues like climate change.
    ● To deal with the emerging challenges, both countries have worked together to strengthen their resilience in trade and technology; and cooperated to expand the scope of their defence, security, and intelligence relations.

  4. Question 4 of 5
    4. Question

    4. Consider the following statements regarding the International Organization of Supreme Audit Institutions (INTOSAI):
    1. It has a special consultative status with the Economic and Social Council (ECOSOC) of the United Nations.
    2. Its governing board meets annually to provide strategic leadership, stewardship, and continuity of INTOSAI activities.
    3. The CAG of India is a member of the Governing Board of the INTOSAI.
    How many of the statements given above are correct?

    Correct

    Answer: C
    Context: CAG of India inaugurated the International Organization of Supreme Audit Institutions’s (INTOSAI) 15th Knowledge Sharing Committee meeting at Abu Dhabi.
    Explanation:
    Statement 1 is correct: It has special consultative status with the Economic and Social Council (ECOSOC) of the United Nations.
    Statement 2 is correct: The INTOSAI Governing Board meets annually to provide strategic leadership, stewardship, and continuity of INTOSAI activities.
    Statement 3 is correct: The CAG of India is a member of the Governing Board of the INTOSAI.
    Additional information:
    ● The International Organization of Supreme Audit Institutions (INTOSAI) is an intergovernmental organization whose members are supreme audit institutions.
    ● Nearly every supreme audit institution in the world is a member of INTOSAI. Depending on the type of system used in their home country, the members of INTOSAI may be variously titled the Chief Financial Controller, the Office of the Comptroller General, the Office of the Auditor General, the Court of Accounts, or the Board of Audit.
    ● INTOSAI holds a triennial conference entitled the International Congress of Supreme Audit Institutions (INCOSAI).
    ● It publishes the quarterly International Journal of Government Auditing and publishes guidelines and international standards on auditing.

    Incorrect

    Answer: C
    Context: CAG of India inaugurated the International Organization of Supreme Audit Institutions’s (INTOSAI) 15th Knowledge Sharing Committee meeting at Abu Dhabi.
    Explanation:
    Statement 1 is correct: It has special consultative status with the Economic and Social Council (ECOSOC) of the United Nations.
    Statement 2 is correct: The INTOSAI Governing Board meets annually to provide strategic leadership, stewardship, and continuity of INTOSAI activities.
    Statement 3 is correct: The CAG of India is a member of the Governing Board of the INTOSAI.
    Additional information:
    ● The International Organization of Supreme Audit Institutions (INTOSAI) is an intergovernmental organization whose members are supreme audit institutions.
    ● Nearly every supreme audit institution in the world is a member of INTOSAI. Depending on the type of system used in their home country, the members of INTOSAI may be variously titled the Chief Financial Controller, the Office of the Comptroller General, the Office of the Auditor General, the Court of Accounts, or the Board of Audit.
    ● INTOSAI holds a triennial conference entitled the International Congress of Supreme Audit Institutions (INCOSAI).
    ● It publishes the quarterly International Journal of Government Auditing and publishes guidelines and international standards on auditing.

  5. Question 5 of 5
    5. Question

    5. Consider the following statements:
    1. India has notified a greater number of critical minerals than the USA to drive its clean energy push.
    2. The Geological Survey of India has been given the responsibility to periodically update the list of critical minerals for India.
    Which of the statements given above is/are correct?

    Correct

    Answer: D
    Context: The Indian government recently listed 30 minerals, including nickel, titanium, vanadium and tungsten as critical to drive its clean energy push.
    Explanation:
    Statement 1 is incorrect: India has notified 30 critical minerals. The US has declared 50 minerals critical in light of their role in national security or economic development. Japan has identified a set of 31 minerals as critical for their economy. The UK considers 18 minerals critical, EU (34) and Canada (31).
    Statement 2 is incorrect: Creation of Centre of Excellence for Critical Minerals (CECM) in the Ministry of Mines is recommended by the Expert Committee of Ministry of Mines. CECM has the responsibility to periodically update the list of critical minerals for India and notify the critical mineral strategy from time to time.
    Statement 3 is correct:
    Additional information:
    ● Expert Committe under Ministry of Mines has identified a set of 30 critical minerals for India.
    ● These are Antimony, Beryllium, Bismuth, Cobalt, Copper, Gallium, Germanium, Graphite, Hafnium, Indium, Lithium, Molybdenum, Niobium, Nickel, PGE, Phosphorous, Potash, REE, Rhenium, Silicon, Strontium, Tantalum, Tellurium, Tin, Titanium, Tungsten, Vanadium, Zirconium, Selenium and Cadmium.
    ● Top Producers: Chile, Indonesia, Congo, China, Australia and South Africa.

    Incorrect

    Answer: D
    Context: The Indian government recently listed 30 minerals, including nickel, titanium, vanadium and tungsten as critical to drive its clean energy push.
    Explanation:
    Statement 1 is incorrect: India has notified 30 critical minerals. The US has declared 50 minerals critical in light of their role in national security or economic development. Japan has identified a set of 31 minerals as critical for their economy. The UK considers 18 minerals critical, EU (34) and Canada (31).
    Statement 2 is incorrect: Creation of Centre of Excellence for Critical Minerals (CECM) in the Ministry of Mines is recommended by the Expert Committee of Ministry of Mines. CECM has the responsibility to periodically update the list of critical minerals for India and notify the critical mineral strategy from time to time.
    Statement 3 is correct:
    Additional information:
    ● Expert Committe under Ministry of Mines has identified a set of 30 critical minerals for India.
    ● These are Antimony, Beryllium, Bismuth, Cobalt, Copper, Gallium, Germanium, Graphite, Hafnium, Indium, Lithium, Molybdenum, Niobium, Nickel, PGE, Phosphorous, Potash, REE, Rhenium, Silicon, Strontium, Tantalum, Tellurium, Tin, Titanium, Tungsten, Vanadium, Zirconium, Selenium and Cadmium.
    ● Top Producers: Chile, Indonesia, Congo, China, Australia and South Africa.

window.wpAdvQuizInitList = window.wpAdvQuizInitList || []; window.wpAdvQuizInitList.push({ id: '#wpAdvQuiz_596', init: { quizId: 596, mode: 0, globalPoints: 10, timelimit: 0, resultsGrade: [0], bo: 0, qpp: 0, catPoints: [10], formPos: 0, lbn: "Finish quiz", json: {"2774":{"type":"single","id":2774,"catId":0,"points":2,"correct":[1,0,0,0]},"2775":{"type":"single","id":2775,"catId":0,"points":2,"correct":[0,1,0,0]},"2776":{"type":"single","id":2776,"catId":0,"points":2,"correct":[0,0,1,0]},"2777":{"type":"single","id":2777,"catId":0,"points":2,"correct":[0,0,1,0]},"2778":{"type":"single","id":2778,"catId":0,"points":2,"correct":[0,0,0,1]}} } });




Day-556 | Daily MCQs | UPSC Prelims | ECONOMICS

Day-556

Time limit: 0

Quiz-summary

0 of 5 questions completed

Questions:

  1. 1
  2. 2
  3. 3
  4. 4
  5. 5

Information

DAILY MCQ

You have already completed the quiz before. Hence you can not start it again.

Quiz is loading...

You must sign in or sign up to start the quiz.

You have to finish following quiz, to start this quiz:

Results

0 of 5 questions answered correctly

Your time:

Time has elapsed

You have reached 0 of 0 points, (0)

Categories

  1. Not categorized 0%
  1. 1
  2. 2
  3. 3
  4. 4
  5. 5
  1. Answered
  2. Review
  1. Question 1 of 5
    1. Question

    1. In the context of economics, which of the following statements correctly describes the ‘Lewis Model’?

    Correct

    Answer: A
    Explanation:
    • In 1954, economist William Arthur Lewis put forth the “Economic Development with Unlimited Supplies of Labor”. For this work Lewis won the Nobel Prize in Economics in 1979.
    • The crux of the model suggested that surplus labor in agriculture could be redirected to the manufacturing sector by offering wages just high enough to attract workers away from the farm.
    • This shift, in theory, would stimulate industrial growth, enhance productivity, and lead to economic development.
    Challenges in Implementation of Lewis Model in India:
    • Obstacles of Low Wages: Low wages and inadequate social security in urban manufacturing facilities fails to entice rural agricultural laborers to relocate, given the high costs of urban living, posing a hurdle to the implementation of the Lewis model.
    • Technological Shift in Manufacturing: Manufacturing industries are increasingly capital-intensive, relying on labor-displacing technologies like robotics and artificial intelligence.
    • This transition restricts the absorptive capacity of labor-intensive sectors to accommodate surplus agricultural workers.
    • Disguised Unemployment: India faces a scenario of disguised unemployment in the agricultural sector, where a surplus of workers is engaged in activities that do not significantly contribute to increased productivity or income.
    • This surplus labor situation complicates the transition of workers to other sectors.
    • Skill Mismatch: There exists a mismatch between the skills demanded by the industries and the skills possessed by the workforce.
    • The education system might not adequately prepare individuals for the demands of the modern job market, resulting in a skill gap that impedes labor absorption in industries.
    • Overemphasis on White-Collar Jobs: Societal perceptions often prioritize white-collar jobs over vocational or technical skills.
    • This bias against blue-collar work can limit the workforce available for skilled trade positions and technical jobs, affecting industrial growth.

    Incorrect

    Answer: A
    Explanation:
    • In 1954, economist William Arthur Lewis put forth the “Economic Development with Unlimited Supplies of Labor”. For this work Lewis won the Nobel Prize in Economics in 1979.
    • The crux of the model suggested that surplus labor in agriculture could be redirected to the manufacturing sector by offering wages just high enough to attract workers away from the farm.
    • This shift, in theory, would stimulate industrial growth, enhance productivity, and lead to economic development.
    Challenges in Implementation of Lewis Model in India:
    • Obstacles of Low Wages: Low wages and inadequate social security in urban manufacturing facilities fails to entice rural agricultural laborers to relocate, given the high costs of urban living, posing a hurdle to the implementation of the Lewis model.
    • Technological Shift in Manufacturing: Manufacturing industries are increasingly capital-intensive, relying on labor-displacing technologies like robotics and artificial intelligence.
    • This transition restricts the absorptive capacity of labor-intensive sectors to accommodate surplus agricultural workers.
    • Disguised Unemployment: India faces a scenario of disguised unemployment in the agricultural sector, where a surplus of workers is engaged in activities that do not significantly contribute to increased productivity or income.
    • This surplus labor situation complicates the transition of workers to other sectors.
    • Skill Mismatch: There exists a mismatch between the skills demanded by the industries and the skills possessed by the workforce.
    • The education system might not adequately prepare individuals for the demands of the modern job market, resulting in a skill gap that impedes labor absorption in industries.
    • Overemphasis on White-Collar Jobs: Societal perceptions often prioritize white-collar jobs over vocational or technical skills.
    • This bias against blue-collar work can limit the workforce available for skilled trade positions and technical jobs, affecting industrial growth.

  2. Question 2 of 5
    2. Question

    2. Consider the following statements:
    1. Mutual funds invest money in equities only.
    2. Mutual funds can reduce the volatility in return on investment.
    3. Infrastructure Investment Trust (InvIT) and Real estate Investment Trust (ReITs) are similar to mutual funds.
    How many of the statements given above are correct?

    Correct

    Answer: B
    Explanation:
    Statement 1 is incorrect: Mutual funds collect money from investors and invest the money, on their behalf, in different securities (debt, equity or both).
    Statement 2 is correct: Equity mutual funds pool money from multiple investors to create a diversified portfolio of stocks. Due to risk diversification, the volatility in returns generated from mutual funds investments are generally lower than that of direct equity investing.
    Statement 3 is correct: Infrastructure investment Trust (InvIt) and Real estate Investment Trust(ReITs) are similar to mutual funds. This is because both REITs and InvITs pool money from a large number of investors and invest money on physical assets to generate income which is distributed as dividend.
    Additional information:
    ● An equity mutual fund is a type of mutual fund that primarily invests in stocks or equities. It allows individual investors to pool their money together and invest in a diversified portfolio of stocks selected and managed by professional fund managers.
    ● The objective of equity mutual funds is to provide investors with the opportunity to participate in the potential returns and dividends generated by a wide range of publicly traded companies.
    ● Equity funds are actively managed by experienced fund managers who conduct research and analysis to make investment decisions. These professionals aim to select stocks that they believe will generate positive returns for investors.

    Incorrect

    Answer: B
    Explanation:
    Statement 1 is incorrect: Mutual funds collect money from investors and invest the money, on their behalf, in different securities (debt, equity or both).
    Statement 2 is correct: Equity mutual funds pool money from multiple investors to create a diversified portfolio of stocks. Due to risk diversification, the volatility in returns generated from mutual funds investments are generally lower than that of direct equity investing.
    Statement 3 is correct: Infrastructure investment Trust (InvIt) and Real estate Investment Trust(ReITs) are similar to mutual funds. This is because both REITs and InvITs pool money from a large number of investors and invest money on physical assets to generate income which is distributed as dividend.
    Additional information:
    ● An equity mutual fund is a type of mutual fund that primarily invests in stocks or equities. It allows individual investors to pool their money together and invest in a diversified portfolio of stocks selected and managed by professional fund managers.
    ● The objective of equity mutual funds is to provide investors with the opportunity to participate in the potential returns and dividends generated by a wide range of publicly traded companies.
    ● Equity funds are actively managed by experienced fund managers who conduct research and analysis to make investment decisions. These professionals aim to select stocks that they believe will generate positive returns for investors.

  3. Question 3 of 5
    3. Question

    3. With reference to various curves in economics, consider the following pairs:
    Economics curves – Relationships between variables
    1. Laffer Curve – Inflation rate and Unemployment rate
    2. Phillips Curve – Tax revenue and Tax rate
    3. Lorenz Curve – National Income and Population Distribution
    How many of the above pairs are correctly matched?

    Correct

    Answer: A
    Explanation
    • Pair 1 is incorrect: Laffer curve explains the relationship between tax revenue and tax rate. It states that at lower as well as higher rate of tax, the tax revenue is low but tax revenue is high at the optimal rate of tax. According to the Laffer curve, if tax rates are increased above a certain level, then tax revenues can actually fall because higher tax rates discourage people from working, also there is high tax evasion.
    • Pair 2 is incorrect: The Phillips curve is an economic theory that inflation and unemployment have a stable and inverse relationship. Developed by William Phillips, it claims that with economic growth comes inflation, which in turn should lead to more jobs and less unemployment.
    • Pair 3 is correct: Lorenz Curve is a graphical distribution of wealth. It shows the proportion of income earned by any given percentage of the population.

    Incorrect

    Answer: A
    Explanation
    • Pair 1 is incorrect: Laffer curve explains the relationship between tax revenue and tax rate. It states that at lower as well as higher rate of tax, the tax revenue is low but tax revenue is high at the optimal rate of tax. According to the Laffer curve, if tax rates are increased above a certain level, then tax revenues can actually fall because higher tax rates discourage people from working, also there is high tax evasion.
    • Pair 2 is incorrect: The Phillips curve is an economic theory that inflation and unemployment have a stable and inverse relationship. Developed by William Phillips, it claims that with economic growth comes inflation, which in turn should lead to more jobs and less unemployment.
    • Pair 3 is correct: Lorenz Curve is a graphical distribution of wealth. It shows the proportion of income earned by any given percentage of the population.

  4. Question 4 of 5
    4. Question

    4. Consider the following statements regarding Gross Domestic Product (GDP)and Gross Value Added (GVA):
    1. GDP gives the picture of an economy from the demand perspective while GVA reflects the state of economic activity from the supply side.
    2. GDP includes the value of intermediate goods whereas GVA does not.
    Which of the statements given above is/are correct?

    Correct

    Answer: A
    Explanation:
    • Statement 1 is correct: GVA is the value added to the product to enhance the various aspects of the product whereas GDP is the total amount of products produced in the country. While GDP gives the picture from the consumers’ side or demand perspective, GVA gives a picture of the state of economic activity from the producers’ side or supply side. Both measures need not match because of the difference in treatment of net taxes.
    • Statement 2 is incorrect: Both GDP as well as GVA do not include the value of intermediate goods. GDP is calculated by adding Net Taxes (taxes earned by the government – subsidies paid by the government) to the GVA. GDP fails to gauge the real economic scenario because a sharp increase in the output can be due to higher tax collections which could be on account of better compliance or coverage, rather than the real output situation. As a result, GVA is a more precise measure of the value added by each sector of the economy.
    Additional information:
    ● GDP is the sum of private consumption, gross investment in the economy, government investment, government spending and net foreign trade (the difference between exports and imports).
    ● GDP = private consumption + gross investment + government investment + government spending + (exports-imports)
    ● GDP measures the value of a country’s final goods and services (those purchased by the final user) generated in a specific time period (say, a quarter or a year). It includes all of the output produced within a country’s borders.
    GDP = Private consumption + gross investment + government investment + government spending + (exports – imports)
    ● Private Consumption Expenditure refers to the value of all goods and services purchased for consumption by households.
    ● Government Consumption Expenditure refers to the value of all goods and services purchased for consumption by the government.
    ● Gross Investment refers to the total value of all capital investments made in the economy.
    The GVA of a sector is defined as the value of output minus the value of its intermediary inputs. This “value added” is distributed among the primary production factors, labour and capital. By examining GVA growth, one may determine which sectors of the economy are doing well and which are struggling.
    Calculation of GVA
    ● GVA is the sum of a country’s GDP and net of subsidies and taxes in the economy at the macro level, according to national accounting.
    ● Gross Value Added = GDP + product subsidies – product taxes
    ● Previously, India measured GVA at ‘factor cost’ until a new methodology was implemented, in which GVA at ‘basic prices’ became the primary measure of economic output.
    ● GVA at basic prices will include production taxes and exclude production subsidies.
    ● GVA at factor cost included no taxes and excluded no subsidies

    Incorrect

    Answer: A
    Explanation:
    • Statement 1 is correct: GVA is the value added to the product to enhance the various aspects of the product whereas GDP is the total amount of products produced in the country. While GDP gives the picture from the consumers’ side or demand perspective, GVA gives a picture of the state of economic activity from the producers’ side or supply side. Both measures need not match because of the difference in treatment of net taxes.
    • Statement 2 is incorrect: Both GDP as well as GVA do not include the value of intermediate goods. GDP is calculated by adding Net Taxes (taxes earned by the government – subsidies paid by the government) to the GVA. GDP fails to gauge the real economic scenario because a sharp increase in the output can be due to higher tax collections which could be on account of better compliance or coverage, rather than the real output situation. As a result, GVA is a more precise measure of the value added by each sector of the economy.
    Additional information:
    ● GDP is the sum of private consumption, gross investment in the economy, government investment, government spending and net foreign trade (the difference between exports and imports).
    ● GDP = private consumption + gross investment + government investment + government spending + (exports-imports)
    ● GDP measures the value of a country’s final goods and services (those purchased by the final user) generated in a specific time period (say, a quarter or a year). It includes all of the output produced within a country’s borders.
    GDP = Private consumption + gross investment + government investment + government spending + (exports – imports)
    ● Private Consumption Expenditure refers to the value of all goods and services purchased for consumption by households.
    ● Government Consumption Expenditure refers to the value of all goods and services purchased for consumption by the government.
    ● Gross Investment refers to the total value of all capital investments made in the economy.
    The GVA of a sector is defined as the value of output minus the value of its intermediary inputs. This “value added” is distributed among the primary production factors, labour and capital. By examining GVA growth, one may determine which sectors of the economy are doing well and which are struggling.
    Calculation of GVA
    ● GVA is the sum of a country’s GDP and net of subsidies and taxes in the economy at the macro level, according to national accounting.
    ● Gross Value Added = GDP + product subsidies – product taxes
    ● Previously, India measured GVA at ‘factor cost’ until a new methodology was implemented, in which GVA at ‘basic prices’ became the primary measure of economic output.
    ● GVA at basic prices will include production taxes and exclude production subsidies.
    ● GVA at factor cost included no taxes and excluded no subsidies

  5. Question 5 of 5
    5. Question

    5. Consider the following statements regarding Nominal Effective Exchange Rate (NEER) and Real Effective Exchange Rate (REER):
    1. A Nominal Effective Exchange Rate (NEER) adjusted for inflation equals its Real Effective Exchange Rate (REER).
    2. An increase in a nation’s REER is an indication that its exports are becoming more expensive and its imports are becoming cheaper.
    Which of the statements given above is/are correct?

    Correct

    Answer: C
    Explanation:
    Statement 1 is correct: A Nominal Effective Exchange Rate (NEER) adjusted for inflation equals its Real Effective Exchange Rate (REER). The important aspect of NEER is that it is not adjusted for inflation. Thus, it does not say anything about the real strength of a currency; it is just a relative value. REER is considered a more accurate measure to gauge a currency’s strength. The Real Effective Exchange Rate (REER) is the NEER adjusted by inflation differentials between the domestic currency and that of the trading partners. Thus, it says about the real strength of a domestic currency against a basket of currencies, unlike NEER.
    Statement 2 is correct: An increase in a nation’s REER is an indication that its exports are becoming more expensive and its imports are becoming cheaper.
    Additional information:
    ● The Real Effective Exchange Rate (REER) is the weighted average of a country’s currency in relation to an index or basket of other major currencies.
    ● The weights are determined by comparing the relative trade balance of a country’s currency against that of each country in the index.
    ● The Nominal Effective Exchange Rate (NEER) is an index of the weighted average of bilateral exchange rates of home currency with respect to a basket of currencies of trading partners. It is also known as the trade-weighted currency index.
    ● An increase in NEER indicates that the domestic currency has appreciated against the basket of currencies whereas a decrease indicates a relative depreciation.
    NEER and REER in the Indian context
    ● The Reserve Bank compiles and disseminates indices of NEER and REER of the Indian rupee.
    ● The latest series was constructed with the base year 2015-16. The new indices of NEER and REER replaced the old series from 2004-05 onwards.
    ● The coverage of the NEER and REER basket has been expanded from 36 to 40 currencies. This was in view of the growing importance of emerging markets and developing economies in India’s foreign trade and to better reflect shifts in external competitiveness.
    ● The selection of currencies for NEER and REER in the new series is based on two criteria:
    ● Trading partners with extremely high and volatile inflation are excluded. This is because their currencies tend to experience rapid nominal declines, undermining the stability of the NEER and REER indices. This obscures their usefulness in the assessment of external competitiveness.
    ● Data on inflation and exchange rates of trading partners should be available on a regular basis.

    Incorrect

    Answer: C
    Explanation:
    Statement 1 is correct: A Nominal Effective Exchange Rate (NEER) adjusted for inflation equals its Real Effective Exchange Rate (REER). The important aspect of NEER is that it is not adjusted for inflation. Thus, it does not say anything about the real strength of a currency; it is just a relative value. REER is considered a more accurate measure to gauge a currency’s strength. The Real Effective Exchange Rate (REER) is the NEER adjusted by inflation differentials between the domestic currency and that of the trading partners. Thus, it says about the real strength of a domestic currency against a basket of currencies, unlike NEER.
    Statement 2 is correct: An increase in a nation’s REER is an indication that its exports are becoming more expensive and its imports are becoming cheaper.
    Additional information:
    ● The Real Effective Exchange Rate (REER) is the weighted average of a country’s currency in relation to an index or basket of other major currencies.
    ● The weights are determined by comparing the relative trade balance of a country’s currency against that of each country in the index.
    ● The Nominal Effective Exchange Rate (NEER) is an index of the weighted average of bilateral exchange rates of home currency with respect to a basket of currencies of trading partners. It is also known as the trade-weighted currency index.
    ● An increase in NEER indicates that the domestic currency has appreciated against the basket of currencies whereas a decrease indicates a relative depreciation.
    NEER and REER in the Indian context
    ● The Reserve Bank compiles and disseminates indices of NEER and REER of the Indian rupee.
    ● The latest series was constructed with the base year 2015-16. The new indices of NEER and REER replaced the old series from 2004-05 onwards.
    ● The coverage of the NEER and REER basket has been expanded from 36 to 40 currencies. This was in view of the growing importance of emerging markets and developing economies in India’s foreign trade and to better reflect shifts in external competitiveness.
    ● The selection of currencies for NEER and REER in the new series is based on two criteria:
    ● Trading partners with extremely high and volatile inflation are excluded. This is because their currencies tend to experience rapid nominal declines, undermining the stability of the NEER and REER indices. This obscures their usefulness in the assessment of external competitiveness.
    ● Data on inflation and exchange rates of trading partners should be available on a regular basis.

window.wpAdvQuizInitList = window.wpAdvQuizInitList || []; window.wpAdvQuizInitList.push({ id: '#wpAdvQuiz_595', init: { quizId: 595, mode: 0, globalPoints: 10, timelimit: 0, resultsGrade: [0], bo: 0, qpp: 0, catPoints: [10], formPos: 0, lbn: "Finish quiz", json: {"2769":{"type":"single","id":2769,"catId":0,"points":2,"correct":[1,0,0,0]},"2770":{"type":"single","id":2770,"catId":0,"points":2,"correct":[0,1,0,0]},"2771":{"type":"single","id":2771,"catId":0,"points":2,"correct":[1,0,0,0]},"2772":{"type":"single","id":2772,"catId":0,"points":2,"correct":[1,0,0,0]},"2773":{"type":"single","id":2773,"catId":0,"points":2,"correct":[0,0,1,0]}} } });




Day-555 | Daily MCQs | UPSC Prelims | SCIENCE AND TECHNOLOGY

Day-555

Time limit: 0

Quiz-summary

0 of 5 questions completed

Questions:

  1. 1
  2. 2
  3. 3
  4. 4
  5. 5

Information

DAILY MCQ

You have already completed the quiz before. Hence you can not start it again.

Quiz is loading...

You must sign in or sign up to start the quiz.

You have to finish following quiz, to start this quiz:

Results

0 of 5 questions answered correctly

Your time:

Time has elapsed

You have reached 0 of 0 points, (0)

Categories

  1. Not categorized 0%
  1. 1
  2. 2
  3. 3
  4. 4
  5. 5
  1. Answered
  2. Review
  1. Question 1 of 5
    1. Question

    1. With reference to the birth control pills, consider the following mechanisms:
    1. Stopping or reducing ovulation
    2. Thinning of the cervical mucus
    3. Thickening of uterus lining
    Birth control pills can target how many of the above mechanisms to prevent pregnancy in females?

    Correct

    Answer: A
    Explanation
    The birth control pill is a type of oral contraception that uses hormones to prevent pregnancy. The hormones in birth control pills prevent pregnancy by blocking conception when sperm fertilizes an egg. They also cause changes in the uterus so that it can’t support a pregnancy. Oral contraceptive pills are either combined estrogen-progesterone (also called combined oral contraceptive pill- COC) or progesterone-only pill (POP).
    ● Mechanism 1 is correct: One of the mechanisms targeted by oral contraceptives is to stop ovulation. Ovulation is the release of eggs from the ovaries of females. Thus, there is no egg for fertilization which prevents pregnancy.
    ● Mechanism 2 is incorrect: Thickening of cervical mucus is another mechanism. This creates a barrier that prevents sperm from entering the uterus and eventually reaching the egg.
    ● Mechanism 3 is incorrect: Thinning of uterus lining is also achieved through oral contraceptive pills. This prevents fertilized eggs from getting attached and growing into a fetus.

    Incorrect

    Answer: A
    Explanation
    The birth control pill is a type of oral contraception that uses hormones to prevent pregnancy. The hormones in birth control pills prevent pregnancy by blocking conception when sperm fertilizes an egg. They also cause changes in the uterus so that it can’t support a pregnancy. Oral contraceptive pills are either combined estrogen-progesterone (also called combined oral contraceptive pill- COC) or progesterone-only pill (POP).
    ● Mechanism 1 is correct: One of the mechanisms targeted by oral contraceptives is to stop ovulation. Ovulation is the release of eggs from the ovaries of females. Thus, there is no egg for fertilization which prevents pregnancy.
    ● Mechanism 2 is incorrect: Thickening of cervical mucus is another mechanism. This creates a barrier that prevents sperm from entering the uterus and eventually reaching the egg.
    ● Mechanism 3 is incorrect: Thinning of uterus lining is also achieved through oral contraceptive pills. This prevents fertilized eggs from getting attached and growing into a fetus.

  2. Question 2 of 5
    2. Question

    2. With reference to the Worldcoin project, consider the following statements:
    1. It aims to create one of the world’s largest cryptocurrency networks for digital finance.
    2. Fingerprint scan will be used for authentication and signup purposes on the network.
    3. It will be open to everyone regardless of his/her country.
    How many of the above statements are correct?

    Correct

    Answer: B
    Explanation
    A project called Worldcoin has been launched by OpenAI, an Artificial Intelligence company. The project claims to be building the world’s largest identity and financial public network.
    ● Statement 1 is correct: The project aims to introduce Worldcoin token as a new cryptocurrency that offers a new and unique method for identification to reduce the risk of fraud. Worldcoin token (WLD) can be used as a medium of purchases and transfers globally using digital assets and traditional currencies.
    ● Statement 2 is incorrect: Iris scan (eye) is essential to sign up to the network. It will be done through a ball-like object called an ‘orb’. Once the orb’s iris scan verifies the person is a real human, it creates a World ID for them. The biometric data would help differentiate between humans and Artificial Intelligence systems and prevent duplication of IDs from the same person. It can then be used as an ID in a variety of everyday applications – such as a cryptocurrency wallet – without revealing the user’s identity.
    ● Statement 3 is correct: The Worldcoin protocol is intended to be the world’s largest identity and financial public network, open to everyone regardless of their country, background, or economic status.

    Incorrect

    Answer: B
    Explanation
    A project called Worldcoin has been launched by OpenAI, an Artificial Intelligence company. The project claims to be building the world’s largest identity and financial public network.
    ● Statement 1 is correct: The project aims to introduce Worldcoin token as a new cryptocurrency that offers a new and unique method for identification to reduce the risk of fraud. Worldcoin token (WLD) can be used as a medium of purchases and transfers globally using digital assets and traditional currencies.
    ● Statement 2 is incorrect: Iris scan (eye) is essential to sign up to the network. It will be done through a ball-like object called an ‘orb’. Once the orb’s iris scan verifies the person is a real human, it creates a World ID for them. The biometric data would help differentiate between humans and Artificial Intelligence systems and prevent duplication of IDs from the same person. It can then be used as an ID in a variety of everyday applications – such as a cryptocurrency wallet – without revealing the user’s identity.
    ● Statement 3 is correct: The Worldcoin protocol is intended to be the world’s largest identity and financial public network, open to everyone regardless of their country, background, or economic status.

  3. Question 3 of 5
    3. Question

    3. With reference to the ‘Formation water’, consider the following statements:
    1. It appears during the mining of coal from underground.
    2. It has a very high salt content.
    3. It cannot be treated by any means.
    How many of the above statements are correct?

    Correct

    Answer: A
    Explanation
    ● Statement 1 is incorrect: Formation water is water that appears during the drilling process for oil and gas extraction. It is very dense and viscous.
    ● Statement 2 is correct: The Formation water consists of oily components, brine solutions, and solvents that are used during various phases in the oil industry. It has a high salt content which includes sodium, potassium, calcium, and magnesium salts.
    ● Statement 3 is incorrect: There are ways and means to treat formation water. It depends on the usage of the treated product water.

    Treatment of the formation water
    The process can be summed up in four stages:
    1. Separation: removing crude, oil and other contaminants in the water that can clog filters downstream.
    2. Suspended Solid Removal: eliminating small size particles remaining.
    3. Dissolved solid removal: using low- and high-pressure membranes.
    4. Conditioning: depending on the quality of the water it may be a post treatment station for disinfection or activated carbon filtration.
    More about formation water
    ● If formation water is released without treatment, it can lead to the degradation of water quality, posing a significant threat to aquatic life.
    ● The toxins in formation water can also bio-magnify into the human food chain, if fish and other aquatic products, harvested from the affected area, are consumed.

    Incorrect

    Answer: A
    Explanation
    ● Statement 1 is incorrect: Formation water is water that appears during the drilling process for oil and gas extraction. It is very dense and viscous.
    ● Statement 2 is correct: The Formation water consists of oily components, brine solutions, and solvents that are used during various phases in the oil industry. It has a high salt content which includes sodium, potassium, calcium, and magnesium salts.
    ● Statement 3 is incorrect: There are ways and means to treat formation water. It depends on the usage of the treated product water.

    Treatment of the formation water
    The process can be summed up in four stages:
    1. Separation: removing crude, oil and other contaminants in the water that can clog filters downstream.
    2. Suspended Solid Removal: eliminating small size particles remaining.
    3. Dissolved solid removal: using low- and high-pressure membranes.
    4. Conditioning: depending on the quality of the water it may be a post treatment station for disinfection or activated carbon filtration.
    More about formation water
    ● If formation water is released without treatment, it can lead to the degradation of water quality, posing a significant threat to aquatic life.
    ● The toxins in formation water can also bio-magnify into the human food chain, if fish and other aquatic products, harvested from the affected area, are consumed.

  4. Question 4 of 5
    4. Question

    4. The ‘Turing Test’ is related to which of the following scientific developments?

    Correct

    Answer: B
    Explanation
    ● The Turing Test is used in artificial intelligence (AI) for determining whether a computer is capable of thinking like a human being. The test is named after Alan Turing, the founder of the Turing Test and an English computer scientist, cryptanalyst, mathematician, and theoretical biologist.
    ● The original Turing Test requires three terminals, each of which is physically separated from the other two. One terminal is operated by a computer, while the other two are operated by humans.
    ● During the test, one of the human being functions as the questioner, while the second human and the computer function as respondents. The questioner interrogates the respondents within a specific subject area, using a specified format and context. After a preset length of time or number of questions, the questioner is then asked to decide which respondent was human and which was a computer.

    Incorrect

    Answer: B
    Explanation
    ● The Turing Test is used in artificial intelligence (AI) for determining whether a computer is capable of thinking like a human being. The test is named after Alan Turing, the founder of the Turing Test and an English computer scientist, cryptanalyst, mathematician, and theoretical biologist.
    ● The original Turing Test requires three terminals, each of which is physically separated from the other two. One terminal is operated by a computer, while the other two are operated by humans.
    ● During the test, one of the human being functions as the questioner, while the second human and the computer function as respondents. The questioner interrogates the respondents within a specific subject area, using a specified format and context. After a preset length of time or number of questions, the questioner is then asked to decide which respondent was human and which was a computer.

  5. Question 5 of 5
    5. Question

    5. Meissner Effect is related to which of the following technologies?

    Correct

    Answer: B
    Explanation
    ● Meissner effect is the expulsion of a magnetic field from the interior of a material that is in the process of becoming a superconductor.

    ● A super conductor loses its resistance to the flow of electrical currents when cooled below a certain temperature called the critical temperature. It is close to absolute zero or zero kelvin.
    ● The Meissner Effect is used in magnetic levitation, which means a body is suspended with no support except a magnetic field. Modern high-speed bullet trains use the phenomenon of magnetic levitation.

    Incorrect

    Answer: B
    Explanation
    ● Meissner effect is the expulsion of a magnetic field from the interior of a material that is in the process of becoming a superconductor.

    ● A super conductor loses its resistance to the flow of electrical currents when cooled below a certain temperature called the critical temperature. It is close to absolute zero or zero kelvin.
    ● The Meissner Effect is used in magnetic levitation, which means a body is suspended with no support except a magnetic field. Modern high-speed bullet trains use the phenomenon of magnetic levitation.

window.wpAdvQuizInitList = window.wpAdvQuizInitList || []; window.wpAdvQuizInitList.push({ id: '#wpAdvQuiz_594', init: { quizId: 594, mode: 0, globalPoints: 10, timelimit: 0, resultsGrade: [0], bo: 0, qpp: 0, catPoints: [10], formPos: 0, lbn: "Finish quiz", json: {"2764":{"type":"single","id":2764,"catId":0,"points":2,"correct":[1,0,0,0]},"2765":{"type":"single","id":2765,"catId":0,"points":2,"correct":[0,1,0,0]},"2766":{"type":"single","id":2766,"catId":0,"points":2,"correct":[1,0,0,0]},"2767":{"type":"single","id":2767,"catId":0,"points":2,"correct":[0,1,0,0]},"2768":{"type":"single","id":2768,"catId":0,"points":2,"correct":[0,1,0,0]}} } });




Day-554 | Daily MCQs | UPSC Prelims | POLITY

Day-554

Time limit: 0

Quiz-summary

0 of 5 questions completed

Questions:

  1. 1
  2. 2
  3. 3
  4. 4
  5. 5

Information

DAILY MCQ

You have already completed the quiz before. Hence you can not start it again.

Quiz is loading...

You must sign in or sign up to start the quiz.

You have to finish following quiz, to start this quiz:

Results

0 of 5 questions answered correctly

Your time:

Time has elapsed

You have reached 0 of 0 points, (0)

Categories

  1. Not categorized 0%
  1. 1
  2. 2
  3. 3
  4. 4
  5. 5
  1. Answered
  2. Review
  1. Question 1 of 5
    1. Question

    1. Consider the following:
    1. Consolidated Fund of India
    2. Contingency Fund of India
    3. Public Account of India
    The Union Government Finance Accounts depicts the receipts and payments from how many of the funds mentioned above?

    Correct

    Answer: C
    Explanation:
    Statement 1 is correct: The Annual Accounts of the Union Government provide a comprehensive overview of the government’s financial transactions for a given fiscal year. These audit of these accounts are presented to the Parliament by the President of India while the report is prepared by the Comptroller and Auditor General (CAG) of India, typically within six months of the conclusion of the fiscal year. The Annual Accounts serve as a crucial tool for ensuring transparency and accountability in the government’s financial management.
    Components of the Annual Accounts: It consists of two main components:
    1. Finance Accounts: The Finance Accounts provide a detailed record of all receipts and payments made by the government during the fiscal year. These accounts include information on revenue collection, expenditure incurred; from the Consolidated Fund of India, Contingency Fund and Public Account; and the overall financial position of the government.(Hence Option C is correct)
    2. Appropriation Accounts: The Appropriation Accounts compare the actual expenditure incurred by the government with the amounts authorized by the Parliament through the annual budget. These accounts highlight any variances between the budgeted and actual expenditure and provide explanations for these variations.

    Incorrect

    Answer: C
    Explanation:
    Statement 1 is correct: The Annual Accounts of the Union Government provide a comprehensive overview of the government’s financial transactions for a given fiscal year. These audit of these accounts are presented to the Parliament by the President of India while the report is prepared by the Comptroller and Auditor General (CAG) of India, typically within six months of the conclusion of the fiscal year. The Annual Accounts serve as a crucial tool for ensuring transparency and accountability in the government’s financial management.
    Components of the Annual Accounts: It consists of two main components:
    1. Finance Accounts: The Finance Accounts provide a detailed record of all receipts and payments made by the government during the fiscal year. These accounts include information on revenue collection, expenditure incurred; from the Consolidated Fund of India, Contingency Fund and Public Account; and the overall financial position of the government.(Hence Option C is correct)
    2. Appropriation Accounts: The Appropriation Accounts compare the actual expenditure incurred by the government with the amounts authorized by the Parliament through the annual budget. These accounts highlight any variances between the budgeted and actual expenditure and provide explanations for these variations.

  2. Question 2 of 5
    2. Question

    2. Consider the following:
    1. Equality before the law
    2. Transparency and predictability
    3. Civic duty
    4. Independence of the judiciary
    How many of the given above are considered as the important features of “Rule of Law” in the Indian context?

    Correct

    Answer: C
    Explanation:
    Option 1 is correct: The rule of law implies that everyone, regardless of their status or position, is subject to the law and must abide by it. This means that no one is above the law and that everyone is treated equally under the law.The principle of equality before the law is essential for ensuring fairness and justice in society. It prevents discrimination and ensures that everyone has equal access to the legal system. When the law is applied equally, it creates a sense of trust and legitimacy among the people, fostering a more stable and harmonious society.
    Option 2 is correct: The rule of law requires that the law be clear, transparent, and predictable. This means that the law should be written in a way that is easy to understand and that it should be applied consistently and impartially.When the law is clear and predictable, it reduces uncertainty and promotes compliance. It also allows individuals to hold the government accountable for its actions and to challenge decisions that they believe violate their rights.
    Option 3 is incorrect: Civic duties are the responsibilities that citizens have to their communities and their country. These duties are often enshrined in law, but they can also be based on tradition, custom, or moral values. In India, there are a number of civic duties that are considered to be important for all citizens. These duties include:
    ● Obeying the law: This is the most basic civic duty, and it is essential for maintaining order and stability in society. Citizens should obey all laws, even those they disagree with, and they should report any violations to the authorities.
    ● Voting: Voting is another fundamental civic duty. It allows citizens to participate in the democratic process and to choose their representatives. Citizens should register to vote and participate in all elections.
    ● Paying taxes: Taxes are an important source of revenue for the government, and they are used to fund a variety of public services, such as education, healthcare, and infrastructure. Citizens should pay their taxes on time and in full.
    ● Protecting the environment: The environment is a vital resource that we must protect for future generations. Citizens should take steps to conserve resources, reduce pollution, and recycle.
    In addition to these general civic duties, there are also a number of specific civic duties that are enshrined in the Indian Constitution. These duties include:
    ● Abide by the Constitution and respect its ideals and institutions, the National Flag and the National Anthem: This duty requires citizens to uphold the values and principles of the Constitution and to respect the symbols of national unity.
    ● Cherish and follow the noble ideals which inspired our national struggle for freedom: This duty requires citizens to appreciate the sacrifices made by those who fought for India’s independence and to strive to live up to their ideals.
    ● Uphold and protect the sovereignty, unity, and integrity of India: This duty requires citizens to defend India’s borders, to promote unity among its people, and to protect its national interests.
    Option 4 is correct: The rule of law requires that the judiciary be independent of the executive and legislative branches of government. This means that the judiciary should be able to make decisions without fear of political interference or intimidation.An independent judiciary is essential for upholding the rule of law and protecting individual rights. When the judiciary is free from political influence, it can ensure that the law is applied fairly and impartially. It can also protect individuals from arbitrary or unjust actions by the government.
    Additional Information:
    Rule of Law vs. Due Process of Law
    While the rule of law and due process of law are related concepts, they are not the same. The rule of law is a broader principle that encompasses the entire legal system, including the laws themselves, the institutions that enforce them, and the values that underpin them.
    Due process of law, on the other hand, is a specific legal concept that refers to the procedures that must be followed when the government takes action against an individual.
    In essence, the rule of law is the foundation upon which due process of law is built. The rule of law ensures that the legal system is fair, just, and impartial, while due process of law ensures that individuals are treated fairly and have their rights protected when the government takes action against them.

    Incorrect

    Answer: C
    Explanation:
    Option 1 is correct: The rule of law implies that everyone, regardless of their status or position, is subject to the law and must abide by it. This means that no one is above the law and that everyone is treated equally under the law.The principle of equality before the law is essential for ensuring fairness and justice in society. It prevents discrimination and ensures that everyone has equal access to the legal system. When the law is applied equally, it creates a sense of trust and legitimacy among the people, fostering a more stable and harmonious society.
    Option 2 is correct: The rule of law requires that the law be clear, transparent, and predictable. This means that the law should be written in a way that is easy to understand and that it should be applied consistently and impartially.When the law is clear and predictable, it reduces uncertainty and promotes compliance. It also allows individuals to hold the government accountable for its actions and to challenge decisions that they believe violate their rights.
    Option 3 is incorrect: Civic duties are the responsibilities that citizens have to their communities and their country. These duties are often enshrined in law, but they can also be based on tradition, custom, or moral values. In India, there are a number of civic duties that are considered to be important for all citizens. These duties include:
    ● Obeying the law: This is the most basic civic duty, and it is essential for maintaining order and stability in society. Citizens should obey all laws, even those they disagree with, and they should report any violations to the authorities.
    ● Voting: Voting is another fundamental civic duty. It allows citizens to participate in the democratic process and to choose their representatives. Citizens should register to vote and participate in all elections.
    ● Paying taxes: Taxes are an important source of revenue for the government, and they are used to fund a variety of public services, such as education, healthcare, and infrastructure. Citizens should pay their taxes on time and in full.
    ● Protecting the environment: The environment is a vital resource that we must protect for future generations. Citizens should take steps to conserve resources, reduce pollution, and recycle.
    In addition to these general civic duties, there are also a number of specific civic duties that are enshrined in the Indian Constitution. These duties include:
    ● Abide by the Constitution and respect its ideals and institutions, the National Flag and the National Anthem: This duty requires citizens to uphold the values and principles of the Constitution and to respect the symbols of national unity.
    ● Cherish and follow the noble ideals which inspired our national struggle for freedom: This duty requires citizens to appreciate the sacrifices made by those who fought for India’s independence and to strive to live up to their ideals.
    ● Uphold and protect the sovereignty, unity, and integrity of India: This duty requires citizens to defend India’s borders, to promote unity among its people, and to protect its national interests.
    Option 4 is correct: The rule of law requires that the judiciary be independent of the executive and legislative branches of government. This means that the judiciary should be able to make decisions without fear of political interference or intimidation.An independent judiciary is essential for upholding the rule of law and protecting individual rights. When the judiciary is free from political influence, it can ensure that the law is applied fairly and impartially. It can also protect individuals from arbitrary or unjust actions by the government.
    Additional Information:
    Rule of Law vs. Due Process of Law
    While the rule of law and due process of law are related concepts, they are not the same. The rule of law is a broader principle that encompasses the entire legal system, including the laws themselves, the institutions that enforce them, and the values that underpin them.
    Due process of law, on the other hand, is a specific legal concept that refers to the procedures that must be followed when the government takes action against an individual.
    In essence, the rule of law is the foundation upon which due process of law is built. The rule of law ensures that the legal system is fair, just, and impartial, while due process of law ensures that individuals are treated fairly and have their rights protected when the government takes action against them.

  3. Question 3 of 5
    3. Question

    3. Consider the following statements about the Mayoral System in India:
    1. Mayoral system in India is not explicitly mentioned in the Constitution of India.
    2. The Mayor is chosen through indirect election by the councilors from among themselves in all the states.
    3. It aims to decentralize the power from the state government to the municipal level
    How many of the above given statements are correct?

    Correct

    Answer: B
    Explanation:
    Statement 1 is correct:The Constitution of India does not explicitly mandate the mayoral system, but it allows states to adopt it through their own legislation. Several Indian states have implemented the mayoral system, including Andhra Pradesh, Chhattisgarh, Jharkhand, Madhya Pradesh, Odisha, Tamil Nadu, and Telangana.The mayor is merely a ceremonial authority, and executive decisions are carried out by the municipal commissioner appointed by the state government.
    Statement 2 is incorrect:The Mayor in the Municipal Corporation is usually chosen through indirect election by the councilors from among themselves. However, few states including Uttarakhand, Chhattisgarh, Jharkhand, Madhya Pradesh, Uttar Pradesh, and Tamil Nadu – have mayors who are elected directly. This inconsistency is because of the lack of clarity about the Mayoral system in the Constitution of India. Also the 74th amendment did not prescribe the manner of election, tenure, or powers of the Mayors/Chairpersons of Urban Local Bodies.
    Statement 3 is correct: The mayoral system aims to decentralize power from the state government to the municipal level, empowering local bodies to make decisions that directly impact their citizens.The mayoral system seeks to bring governance closer to the people, giving them a direct say in matters that affect their daily lives. This decentralization of power enhances local accountability and encourages greater citizen participation in the decision-making process.

    Roles, Functions, and Powers of the Mayor
    The specific roles, functions, and powers of the mayor vary from state to state. However, in general, the mayor is responsible for:
    ● Providing leadership and direction to the municipal administration
    ● Presiding over the meetings of the municipal corporation
    ● Overseeing the implementation of municipal policies and programs
    ● Representing the municipality in official engagements
    ● Ensuring the financial well-being of the municipality
    ● Promoting the interests of the city and its citizens
    The exact extent of the mayor’s powers depends on the specific provisions of the state legislation governing the mayoral system.

    Incorrect

    Answer: B
    Explanation:
    Statement 1 is correct:The Constitution of India does not explicitly mandate the mayoral system, but it allows states to adopt it through their own legislation. Several Indian states have implemented the mayoral system, including Andhra Pradesh, Chhattisgarh, Jharkhand, Madhya Pradesh, Odisha, Tamil Nadu, and Telangana.The mayor is merely a ceremonial authority, and executive decisions are carried out by the municipal commissioner appointed by the state government.
    Statement 2 is incorrect:The Mayor in the Municipal Corporation is usually chosen through indirect election by the councilors from among themselves. However, few states including Uttarakhand, Chhattisgarh, Jharkhand, Madhya Pradesh, Uttar Pradesh, and Tamil Nadu – have mayors who are elected directly. This inconsistency is because of the lack of clarity about the Mayoral system in the Constitution of India. Also the 74th amendment did not prescribe the manner of election, tenure, or powers of the Mayors/Chairpersons of Urban Local Bodies.
    Statement 3 is correct: The mayoral system aims to decentralize power from the state government to the municipal level, empowering local bodies to make decisions that directly impact their citizens.The mayoral system seeks to bring governance closer to the people, giving them a direct say in matters that affect their daily lives. This decentralization of power enhances local accountability and encourages greater citizen participation in the decision-making process.

    Roles, Functions, and Powers of the Mayor
    The specific roles, functions, and powers of the mayor vary from state to state. However, in general, the mayor is responsible for:
    ● Providing leadership and direction to the municipal administration
    ● Presiding over the meetings of the municipal corporation
    ● Overseeing the implementation of municipal policies and programs
    ● Representing the municipality in official engagements
    ● Ensuring the financial well-being of the municipality
    ● Promoting the interests of the city and its citizens
    The exact extent of the mayor’s powers depends on the specific provisions of the state legislation governing the mayoral system.

  4. Question 4 of 5
    4. Question

    4. “Appointment of Governors shall be made on the recommendation of a committee comprising the Prime Minister, the Home Minister, the Lok Sabha Speaker and the concerned Chief Minister of the State”.
    Which of the following commissions made the recommendation given above?

    Correct

    Answer: C
    Explanation: The correct answer is MM Punchhi Commission. Among other recommendations, the recommendations with respect to the appointment and removal of the Governor of the State include:
    ● There should be a say of the state’s Chief Minister while making the Governor’s appointment.
    ● A committee should be formed that is entrusted with the task of appointment of governors. This committee may comprise the Prime Minister, the Home Minister, the Lok Sabha’s speaker and the concerned Chief Minister of the State.
    ● The term of appointment should be five years.
    ● Governor could only be removed via a resolution by the State Legislature.
    Rajamannar Committee (1971) stressed that the Governor of the state should not consider himself as an agent of the centre but play his role as the constitutional head of the State.
    Sarkaria Commission:
    ● The Constitution itself should lay down the process for consulting the chief minister when appointing a state governor.
    ● As long as the council of ministers has a majority in the assembly, the governor cannot dismiss it.
    ● Unless there are some exceptionally compelling circumstances, a governor’s five-year governorship should not be disrupted.

    Incorrect

    Answer: C
    Explanation: The correct answer is MM Punchhi Commission. Among other recommendations, the recommendations with respect to the appointment and removal of the Governor of the State include:
    ● There should be a say of the state’s Chief Minister while making the Governor’s appointment.
    ● A committee should be formed that is entrusted with the task of appointment of governors. This committee may comprise the Prime Minister, the Home Minister, the Lok Sabha’s speaker and the concerned Chief Minister of the State.
    ● The term of appointment should be five years.
    ● Governor could only be removed via a resolution by the State Legislature.
    Rajamannar Committee (1971) stressed that the Governor of the state should not consider himself as an agent of the centre but play his role as the constitutional head of the State.
    Sarkaria Commission:
    ● The Constitution itself should lay down the process for consulting the chief minister when appointing a state governor.
    ● As long as the council of ministers has a majority in the assembly, the governor cannot dismiss it.
    ● Unless there are some exceptionally compelling circumstances, a governor’s five-year governorship should not be disrupted.

  5. Question 5 of 5
    5. Question

    5. The Simon Commission was appointed by the British government to report on the working of which of the following?

    Correct

    Answer: D
    Explanation:
    Government of India Act, 1919 provided for the appointment of the statutory commission to inquire into and report on its working after ten years of coming into force.
    The Government of India Act 1919 was the codified version of the Montague-Chelmsford reforms – named after Edwin Charles Montague and Lord Chelmsford, who held positions of the Secretary of State and Viceroy of British India respectively. The Act was sold to the Indians as ‘a step in the progressive realisation of responsible government in India as an integral part of the empire’.
    The persistent demand for further reforms led the British government to appoint Simon Commission in 1927 and report on the working of the Indian Constitution as established under Government of India Act, 1919.

    Incorrect

    Answer: D
    Explanation:
    Government of India Act, 1919 provided for the appointment of the statutory commission to inquire into and report on its working after ten years of coming into force.
    The Government of India Act 1919 was the codified version of the Montague-Chelmsford reforms – named after Edwin Charles Montague and Lord Chelmsford, who held positions of the Secretary of State and Viceroy of British India respectively. The Act was sold to the Indians as ‘a step in the progressive realisation of responsible government in India as an integral part of the empire’.
    The persistent demand for further reforms led the British government to appoint Simon Commission in 1927 and report on the working of the Indian Constitution as established under Government of India Act, 1919.

window.wpAdvQuizInitList = window.wpAdvQuizInitList || []; window.wpAdvQuizInitList.push({ id: '#wpAdvQuiz_593', init: { quizId: 593, mode: 0, globalPoints: 10, timelimit: 0, resultsGrade: [0], bo: 0, qpp: 0, catPoints: [10], formPos: 0, lbn: "Finish quiz", json: {"2759":{"type":"single","id":2759,"catId":0,"points":2,"correct":[0,0,1,0]},"2760":{"type":"single","id":2760,"catId":0,"points":2,"correct":[0,0,1,0]},"2761":{"type":"single","id":2761,"catId":0,"points":2,"correct":[0,1,0,0]},"2762":{"type":"single","id":2762,"catId":0,"points":2,"correct":[0,0,1,0]},"2763":{"type":"single","id":2763,"catId":0,"points":2,"correct":[0,0,0,1]}} } });




Day-553 | Daily MCQs | UPSC Prelims | GEOGRAPHY

Day-553

Time limit: 0

Quiz-summary

0 of 5 questions completed

Questions:

  1. 1
  2. 2
  3. 3
  4. 4
  5. 5

Information

DAILY MCQ

You have already completed the quiz before. Hence you can not start it again.

Quiz is loading...

You must sign in or sign up to start the quiz.

You have to finish following quiz, to start this quiz:

Results

0 of 5 questions answered correctly

Your time:

Time has elapsed

You have reached 0 of 0 points, (0)

Categories

  1. Not categorized 0%
  1. 1
  2. 2
  3. 3
  4. 4
  5. 5
  1. Answered
  2. Review
  1. Question 1 of 5
    1. Question

    1. With reference to temperature inversion, consider the following statements:
    1. In temperature inversion, temperature decreases with increase in elevation.
    2. Temperature inversion at surface level promotes instability in lower layers of atmosphere.
    Which of the statements given above is/are correct?

    Correct

    Answer: D
    Explanation:
    Statement 1 is incorrect:
    ● Normal lapse rate (NLR): When temperature decreases with increase in elevation, it is called NLR. But sometimes, the situation is reversed and the normal lapse rate is inverted. It is called Inversion of temperature i.e. increase in temperature with increase in elevation.
    ● Inversion is usually of short duration but quite common nonetheless.
    Normal conditions required:
    ● A long winter night with clear skies and still air is ideal situation for inversion.
    ● The heat of the day is radiated off during the night, and by early morning hours, the earth is cooler than the air above.
    ● Example – over polar areas, temperature inversion is normal throughout the year.
    Statement 2 is incorrect:
    ● Surface inversion promotes stability in the lower layers of the atmosphere.
    ● Smoke and dust particles get collected beneath the inversion layer and spread horizontally to fill the lower strata of the atmosphere.
    ● Dense fogs in mornings are common occurrences especially during winter season.
    ● This inversion commonly lasts for few hours until the sun comes up and beings to warm the earth.
    Additional Information
    Temperature inversion on hilly regions:
    ● The inversion takes place in hills and mountains due to air drainage.
    ● Cold air at the hills and mountains, produced during night, flows under the influence of gravity.
    ● Being heavy and dense, the cold air acts almost like water and moves down the slope to pile up deeply in pockets and valley bottoms with warm air above. This is called air drainage.
    ● It protects plants from frost damages.

    Incorrect

    Answer: D
    Explanation:
    Statement 1 is incorrect:
    ● Normal lapse rate (NLR): When temperature decreases with increase in elevation, it is called NLR. But sometimes, the situation is reversed and the normal lapse rate is inverted. It is called Inversion of temperature i.e. increase in temperature with increase in elevation.
    ● Inversion is usually of short duration but quite common nonetheless.
    Normal conditions required:
    ● A long winter night with clear skies and still air is ideal situation for inversion.
    ● The heat of the day is radiated off during the night, and by early morning hours, the earth is cooler than the air above.
    ● Example – over polar areas, temperature inversion is normal throughout the year.
    Statement 2 is incorrect:
    ● Surface inversion promotes stability in the lower layers of the atmosphere.
    ● Smoke and dust particles get collected beneath the inversion layer and spread horizontally to fill the lower strata of the atmosphere.
    ● Dense fogs in mornings are common occurrences especially during winter season.
    ● This inversion commonly lasts for few hours until the sun comes up and beings to warm the earth.
    Additional Information
    Temperature inversion on hilly regions:
    ● The inversion takes place in hills and mountains due to air drainage.
    ● Cold air at the hills and mountains, produced during night, flows under the influence of gravity.
    ● Being heavy and dense, the cold air acts almost like water and moves down the slope to pile up deeply in pockets and valley bottoms with warm air above. This is called air drainage.
    ● It protects plants from frost damages.

  2. Question 2 of 5
    2. Question

    2. Consider the following statements:
    Statement I: The North Sea records higher salinity despite being located at higher latitude.
    Statement II: Ocean currents are major determinants of salinity of oceans/seas.
    Which one of the following is correct in respect of the above statements?

    Correct

    Answer: A
    Explanation
    Both the Statements are correct and Statement II is correct explanation of Statement I:
    ● The North Sea, in spite of its location in higher latitudes, records higher salinity due to more saline water brought by the North Atlantic Drift. While, Baltic Sea records low salinity due to influx of river waters in large quantity.
    Horizontal Distribution of Salinity:
    • The horizontal distribution of salinity in oceans and seas is influence by several factors.
    These include:
    ● The proximity to major river mouths, where freshwater input can lower salinity levels. Example – Salinity is very low in Black Sea due to enormous fresh water influx by rivers.
    ● Evaporation rates, which can increase salinity. Example – The Mediterranean Sea records higher salinity due to high evaporation.
    ● Ocean currents also play a role in distributing salinity horizontally, as they transport water masses with different salinity levels across different regions.
    Other factors may be:
    ● Precipitation
    ● Influx of river water
    ● Atmospheric pressure and wind direction
    ● Global warming

    Incorrect

    Answer: A
    Explanation
    Both the Statements are correct and Statement II is correct explanation of Statement I:
    ● The North Sea, in spite of its location in higher latitudes, records higher salinity due to more saline water brought by the North Atlantic Drift. While, Baltic Sea records low salinity due to influx of river waters in large quantity.
    Horizontal Distribution of Salinity:
    • The horizontal distribution of salinity in oceans and seas is influence by several factors.
    These include:
    ● The proximity to major river mouths, where freshwater input can lower salinity levels. Example – Salinity is very low in Black Sea due to enormous fresh water influx by rivers.
    ● Evaporation rates, which can increase salinity. Example – The Mediterranean Sea records higher salinity due to high evaporation.
    ● Ocean currents also play a role in distributing salinity horizontally, as they transport water masses with different salinity levels across different regions.
    Other factors may be:
    ● Precipitation
    ● Influx of river water
    ● Atmospheric pressure and wind direction
    ● Global warming

  3. Question 3 of 5
    3. Question

    3. Consider the following pairs:
    Cropping Seasons – Crops
    1. Kharif Season – Gram
    2. Rabi Season – Cotton
    3. Zaid Season – Cucumber
    How many of the pairs given above are correctly matched?

    Correct

    Answer: A
    Explanation:
    Pair 1 is not correctly matched:
    ● There are three distinct crop seasons in the northern and interior parts of the country, namely Kharif, Rabi and Zaid.
    ● The Kharif season largely coincides with Southwest Monsoon under which the cultivation of tropical crops, such as rice, cotton, jute, jowar, bajra, and tur is possible.
    Pair 2 is not correctly matched:
    ● The rabi season begins with the onset of winter in October-November and ends in March-April.
    ● The low-temperature conditions during this season facilitate the cultivation of temperate and subtropical crops such as wheat, gram, and mustard.
    Pair 3 is correctly matched:
    ● Zaid is a short duration summer cropping season beginning after harvesting of rabi crops.
    ● The cultivation of watermelons, cucumbers, vegetables and fodder crops during this season is done on irrigated lands.
    However, this type of distinction in the cropping season does not exist in the southern parts of the country. Here, the temperature is high enough to grow tropical crops during any period in the year provided the soil moisture is available. Therefore, in this region same crops can be grown thrice in an agricultural year provided there is sufficient soil moisture.

    Incorrect

    Answer: A
    Explanation:
    Pair 1 is not correctly matched:
    ● There are three distinct crop seasons in the northern and interior parts of the country, namely Kharif, Rabi and Zaid.
    ● The Kharif season largely coincides with Southwest Monsoon under which the cultivation of tropical crops, such as rice, cotton, jute, jowar, bajra, and tur is possible.
    Pair 2 is not correctly matched:
    ● The rabi season begins with the onset of winter in October-November and ends in March-April.
    ● The low-temperature conditions during this season facilitate the cultivation of temperate and subtropical crops such as wheat, gram, and mustard.
    Pair 3 is correctly matched:
    ● Zaid is a short duration summer cropping season beginning after harvesting of rabi crops.
    ● The cultivation of watermelons, cucumbers, vegetables and fodder crops during this season is done on irrigated lands.
    However, this type of distinction in the cropping season does not exist in the southern parts of the country. Here, the temperature is high enough to grow tropical crops during any period in the year provided the soil moisture is available. Therefore, in this region same crops can be grown thrice in an agricultural year provided there is sufficient soil moisture.

  4. Question 4 of 5
    4. Question

    4. According to the Koeppen’s scheme of classification of Indian climatic regions, ‘As’ type of climatic region refers to:

    Correct

    Answer: D
    Explanation:
    ● Koeppen’s scheme of classification of Indian climatic regions divided India into 8 major climatic regions.
    ● Coromandel coast of Tamil Nadu has been categorised as a ‘Monsoon with dry summer’ climatic region i.e. ‘As’ type of climate.
    ● Major climatic region according to Koeppen’s climatic classification in India are as given in the table below:

    Additional Information
    ● A climatic region has a homogeneous climatic condition which is the result of a combination of factors. Temperature and rainfall are two important elements which are considered to be decisive in all the schemes of climatic classification. The classification of climate, however, is a complex exercise. There are different schemes of classification of climate.
    ● Major climatic types of India are based on Koeppen’s scheme.
    Koeppen’s scheme
    ● Koeppen based his scheme of Climatic classification on monthly values of temperature and precipitation.
    ● He identified five major climatic types, namely:
    I. Tropical climates, where mean monthly temperature throughout the year is over 18°C.
    II. Dry climates, where precipitation is very low in comparison to temperature, and hence, dry. If dryness is less, it is semi-arid (S); if it is more, the climate is arid (W).
    III. Warm temperate climates, where mean temperature of the coldest month is between 18°C and minus 3°C.
    IV. Cool temperate climates, where mean temperature of the warmest month is over 10°C, and mean temperature of the coldest month is under minus 3°C.
    V. Ice climates, where mean temperature of the warmest month is under 10°C.
    ● Koeppen used letter symbols to denote climatic types as given above.
    ● Each type is further sub-divided into sub-types on the basis of seasonal variations in the distributional pattern of rainfall and temperature.
    ● He used S for semi-arid and W for arid and the following small letters to define sub-types:
    ⮚ f (sufficient precipitation)
    ⮚ m (rainforest despite a dry monsoon season)
    ⮚ w (dryseason in winter)
    ⮚ h (dry and hot)
    ⮚ c (less than four months with mean temperature over 10°C)
    ⮚ g (Gangetic plain).
    ● Accordingly, India can be divided into nine climatic regions (including steppe climate – Bshw).
    ● India’s climatic regions are shown in the following map.

    Incorrect

    Answer: D
    Explanation:
    ● Koeppen’s scheme of classification of Indian climatic regions divided India into 8 major climatic regions.
    ● Coromandel coast of Tamil Nadu has been categorised as a ‘Monsoon with dry summer’ climatic region i.e. ‘As’ type of climate.
    ● Major climatic region according to Koeppen’s climatic classification in India are as given in the table below:

    Additional Information
    ● A climatic region has a homogeneous climatic condition which is the result of a combination of factors. Temperature and rainfall are two important elements which are considered to be decisive in all the schemes of climatic classification. The classification of climate, however, is a complex exercise. There are different schemes of classification of climate.
    ● Major climatic types of India are based on Koeppen’s scheme.
    Koeppen’s scheme
    ● Koeppen based his scheme of Climatic classification on monthly values of temperature and precipitation.
    ● He identified five major climatic types, namely:
    I. Tropical climates, where mean monthly temperature throughout the year is over 18°C.
    II. Dry climates, where precipitation is very low in comparison to temperature, and hence, dry. If dryness is less, it is semi-arid (S); if it is more, the climate is arid (W).
    III. Warm temperate climates, where mean temperature of the coldest month is between 18°C and minus 3°C.
    IV. Cool temperate climates, where mean temperature of the warmest month is over 10°C, and mean temperature of the coldest month is under minus 3°C.
    V. Ice climates, where mean temperature of the warmest month is under 10°C.
    ● Koeppen used letter symbols to denote climatic types as given above.
    ● Each type is further sub-divided into sub-types on the basis of seasonal variations in the distributional pattern of rainfall and temperature.
    ● He used S for semi-arid and W for arid and the following small letters to define sub-types:
    ⮚ f (sufficient precipitation)
    ⮚ m (rainforest despite a dry monsoon season)
    ⮚ w (dryseason in winter)
    ⮚ h (dry and hot)
    ⮚ c (less than four months with mean temperature over 10°C)
    ⮚ g (Gangetic plain).
    ● Accordingly, India can be divided into nine climatic regions (including steppe climate – Bshw).
    ● India’s climatic regions are shown in the following map.

  5. Question 5 of 5
    5. Question

    5. With reference to the various types of droughts, consider the following statements:
    1. Meteorological drought is a situation when the productivity of a natural ecosystem fails due to shortage of water and as a consequence of ecological distress; damages are induced in the ecosystem.
    2. Ecological drought is a situation when there is a prolonged period of inadequate rainfall marked with mal-distribution of the same over time and space.
    3. Agricultural drought, also known as soil moisture drought, is characterised by low soil moisture that is necessary to support the crops, thereby resulting in crop failures.
    4. Hydrological drought is a situation when the availability of water in different storages and reservoirs like aquifers, lakes, reservoirs, etc. falls below what the precipitation can replenish.
    How many of the statements given above are correct?

    Correct

    Answer: B
    Explanation:
    Statement 1 is incorrect:
    ● Meteorological drought: It is a situation when there is a prolonged period of inadequate rainfall marked with mal-distribution of the same over time and space.
    Statement 2 is incorrect:
    ● Ecological drought: When the productivity of a natural ecosystem fails due to shortage of water and as a consequence of ecological distress, damages are induced in the ecosystem.
    Statement 3 is correct:
    ● Agricultural drought: It is also known as soil moisture drought, characterised by low soil moisture that is necessary to support the crops, thereby resulting in crop failures. Moreover, if an area has more than 30 percent of its gross cropped area under irrigation, the area is excluded from the drought-prone category.
    Statement 4 is correct:
    ● Hydrological drought: It results when the availability of water in different storages and reservoirs like aquifers, lakes, reservoirs, etc. falls below what the precipitation can replenish.

    Incorrect

    Answer: B
    Explanation:
    Statement 1 is incorrect:
    ● Meteorological drought: It is a situation when there is a prolonged period of inadequate rainfall marked with mal-distribution of the same over time and space.
    Statement 2 is incorrect:
    ● Ecological drought: When the productivity of a natural ecosystem fails due to shortage of water and as a consequence of ecological distress, damages are induced in the ecosystem.
    Statement 3 is correct:
    ● Agricultural drought: It is also known as soil moisture drought, characterised by low soil moisture that is necessary to support the crops, thereby resulting in crop failures. Moreover, if an area has more than 30 percent of its gross cropped area under irrigation, the area is excluded from the drought-prone category.
    Statement 4 is correct:
    ● Hydrological drought: It results when the availability of water in different storages and reservoirs like aquifers, lakes, reservoirs, etc. falls below what the precipitation can replenish.

window.wpAdvQuizInitList = window.wpAdvQuizInitList || []; window.wpAdvQuizInitList.push({ id: '#wpAdvQuiz_592', init: { quizId: 592, mode: 0, globalPoints: 10, timelimit: 0, resultsGrade: [0], bo: 0, qpp: 0, catPoints: [10], formPos: 0, lbn: "Finish quiz", json: {"2754":{"type":"single","id":2754,"catId":0,"points":2,"correct":[0,0,0,1]},"2755":{"type":"single","id":2755,"catId":0,"points":2,"correct":[1,0,0,0]},"2756":{"type":"single","id":2756,"catId":0,"points":2,"correct":[1,0,0,0]},"2757":{"type":"single","id":2757,"catId":0,"points":2,"correct":[0,0,0,1]},"2758":{"type":"single","id":2758,"catId":0,"points":2,"correct":[0,1,0,0]}} } });




Day-552 | Daily MCQs | UPSC Prelims | HISTORY

14-12-2023

Time limit: 0

Quiz-summary

0 of 5 questions completed

Questions:

  1. 1
  2. 2
  3. 3
  4. 4
  5. 5

Information

DAILY MCQS

You have already completed the quiz before. Hence you can not start it again.

Quiz is loading...

You must sign in or sign up to start the quiz.

You have to finish following quiz, to start this quiz:

Results

0 of 5 questions answered correctly

Your time:

Time has elapsed

You have reached 0 of 0 points, (0)

Categories

  1. GS Prelims 0%
  1. 1
  2. 2
  3. 3
  4. 4
  5. 5
  1. Answered
  2. Review
  1. Question 1 of 5
    1. Question

    1. With reference to the history of education in India, consider the following statements:

    1. William Jones who was appointed as judge of the Supreme Court took the initiative to set up Calcutta Madrasa.
    2. The English Education Act of 1835, introduced English as the medium of instruction at all levels of learning.

    Which of the statements given above is/are correct?

    Correct

    Answer: D
    Explanation:

    The early idea which helped in shaping the policies of education during colonial rule was Orientalism. It focused on studying and understanding the scholarly knowledge of the language and culture of Asia.

    Some of the early personalities who supported this idea were:

    • William Jones who had an appointment as a junior judge at the Supreme Court that the Company had set up at Calcutta. He along with some of his followers like Henry Thomas Colebrooke and Nathaniel Halhed, set up the Asiatic Society of Bengal, and started a journal called Asiatick Researches. So, statement 1 is incorrect.
    • Warren Hastings took the initiative to set up the Calcutta Madrasa in 1781 to promote the study of Arabic, Persian and Islamic law and believed that the ancient customs of the country and Oriental learning ought to be the basis of British rule in India.
    • From the early nineteenth century many British officials began to criticise the Orientalist vision of learning. The British effort ought to be to teach what was useful and practical. So Indians should be made familiar with the scientific and technical advances that the West had made, rather than with the poetry and sacred literature of the Orient.
    • The most influential ideologues of these were James Mill and Thomas Babington Macaulay. With great energy and passion, Macaulay emphasised the need to teach the English language.
    • Following Macaulay’s minute, the English Education Act of 1835 was introduced. The decision was to make English the medium of instruction for higher education, and to stop the promotion of Oriental institutions like the Calcutta Madrasa and Benaras Sanskrit College. So, statement 2 is incorrect.
    Incorrect

    Answer: D
    Explanation:

    The early idea which helped in shaping the policies of education during colonial rule was Orientalism. It focused on studying and understanding the scholarly knowledge of the language and culture of Asia.

    Some of the early personalities who supported this idea were:

    • William Jones who had an appointment as a junior judge at the Supreme Court that the Company had set up at Calcutta. He along with some of his followers like Henry Thomas Colebrooke and Nathaniel Halhed, set up the Asiatic Society of Bengal, and started a journal called Asiatick Researches. So, statement 1 is incorrect.
    • Warren Hastings took the initiative to set up the Calcutta Madrasa in 1781 to promote the study of Arabic, Persian and Islamic law and believed that the ancient customs of the country and Oriental learning ought to be the basis of British rule in India.
    • From the early nineteenth century many British officials began to criticise the Orientalist vision of learning. The British effort ought to be to teach what was useful and practical. So Indians should be made familiar with the scientific and technical advances that the West had made, rather than with the poetry and sacred literature of the Orient.
    • The most influential ideologues of these were James Mill and Thomas Babington Macaulay. With great energy and passion, Macaulay emphasised the need to teach the English language.
    • Following Macaulay’s minute, the English Education Act of 1835 was introduced. The decision was to make English the medium of instruction for higher education, and to stop the promotion of Oriental institutions like the Calcutta Madrasa and Benaras Sanskrit College. So, statement 2 is incorrect.
  2. Question 2 of 5
    2. Question

    2. With reference to the cultural history of India, the term ‘charak-puja’, refers to:

    Correct

    Answer: B

    Explanation:

    • The hook-swinging ritual (known as charak puja in Bengal) was a form of devotion to the female deity Mari-amma. The devotee would be beaten on the back by a priest until the flesh there was numbed, at which point metal hooks would be passed through the flesh. He would then be raised on the gibbet and swung around, taking care not to show any pain and even to shout and laugh.
    • In the vernaculars of the country “hook-swinging” is variously known as charak-puja, pota-puja, khidi-mari, bhokta-puja, chata-parab, soodaloo, and silloo.

    Incorrect

    Answer: B

    Explanation:

    • The hook-swinging ritual (known as charak puja in Bengal) was a form of devotion to the female deity Mari-amma. The devotee would be beaten on the back by a priest until the flesh there was numbed, at which point metal hooks would be passed through the flesh. He would then be raised on the gibbet and swung around, taking care not to show any pain and even to shout and laugh.
    • In the vernaculars of the country “hook-swinging” is variously known as charak-puja, pota-puja, khidi-mari, bhokta-puja, chata-parab, soodaloo, and silloo.

  3. Question 3 of 5
    3. Question

    3. Consider the following statements about Pandita Ramabai:

    1. Her book titled Stripurushtulna is the first known autobiography written by an Indian woman.
    2. She set up a Mission in Khedgaon near Pune in 1898, where widows and poor women were encouraged to become literate and independent.
    3. In one of the remarkable stories written by her, she talks about a place called ‘ladyland’ where women had freedom to study and work.

    How many statements given above are correct?

    Correct

    Answer: A

    Explanation:

    • Statement 1 is incorrect: Stripurushtulna (A Comparison between Women and Men) is authored by Tarabai Shinde from Poona criticising the social differences between men and women.
    • Amar Jiban written by Rashsundari Devi (1800–1890), who was born in West Bengal, is the first known autobiography written by an Indian woman.
    • Statement 2 is correct: Pandita Ramabai (1858–1922) was given the title ‘Pandita’ because she could read and write Sanskrit, a remarkable achievement as women then were not allowed such knowledge. She went on to set up a Mission in Khedgaon near Pune in 1898, where widows and poor women were encouraged not only to become literate but to be independent.
    • Statement 3 is incorrect: Ladyland is a part of a remarkable story written by Rokeya Sakhawat Hossain titled Sultana’s Dream in 1905 to practise her English skills. This story imagined a woman called Sultana who reaches a place called Ladyland. Ladyland is a place where women had the freedom to study, work, and create inventions like controlling rain from the clouds and flying air cars.
    Incorrect

    Answer: A

    Explanation:

    • Statement 1 is incorrect: Stripurushtulna (A Comparison between Women and Men) is authored by Tarabai Shinde from Poona criticising the social differences between men and women.
    • Amar Jiban written by Rashsundari Devi (1800–1890), who was born in West Bengal, is the first known autobiography written by an Indian woman.
    • Statement 2 is correct: Pandita Ramabai (1858–1922) was given the title ‘Pandita’ because she could read and write Sanskrit, a remarkable achievement as women then were not allowed such knowledge. She went on to set up a Mission in Khedgaon near Pune in 1898, where widows and poor women were encouraged not only to become literate but to be independent.
    • Statement 3 is incorrect: Ladyland is a part of a remarkable story written by Rokeya Sakhawat Hossain titled Sultana’s Dream in 1905 to practise her English skills. This story imagined a woman called Sultana who reaches a place called Ladyland. Ladyland is a place where women had the freedom to study, work, and create inventions like controlling rain from the clouds and flying air cars.
  4. Question 4 of 5
    4. Question

    4. Consider the following paragraph:

    He forged a link with the Muslim League, and decided to launch a civil disobedience movement to destroy the Holwell monument that stood in Calcutta as a reminder of a Black hole tragedy which most people believed did never happen and was invented only to tar the memory of Siraj-ud-daula, the last independent ruler of Bengal. But before it could start, he was arrested by the British on 3 July 1940 under the Defence of India Act.

    Which of the following personalities is being referred to in the above paragraph?

    Correct

    Answer: C

    Explanation:

    • When the 2nd World War broke out in Europe, Subhas Chandra Bose was of the view that Indians must utilise this opportunity to take advantage of the empire’s weakest moment to gain Independence.
    • In Bengal, he forged a link with the Muslim League, and decided to launch a civil disobedience movement to destroy the Holwell monument that stood in Calcutta as a reminder of a Black hole tragedy which most people believed did never happen and was invented only to tar the memory of Siraj-ud-daula, the last independent ruler of Bengal. It was a campaign that had an obvious appeal to the Muslims and thus could further strengthen the Hindu-Muslim pact in Bengal. But before it could start, he was arrested by the British on 3 July 1940 under the Defence of India Act.
    • The Holwell Monument was erected by G. Holwell during the short tenure of his Calcutta Governorship in 1760 to commemorate those deceased in the Black Hole tragedy. The monument was a symbol representing the alleged savagery of the last Nawab of Bengal, Siraj-ud-daula, and the bravery of the British soldiers who sacrificed their lives. It was argued, ‘the monument must go because it is not merely an unwarranted stain on the memory of the Nawab but has stood in the heart of Calcutta for the last 150 years or more as the symbol of our slavery and humiliation.’
    Incorrect

    Answer: C

    Explanation:

    • When the 2nd World War broke out in Europe, Subhas Chandra Bose was of the view that Indians must utilise this opportunity to take advantage of the empire’s weakest moment to gain Independence.
    • In Bengal, he forged a link with the Muslim League, and decided to launch a civil disobedience movement to destroy the Holwell monument that stood in Calcutta as a reminder of a Black hole tragedy which most people believed did never happen and was invented only to tar the memory of Siraj-ud-daula, the last independent ruler of Bengal. It was a campaign that had an obvious appeal to the Muslims and thus could further strengthen the Hindu-Muslim pact in Bengal. But before it could start, he was arrested by the British on 3 July 1940 under the Defence of India Act.
    • The Holwell Monument was erected by G. Holwell during the short tenure of his Calcutta Governorship in 1760 to commemorate those deceased in the Black Hole tragedy. The monument was a symbol representing the alleged savagery of the last Nawab of Bengal, Siraj-ud-daula, and the bravery of the British soldiers who sacrificed their lives. It was argued, ‘the monument must go because it is not merely an unwarranted stain on the memory of the Nawab but has stood in the heart of Calcutta for the last 150 years or more as the symbol of our slavery and humiliation.’
  5. Question 5 of 5
    5. Question

    5. With reference to the history of India, the term ‘dalams’ refers to:

    Correct

    Answer: D

    Explanation:

    • The term ‘dalams’ refers to revolutionary units. Mostly women played an active role in these organisations. They joined on their own, acted as couriers of secret messages, arranged shelter and few of them took up guns and became participating members of the dalams. One of the prominent examples of this was the Tebhaga movement which began in Bengal in 1946 under communist-led kisan sabhas with the sharecroppers’ demand for two-thirds share of the produce.
    Incorrect

    Answer: D

    Explanation:

    • The term ‘dalams’ refers to revolutionary units. Mostly women played an active role in these organisations. They joined on their own, acted as couriers of secret messages, arranged shelter and few of them took up guns and became participating members of the dalams. One of the prominent examples of this was the Tebhaga movement which began in Bengal in 1946 under communist-led kisan sabhas with the sharecroppers’ demand for two-thirds share of the produce.
window.wpAdvQuizInitList = window.wpAdvQuizInitList || []; window.wpAdvQuizInitList.push({ id: '#wpAdvQuiz_591', init: { quizId: 591, mode: 0, globalPoints: 10, timelimit: 0, resultsGrade: [0], bo: 0, qpp: 0, catPoints: {"2":10}, formPos: 0, lbn: "Finish quiz", json: {"2749":{"type":"single","id":2749,"catId":2,"points":2,"correct":[0,0,0,1]},"2750":{"type":"single","id":2750,"catId":2,"points":2,"correct":[0,1,0,0]},"2751":{"type":"single","id":2751,"catId":2,"points":2,"correct":[1,0,0,0]},"2752":{"type":"single","id":2752,"catId":2,"points":2,"correct":[0,0,1,0]},"2753":{"type":"single","id":2753,"catId":2,"points":2,"correct":[0,0,0,1]}} } });



Day-551 | Daily MCQs | UPSC Prelims | GENERAL SCIENCE AND TECHNOLOGICAL DEVELOPMENTS

Day-551

Time limit: 0

Quiz-summary

0 of 5 questions completed

Questions:

  1. 1
  2. 2
  3. 3
  4. 4
  5. 5

Information

DAILY MCQ

You have already completed the quiz before. Hence you can not start it again.

Quiz is loading...

You must sign in or sign up to start the quiz.

You have to finish following quiz, to start this quiz:

Results

0 of 5 questions answered correctly

Your time:

Time has elapsed

You have reached 0 of 0 points, (0)

Categories

  1. Not categorized 0%
  1. 1
  2. 2
  3. 3
  4. 4
  5. 5
  1. Answered
  2. Review
  1. Question 1 of 5
    1. Question

    1. With reference to the birth control pills, consider the following mechanisms:
    1. Stopping or reducing ovulation
    2. Thinning of the cervical mucus
    3. Thickening of uterus lining
    Birth control pills can target how many of the above mechanisms to prevent pregnancy in females?

    Correct

    Answer: A
    Explanation
    The birth control pill is a type of oral contraception that uses hormones to prevent pregnancy. The hormones in birth control pills prevent pregnancy by blocking conception when sperm fertilizes an egg. They also cause changes in the uterus so that it can’t support a pregnancy. Oral contraceptive pills are either combined estrogen-progesterone (also called combined oral contraceptive pill- COC) or progesterone-only pill (POP).
    ● Mechanism 1 is correct: One of the mechanisms targeted by oral contraceptives is to stop ovulation. Ovulation is the release of eggs from the ovaries of females. Thus, there is no egg for fertilization which prevents pregnancy.
    ● Mechanism 2 is incorrect: Thickening of cervical mucus is another mechanism. This creates a barrier that prevents sperm from entering the uterus and eventually reaching the egg.
    ● Mechanism 3 is incorrect: Thinning of uterus lining is also achieved through oral contraceptive pills. This prevents fertilized eggs from getting attached and growing into a fetus.

    Incorrect

    Answer: A
    Explanation
    The birth control pill is a type of oral contraception that uses hormones to prevent pregnancy. The hormones in birth control pills prevent pregnancy by blocking conception when sperm fertilizes an egg. They also cause changes in the uterus so that it can’t support a pregnancy. Oral contraceptive pills are either combined estrogen-progesterone (also called combined oral contraceptive pill- COC) or progesterone-only pill (POP).
    ● Mechanism 1 is correct: One of the mechanisms targeted by oral contraceptives is to stop ovulation. Ovulation is the release of eggs from the ovaries of females. Thus, there is no egg for fertilization which prevents pregnancy.
    ● Mechanism 2 is incorrect: Thickening of cervical mucus is another mechanism. This creates a barrier that prevents sperm from entering the uterus and eventually reaching the egg.
    ● Mechanism 3 is incorrect: Thinning of uterus lining is also achieved through oral contraceptive pills. This prevents fertilized eggs from getting attached and growing into a fetus.

  2. Question 2 of 5
    2. Question

    2. With reference to the Worldcoin project, consider the following statements:
    1. It aims to create one of the world’s largest cryptocurrency networks for digital finance.
    2. Fingerprint scan will be used for authentication and signup purposes on the network.
    3. It will be open to everyone regardless of his/her country.
    How many of the above statements are correct?

    Correct

    Answer: B
    Explanation
    A project called Worldcoin has been launched by OpenAI, an Artificial Intelligence company. The project claims to be building the world’s largest identity and financial public network.
    ● Statement 1 is correct: The project aims to introduce Worldcoin token as a new cryptocurrency that offers a new and unique method for identification to reduce the risk of fraud. Worldcoin token (WLD) can be used as a medium of purchases and transfers globally using digital assets and traditional currencies.
    ● Statement 2 is incorrect: Iris scan (eye) is essential to sign up to the network. It will be done through a ball-like object called an ‘orb’. Once the orb’s iris scan verifies the person is a real human, it creates a World ID for them. The biometric data would help differentiate between humans and Artificial Intelligence systems and prevent duplication of IDs from the same person. It can then be used as an ID in a variety of everyday applications – such as a cryptocurrency wallet – without revealing the user’s identity.
    ● Statement 3 is correct: The Worldcoin protocol is intended to be the world’s largest identity and financial public network, open to everyone regardless of their country, background, or economic status.

    Incorrect

    Answer: B
    Explanation
    A project called Worldcoin has been launched by OpenAI, an Artificial Intelligence company. The project claims to be building the world’s largest identity and financial public network.
    ● Statement 1 is correct: The project aims to introduce Worldcoin token as a new cryptocurrency that offers a new and unique method for identification to reduce the risk of fraud. Worldcoin token (WLD) can be used as a medium of purchases and transfers globally using digital assets and traditional currencies.
    ● Statement 2 is incorrect: Iris scan (eye) is essential to sign up to the network. It will be done through a ball-like object called an ‘orb’. Once the orb’s iris scan verifies the person is a real human, it creates a World ID for them. The biometric data would help differentiate between humans and Artificial Intelligence systems and prevent duplication of IDs from the same person. It can then be used as an ID in a variety of everyday applications – such as a cryptocurrency wallet – without revealing the user’s identity.
    ● Statement 3 is correct: The Worldcoin protocol is intended to be the world’s largest identity and financial public network, open to everyone regardless of their country, background, or economic status.

  3. Question 3 of 5
    3. Question

    3. With reference to the ‘Formation water’, consider the following statements:
    1. It appears during the mining of coal from underground.
    2. It has a very high salt content.
    3. It cannot be treated by any means.
    How many of the above statements are correct?

    Correct

    Answer: A
    Explanation
    ● Statement 1 is incorrect: Formation water is water that appears during the drilling process for oil and gas extraction. It is very dense and viscous.
    ● Statement 2 is correct: The Formation water consists of oily components, brine solutions, and solvents that are used during various phases in the oil industry. It has a high salt content which includes sodium, potassium, calcium, and magnesium salts.
    ● Statement 3 is incorrect: There are ways and means to treat formation water. It depends on the usage of the treated product water.
    Treatment of the formation water
    The process can be summed up in four stages:
    1. Separation: removing crude, oil and other contaminants in the water that can clog filters downstream.
    2. Suspended Solid Removal: eliminating small size particles remaining.
    3. Dissolved solid removal: using low- and high-pressure membranes.
    4. Conditioning: depending on the quality of the water it may be a post treatment station for disinfection or activated carbon filtration.
    More about formation water
    ● If formation water is released without treatment, it can lead to the degradation of water quality, posing a significant threat to aquatic life.
    ● The toxins in formation water can also bio-magnify into the human food chain, if fish and other aquatic products, harvested from the affected area, are consumed.

    Incorrect

    Answer: A
    Explanation
    ● Statement 1 is incorrect: Formation water is water that appears during the drilling process for oil and gas extraction. It is very dense and viscous.
    ● Statement 2 is correct: The Formation water consists of oily components, brine solutions, and solvents that are used during various phases in the oil industry. It has a high salt content which includes sodium, potassium, calcium, and magnesium salts.
    ● Statement 3 is incorrect: There are ways and means to treat formation water. It depends on the usage of the treated product water.
    Treatment of the formation water
    The process can be summed up in four stages:
    1. Separation: removing crude, oil and other contaminants in the water that can clog filters downstream.
    2. Suspended Solid Removal: eliminating small size particles remaining.
    3. Dissolved solid removal: using low- and high-pressure membranes.
    4. Conditioning: depending on the quality of the water it may be a post treatment station for disinfection or activated carbon filtration.
    More about formation water
    ● If formation water is released without treatment, it can lead to the degradation of water quality, posing a significant threat to aquatic life.
    ● The toxins in formation water can also bio-magnify into the human food chain, if fish and other aquatic products, harvested from the affected area, are consumed.

  4. Question 4 of 5
    4. Question

    4. The ‘Turing Test’ is related to which of the following scientific developments?

    Correct

    Answer: B
    Explanation
    ● The Turing Test is used in artificial intelligence (AI) for determining whether a computer is capable of thinking like a human being. The test is named after Alan Turing, the founder of the Turing Test and an English computer scientist, cryptanalyst, mathematician, and theoretical biologist.
    ● The original Turing Test requires three terminals, each of which is physically separated from the other two. One terminal is operated by a computer, while the other two are operated by humans.
    ● During the test, one of the human being functions as the questioner, while the second human and the computer function as respondents. The questioner interrogates the respondents within a specific subject area, using a specified format and context. After a preset length of time or number of questions, the questioner is then asked to decide which respondent was human and which was a computer.

    Incorrect

    Answer: B
    Explanation
    ● The Turing Test is used in artificial intelligence (AI) for determining whether a computer is capable of thinking like a human being. The test is named after Alan Turing, the founder of the Turing Test and an English computer scientist, cryptanalyst, mathematician, and theoretical biologist.
    ● The original Turing Test requires three terminals, each of which is physically separated from the other two. One terminal is operated by a computer, while the other two are operated by humans.
    ● During the test, one of the human being functions as the questioner, while the second human and the computer function as respondents. The questioner interrogates the respondents within a specific subject area, using a specified format and context. After a preset length of time or number of questions, the questioner is then asked to decide which respondent was human and which was a computer.

  5. Question 5 of 5
    5. Question

    5. Meissner Effect is related to which of the following technologies?

    Correct

    Answer: B
    Explanation
    ● Meissner effect is the expulsion of a magnetic field from the interior of a material that is in the process of becoming a superconductor.

    ● A super conductor loses its resistance to the flow of electrical currents when cooled below a certain temperature called the critical temperature. It is close to absolute zero or zero kelvin.
    ● The Meissner Effect is used in magnetic levitation, which means a body is suspended with no support except a magnetic field. Modern high-speed bullet trains use the phenomenon of magnetic levitation.

    Incorrect

    Answer: B
    Explanation
    ● Meissner effect is the expulsion of a magnetic field from the interior of a material that is in the process of becoming a superconductor.

    ● A super conductor loses its resistance to the flow of electrical currents when cooled below a certain temperature called the critical temperature. It is close to absolute zero or zero kelvin.
    ● The Meissner Effect is used in magnetic levitation, which means a body is suspended with no support except a magnetic field. Modern high-speed bullet trains use the phenomenon of magnetic levitation.

window.wpAdvQuizInitList = window.wpAdvQuizInitList || []; window.wpAdvQuizInitList.push({ id: '#wpAdvQuiz_590', init: { quizId: 590, mode: 0, globalPoints: 10, timelimit: 0, resultsGrade: [0], bo: 0, qpp: 0, catPoints: [10], formPos: 0, lbn: "Finish quiz", json: {"2744":{"type":"single","id":2744,"catId":0,"points":2,"correct":[1,0,0,0]},"2745":{"type":"single","id":2745,"catId":0,"points":2,"correct":[0,1,0,0]},"2746":{"type":"single","id":2746,"catId":0,"points":2,"correct":[1,0,0,0]},"2747":{"type":"single","id":2747,"catId":0,"points":2,"correct":[0,1,0,0]},"2748":{"type":"single","id":2748,"catId":0,"points":2,"correct":[0,1,0,0]}} } });




Day-550 | Daily MCQs | UPSC Prelims | GENERAL SCIENCE AND TECHNOLOGICAL DEVELOPMENTS

Day-550

Time limit: 0

Quiz-summary

0 of 5 questions completed

Questions:

  1. 1
  2. 2
  3. 3
  4. 4
  5. 5

Information

DAILY MCQ

You have already completed the quiz before. Hence you can not start it again.

Quiz is loading...

You must sign in or sign up to start the quiz.

You have to finish following quiz, to start this quiz:

Results

0 of 5 questions answered correctly

Your time:

Time has elapsed

You have reached 0 of 0 points, (0)

Categories

  1. Not categorized 0%
  1. 1
  2. 2
  3. 3
  4. 4
  5. 5
  1. Answered
  2. Review
  1. Question 1 of 5
    1. Question

    1. With reference to the extraction of oxygen, consider the following statements:
    1. Oxygen for medical purposes is commonly produced through distillation of air.
    2. Oxygen for use in space stations is produced through electrolysis of water.
    Which of the statements given above is/are correct?

    Correct

    Answer: C
    Explanation
    We know that 65% of the human body is oxygen. Oxygen is vital for respiration, the process that transfers energy from glucose to cells. In fact, every cell in our body requires oxygen. When we breathe air in, oxygen molecules enter the lungs and pass through the lung walls into our blood.
    ● Statement 1 is correct: There are various methods through which Liquid Medical Oxygen (LMO) can be produced. The most common production method is the separation of oxygen in Air Separation Units (ASUs). This is also known as Fractional Distillation method.
    Fractional Distillation method
    The various gases constituting atmospheric air are separated into different components after cooling them into a liquid state. Liquid oxygen is then extracted from it.
    Other Method – Pressure Swing Adsorption Technique
    This method leverages the property that under high pressure, gases tend to be attracted to solid surfaces. This process is known as adsorption. The higher the pressure, the more the adsorption of gas.
    ● Statement 2 is correct: Since there is no air in space, water is broken down using electricity in a process known as electrolysis of water. This generates 100% pure oxygen.

    Incorrect

    Answer: C
    Explanation
    We know that 65% of the human body is oxygen. Oxygen is vital for respiration, the process that transfers energy from glucose to cells. In fact, every cell in our body requires oxygen. When we breathe air in, oxygen molecules enter the lungs and pass through the lung walls into our blood.
    ● Statement 1 is correct: There are various methods through which Liquid Medical Oxygen (LMO) can be produced. The most common production method is the separation of oxygen in Air Separation Units (ASUs). This is also known as Fractional Distillation method.
    Fractional Distillation method
    The various gases constituting atmospheric air are separated into different components after cooling them into a liquid state. Liquid oxygen is then extracted from it.
    Other Method – Pressure Swing Adsorption Technique
    This method leverages the property that under high pressure, gases tend to be attracted to solid surfaces. This process is known as adsorption. The higher the pressure, the more the adsorption of gas.
    ● Statement 2 is correct: Since there is no air in space, water is broken down using electricity in a process known as electrolysis of water. This generates 100% pure oxygen.

  2. Question 2 of 5
    2. Question

    2. Consider the following:
    Statement I: A slow motion video camera has a higher frame rate.
    Statement II: Higher frame rate leads to smoother video quality.
    Which one of the following is correct in respect of the above statements?

    Correct

    Answer: A
    Explanation
    Frame rate is the number of individual video frames that your camera captures, per second. In video production, a video’s frame rate is expressed as frames per second (fps).
    Standard Frame Rates:
    24fps: Cinematic Standard
    30fps: Video Standard
    60fps: Slow-Motion Standard

    ● Statement 1 is correct: A slow motion camera has a frame rate of at least 60 fps. Ultra Slow-Motion Cameras, used in sports and movie production, have a very high frame rate of 1000 fps or even more.
    ● Statement 1 is correct and is the correct explanation of statement 1: Higher frame rate means more individual frames can be captured. This is used to smoothen the video as more subtle movements are better captured. This also reduces stuttering of videos and thus videos appear smoother at a higher frame rate. That is why a slow-motion camera has a higher frame rate.

    Incorrect

    Answer: A
    Explanation
    Frame rate is the number of individual video frames that your camera captures, per second. In video production, a video’s frame rate is expressed as frames per second (fps).
    Standard Frame Rates:
    24fps: Cinematic Standard
    30fps: Video Standard
    60fps: Slow-Motion Standard

    ● Statement 1 is correct: A slow motion camera has a frame rate of at least 60 fps. Ultra Slow-Motion Cameras, used in sports and movie production, have a very high frame rate of 1000 fps or even more.
    ● Statement 1 is correct and is the correct explanation of statement 1: Higher frame rate means more individual frames can be captured. This is used to smoothen the video as more subtle movements are better captured. This also reduces stuttering of videos and thus videos appear smoother at a higher frame rate. That is why a slow-motion camera has a higher frame rate.

  3. Question 3 of 5
    3. Question

    3. With reference to the Demon Particles, consider the following statements:
    1. They do not interact with the light and are transparent.
    2. They are chargeless as well as massless in nature.
    Which of the statements given above is/are correct?

    Correct

    Answer: C
    Explanation
    Recently researchers have discovered a unique particle, known as a “demon particle”, within a metal called Strontium Ruthenate. This discovery has the potential to develop superconductors capable of operating at room temperature. They are composite particles made up of a combination of electrons. Demon particles were first predicted by theoretical physicist David Pines in 1956.
    ● Statement 1 is correct: Demon Particles are transparent, so they are tough to be excited by photonic energy and do not interact with light.
    ● Statement 2 is correct: Demon particles are chargeless as well as massless particles. Due to these properties, they are not detected under the effects of electric or magnetic fields. Due to absence of interaction with light, electric or magnetic fields, they are termed Demon Particles.

    Incorrect

    Answer: C
    Explanation
    Recently researchers have discovered a unique particle, known as a “demon particle”, within a metal called Strontium Ruthenate. This discovery has the potential to develop superconductors capable of operating at room temperature. They are composite particles made up of a combination of electrons. Demon particles were first predicted by theoretical physicist David Pines in 1956.
    ● Statement 1 is correct: Demon Particles are transparent, so they are tough to be excited by photonic energy and do not interact with light.
    ● Statement 2 is correct: Demon particles are chargeless as well as massless particles. Due to these properties, they are not detected under the effects of electric or magnetic fields. Due to absence of interaction with light, electric or magnetic fields, they are termed Demon Particles.

  4. Question 4 of 5
    4. Question

    4. Wind Tunnel is related to which of the following activities?

    Correct

    Answer: A
    Explanation
    Wind tunnels are machines where an object is held stationary inside a tube, and air is blown around it to study the interaction between the object and the moving air. They are used to test the aerodynamic effects of aircraft, rockets, cars, and buildings. Most of the time, powerful fans move air through the tube. The object to be tested is fastened in the tunnel so that it will not move. The air moving around the still object shows what would happen if the object were moving through the air. How the air moves can be studied in different ways. Smoke or dye can be placed in the air and can be seen as it moves. Threads can be attached to the object to show how the air is moving. Special instruments are often used to measure the force of the air on the object.

    Incorrect

    Answer: A
    Explanation
    Wind tunnels are machines where an object is held stationary inside a tube, and air is blown around it to study the interaction between the object and the moving air. They are used to test the aerodynamic effects of aircraft, rockets, cars, and buildings. Most of the time, powerful fans move air through the tube. The object to be tested is fastened in the tunnel so that it will not move. The air moving around the still object shows what would happen if the object were moving through the air. How the air moves can be studied in different ways. Smoke or dye can be placed in the air and can be seen as it moves. Threads can be attached to the object to show how the air is moving. Special instruments are often used to measure the force of the air on the object.

  5. Question 5 of 5
    5. Question

    5. Which of the following statements correctly explains the purpose behind launching the ‘Red Line’ campaign by the Government of India?

    Correct

    Answer: C
    Explanation
    ● Ministry of Health and Family Welfare (MoHFW) launched Red Line awareness campaign in 2016 on Antimicrobials Resistance, urging people not to use medicines marked with a red vertical line, including antibiotics, without a doctor’s prescription.

    ● Indian Council of Medical Research (ICMR) has released treatment guidelines for antimicrobial use in common syndromes which contain guidelines for use of antibiotics for viral bronchitis and low-grade fever.
    ● Antibiotics are included in Schedule H and H1 of the Drugs Rules, 1945. These drugs have specific caution labeling requirements and are sold only under the prescription of a Registered Medical Practitioner.
    About Antimicrobial Resistance (AMR)
    AMR occurs when bacteria, viruses, fungi and parasites change over time and no longer respond to medicines making infections harder to treat and increasing the risk of disease spread, severe illness and death.
    Antimicrobials – including antibiotics, antivirals, antifungals and antiparasitics – are medicines used to prevent and treat infections in humans, animals, and plants. Microorganisms that develop antimicrobial resistance are sometimes referred to as “superbugs”.

    Incorrect

    Answer: C
    Explanation
    ● Ministry of Health and Family Welfare (MoHFW) launched Red Line awareness campaign in 2016 on Antimicrobials Resistance, urging people not to use medicines marked with a red vertical line, including antibiotics, without a doctor’s prescription.

    ● Indian Council of Medical Research (ICMR) has released treatment guidelines for antimicrobial use in common syndromes which contain guidelines for use of antibiotics for viral bronchitis and low-grade fever.
    ● Antibiotics are included in Schedule H and H1 of the Drugs Rules, 1945. These drugs have specific caution labeling requirements and are sold only under the prescription of a Registered Medical Practitioner.
    About Antimicrobial Resistance (AMR)
    AMR occurs when bacteria, viruses, fungi and parasites change over time and no longer respond to medicines making infections harder to treat and increasing the risk of disease spread, severe illness and death.
    Antimicrobials – including antibiotics, antivirals, antifungals and antiparasitics – are medicines used to prevent and treat infections in humans, animals, and plants. Microorganisms that develop antimicrobial resistance are sometimes referred to as “superbugs”.

window.wpAdvQuizInitList = window.wpAdvQuizInitList || []; window.wpAdvQuizInitList.push({ id: '#wpAdvQuiz_589', init: { quizId: 589, mode: 0, globalPoints: 10, timelimit: 0, resultsGrade: [0], bo: 0, qpp: 0, catPoints: [10], formPos: 0, lbn: "Finish quiz", json: {"2739":{"type":"single","id":2739,"catId":0,"points":2,"correct":[0,0,1,0]},"2740":{"type":"single","id":2740,"catId":0,"points":2,"correct":[1,0,0,0]},"2741":{"type":"single","id":2741,"catId":0,"points":2,"correct":[0,0,1,0]},"2742":{"type":"single","id":2742,"catId":0,"points":2,"correct":[1,0,0,0]},"2743":{"type":"single","id":2743,"catId":0,"points":2,"correct":[0,0,1,0]}} } });




Day-549 | Daily MCQs | UPSC Prelims | CURRENT DEVELOPMENTS

Day-549

Time limit: 0

Quiz-summary

0 of 5 questions completed

Questions:

  1. 1
  2. 2
  3. 3
  4. 4
  5. 5

Information

DAILY MCQ

You have already completed the quiz before. Hence you can not start it again.

Quiz is loading...

You must sign in or sign up to start the quiz.

You have to finish following quiz, to start this quiz:

Results

0 of 5 questions answered correctly

Your time:

Time has elapsed

You have reached 0 of 0 points, (0)

Categories

  1. Not categorized 0%
  1. 1
  2. 2
  3. 3
  4. 4
  5. 5
  1. Answered
  2. Review
  1. Question 1 of 5
    1. Question

    1. Consider the following statements:
    Statement I: Habitat rights are given to PVTGs under the Scheduled Tribes and Other Traditional Forest Dwellers (Recognition of Forest Rights) Act, 2006.
    Statement II: All the PVTGs in India have been given Habitat rights to safeguard and promote their traditional livelihood.
    Which one of the following is correct in respect of the above statements?

    Correct

    Answer: C
    Context: In October, 2023, the Chhattisgarh Government has granted habitat rights to its Baiga PVTG (Particularly Vulnerable Tribal Group) after the Kamar PVTG received habitat rights in the state.
    Explanation:
    Statement 1 is correct: Habitat rights are given to PVTGs under section 3(1) (e) [rights including community tenures of habitat and habitation for primitive tribal groups and pre-agricultural communities] of The Scheduled Tribes and Other Traditional Forest Dwellers (Recognition of Forest Rights) Act, 2006 also known as the Forest Rights Act (FRA).
    Statement 2 is incorrect: Out of 75 PVTG in India, only three have habitat rights. The Bharia PVTG in Madhya Pradesh was the first, followed by the Kamar tribe and now the Baiga tribe in Chhattisgarh.
    Additional information:
    ● Habitat rights recognition provides the community concerned rights over their customary territory of habitation.
    ● It includes socio-cultural practices, economic and livelihood means, intellectual knowledge of biodiversity and ecology, traditional knowledge of use of natural resources, as well as protection and conservation of their natural and cultural heritage.
    ● According to Section 2(h) of FRA, “Habitat includes the area comprising the customary habitat and such other habitats in reserved forests and protected forests of primitive tribal groups and pre-agricultural communities and other forest dwelling Scheduled Tribes.”
    ● Habitat rights safeguard and promote traditional livelihood and ecological knowledge passed down through generations.
    ● They also help converge different government schemes and initiatives from various departments to empower PVTG communities to develop their habitats.

    Incorrect

    Answer: C
    Context: In October, 2023, the Chhattisgarh Government has granted habitat rights to its Baiga PVTG (Particularly Vulnerable Tribal Group) after the Kamar PVTG received habitat rights in the state.
    Explanation:
    Statement 1 is correct: Habitat rights are given to PVTGs under section 3(1) (e) [rights including community tenures of habitat and habitation for primitive tribal groups and pre-agricultural communities] of The Scheduled Tribes and Other Traditional Forest Dwellers (Recognition of Forest Rights) Act, 2006 also known as the Forest Rights Act (FRA).
    Statement 2 is incorrect: Out of 75 PVTG in India, only three have habitat rights. The Bharia PVTG in Madhya Pradesh was the first, followed by the Kamar tribe and now the Baiga tribe in Chhattisgarh.
    Additional information:
    ● Habitat rights recognition provides the community concerned rights over their customary territory of habitation.
    ● It includes socio-cultural practices, economic and livelihood means, intellectual knowledge of biodiversity and ecology, traditional knowledge of use of natural resources, as well as protection and conservation of their natural and cultural heritage.
    ● According to Section 2(h) of FRA, “Habitat includes the area comprising the customary habitat and such other habitats in reserved forests and protected forests of primitive tribal groups and pre-agricultural communities and other forest dwelling Scheduled Tribes.”
    ● Habitat rights safeguard and promote traditional livelihood and ecological knowledge passed down through generations.
    ● They also help converge different government schemes and initiatives from various departments to empower PVTG communities to develop their habitats.

  2. Question 2 of 5
    2. Question

    2. Consider the following statements regarding ‘Reverse Book Building’:
    1. It involves removing of the securities of a listed company from a stock exchange through a fixed price mechanism.
    2. It is a process used for efficient price discovery.
    Which of the statements given above is/are correct?

    Correct

    Answer: B
    Context: The Securities and Exchange Board of India is exploring the possibility of permitting the delisting of companies through a fixed price mechanism instead of the existing reverse book-building procedure.
    Explanation:
    Statement 1 is incorrect: In this method, shareholders of the company place offers to sell their securities back to the promoters or large shareholders who hold significant influence over the company’s policies. There is no fixed price mechanism in this process.
    Statement 2 is correct: Reverse book building is a process used for efficient price discovery. During the period for which the reverse book-building is open, offers are collected from shareholders at various prices, which are above or equal to the floor price. The buyback price is determined after the offer closing price.
    Additional information:
    ● Delisting means removing the securities of a listed company from a stock exchange. Once delisted, the securities of that company can no longer be traded on the stock exchange.
    ● Delisting can be either voluntary or compulsory.
    1. In voluntary delisting, a company decides on its own to remove its securities from a stock exchange.
    2. In compulsory delisting, they are removed as a penal measure for the company not making submissions or complying with requirements set out in the listing agreement within the prescribed timeframes.
    How does Reverse Book-Building Work?
    ● Price Collection Phase: During the reverse book-building process, the company announces the intention to delist, specifying the floor price—the minimum price at which the company will buy back shares from the public shareholders. The company sets a specific period during which shareholders can submit their offers to sell their shares at various prices above or equal to the floor price.
    ● Gathering Offers: Shareholders, including retail and institutional investors, participate in the process by submitting their offers. They indicate the number of shares they are willing to sell and the respective price at which they want to sell them.
    ● Price Determination: At the end of the offer submission period, the company analyzes all the offers received and determines the final buyback price based on the demand and supply of shares at different price points. The objective is to arrive at a price that encourages a sufficient number of shareholders to participate and ensures a fair exit opportunity.

    Incorrect

    Answer: B
    Context: The Securities and Exchange Board of India is exploring the possibility of permitting the delisting of companies through a fixed price mechanism instead of the existing reverse book-building procedure.
    Explanation:
    Statement 1 is incorrect: In this method, shareholders of the company place offers to sell their securities back to the promoters or large shareholders who hold significant influence over the company’s policies. There is no fixed price mechanism in this process.
    Statement 2 is correct: Reverse book building is a process used for efficient price discovery. During the period for which the reverse book-building is open, offers are collected from shareholders at various prices, which are above or equal to the floor price. The buyback price is determined after the offer closing price.
    Additional information:
    ● Delisting means removing the securities of a listed company from a stock exchange. Once delisted, the securities of that company can no longer be traded on the stock exchange.
    ● Delisting can be either voluntary or compulsory.
    1. In voluntary delisting, a company decides on its own to remove its securities from a stock exchange.
    2. In compulsory delisting, they are removed as a penal measure for the company not making submissions or complying with requirements set out in the listing agreement within the prescribed timeframes.
    How does Reverse Book-Building Work?
    ● Price Collection Phase: During the reverse book-building process, the company announces the intention to delist, specifying the floor price—the minimum price at which the company will buy back shares from the public shareholders. The company sets a specific period during which shareholders can submit their offers to sell their shares at various prices above or equal to the floor price.
    ● Gathering Offers: Shareholders, including retail and institutional investors, participate in the process by submitting their offers. They indicate the number of shares they are willing to sell and the respective price at which they want to sell them.
    ● Price Determination: At the end of the offer submission period, the company analyzes all the offers received and determines the final buyback price based on the demand and supply of shares at different price points. The objective is to arrive at a price that encourages a sufficient number of shareholders to participate and ensures a fair exit opportunity.

  3. Question 3 of 5
    3. Question

    3. With reference to the ‘GIFT NIFTY’, consider the following statements:
    1. It is the first cross-border initiative connecting capital markets of India and the USA.
    2. Any trading member Indian or foreign setting up its office through subsidiary model can start trading in the GIFT Nifty products by taking membership of NSE IX.
    3. The GIFT NIFTY contracts are denominated in US dollars, which reduces the currency risk and conversion costs for international investors.
    How many of the statements given above are correct?

    Correct

    Answer: B
    Context: The launch of GIFT NIFTY, formerly known as SGX NIFTY, marked a new milestone in the collaboration between India and Singapore. It began trading at the Gujarat International Finance Tec-City (GIFT City) in July, 2023 attracting more than 30,000 trades in one session.
    Explanation:
    Statement 1 is incorrect: It is the first cross-border initiative in connecting India and Singapore’s capital markets. It is a new product that allows investors to trade futures and options contracts on the NIFTY 50 Index, which is the benchmark index of the National Stock Exchange of India (NSE), on the Singapore Exchange (SGX).
    Statement 2 is correct: Any trading member Indian or foreign setting up its office through subsidiary/branch model can start trading in the GIFT Nifty products by taking member ship of NSE IX. They can be either registered or non-registered.
    Statement 3 is correct: GIFT NIFTY contracts are denominated in US dollars, which reduces the currency risk and conversion costs for international investors.
    Additional information:
    ● The GIFT NIFTY stands for Gujarat International Finance Tec-City (GIFT) NIFTY, as the contracts are cleared and settled at the International Financial Services Centre (IFSC) in GIFT City, Gujarat, India.
    ● The IFSC is a special economic zone that offers various incentives and benefits to participants, such as lower taxes, relaxed regulations, and access to global markets.
    ● It is supported by the regulatory authorities of both countries, namely the Securities and Exchange Board of India (SEBI) and the Monetary Authority of Singapore (MAS).
    ● The GIFT NIFTY contracts are available for trading from 9:00 am to 11:55 pm Singapore time (6:30 am to 9:25 pm India time), which covers both the Indian and Singapore market hours, as well as some of the European and US market hours.
    ● The GIFT NIFTY contracts also have lower transaction fees and margin requirements than the existing NIFTY 50 contracts traded on the NSE.

    Incorrect

    Answer: B
    Context: The launch of GIFT NIFTY, formerly known as SGX NIFTY, marked a new milestone in the collaboration between India and Singapore. It began trading at the Gujarat International Finance Tec-City (GIFT City) in July, 2023 attracting more than 30,000 trades in one session.
    Explanation:
    Statement 1 is incorrect: It is the first cross-border initiative in connecting India and Singapore’s capital markets. It is a new product that allows investors to trade futures and options contracts on the NIFTY 50 Index, which is the benchmark index of the National Stock Exchange of India (NSE), on the Singapore Exchange (SGX).
    Statement 2 is correct: Any trading member Indian or foreign setting up its office through subsidiary/branch model can start trading in the GIFT Nifty products by taking member ship of NSE IX. They can be either registered or non-registered.
    Statement 3 is correct: GIFT NIFTY contracts are denominated in US dollars, which reduces the currency risk and conversion costs for international investors.
    Additional information:
    ● The GIFT NIFTY stands for Gujarat International Finance Tec-City (GIFT) NIFTY, as the contracts are cleared and settled at the International Financial Services Centre (IFSC) in GIFT City, Gujarat, India.
    ● The IFSC is a special economic zone that offers various incentives and benefits to participants, such as lower taxes, relaxed regulations, and access to global markets.
    ● It is supported by the regulatory authorities of both countries, namely the Securities and Exchange Board of India (SEBI) and the Monetary Authority of Singapore (MAS).
    ● The GIFT NIFTY contracts are available for trading from 9:00 am to 11:55 pm Singapore time (6:30 am to 9:25 pm India time), which covers both the Indian and Singapore market hours, as well as some of the European and US market hours.
    ● The GIFT NIFTY contracts also have lower transaction fees and margin requirements than the existing NIFTY 50 contracts traded on the NSE.

  4. Question 4 of 5
    4. Question

    4. Which of the following best describes the meaning of the term ‘’dotted land’’?

    Correct

    Answer: A
    Context: In May, 2023, the Andhra Pradesh government has started removing “dotted lands” in the state from the prohibited list, restoring full rights of selling or pledging these lands to the farmers who own them. Over 2 lakh acres of these British-era dotted lands have been identified for permanent denotification.
    Explanation:
    ● Dotted lands are disputed lands for which there are no clear ownership documents. Typically, one or more individuals as well as the government’s Revenue Department lay claim over the land.
    ● These lands came to be known as “dotted lands’’ during, land ownership surveys and resettlement of land records the British era.
    ● In British era local revenue officials who were tasked with identifying government-owned and privately-owned lands put dots in the ownership column if more than one person claimed ownership.
    How did these ownership disputes arise?
    ● This could happen if landowners did not leave clear wills passing on land to their heirs or children, and if a dispute arose because more than one heir lay claim over the land.
    ● Also, land could be deemed by the government to belong to the state but was under occupation by private parties.
    ● Some of the land records in question are more than 100 years old, and had been locked up in the prohibited list in and registers.
    ● During subsequent surveys, government officials left the ownership column blank indicating their disputed status as per Section 22A of the Registration Act.

    Incorrect

    Answer: A
    Context: In May, 2023, the Andhra Pradesh government has started removing “dotted lands” in the state from the prohibited list, restoring full rights of selling or pledging these lands to the farmers who own them. Over 2 lakh acres of these British-era dotted lands have been identified for permanent denotification.
    Explanation:
    ● Dotted lands are disputed lands for which there are no clear ownership documents. Typically, one or more individuals as well as the government’s Revenue Department lay claim over the land.
    ● These lands came to be known as “dotted lands’’ during, land ownership surveys and resettlement of land records the British era.
    ● In British era local revenue officials who were tasked with identifying government-owned and privately-owned lands put dots in the ownership column if more than one person claimed ownership.
    How did these ownership disputes arise?
    ● This could happen if landowners did not leave clear wills passing on land to their heirs or children, and if a dispute arose because more than one heir lay claim over the land.
    ● Also, land could be deemed by the government to belong to the state but was under occupation by private parties.
    ● Some of the land records in question are more than 100 years old, and had been locked up in the prohibited list in and registers.
    ● During subsequent surveys, government officials left the ownership column blank indicating their disputed status as per Section 22A of the Registration Act.

  5. Question 5 of 5
    5. Question

    5. Consider the following statements regarding Gandhi Peace Prize:
    1. It is only given to individuals who have worked selflessly for peace, non-violence and amelioration of human sufferings.
    2. The awardees do not get any cash reward, but a certificate signed by the President apart from a medallion which they can wear at public or government function.
    Which of the statements given above is/are correct?

    Correct

    Answer: D
    Context: In June 2023, the Gandhi prize was awarded to Gita Press for its ‘outstanding contribution towards social, economic and political transformation through non-violent and other Gandhian methods’.
    Explanation:
    Statement 1 is incorrect: This award is given to individuals, associations, institutions or organizations who have worked selflessly for peace, non-violence and amelioration of human sufferings.
    Statement 2 is incorrect: The award carries an amount of Rs. 1 crore, a citation, a plaque and an exquisite traditional handicraft/handloom item. The award is conferred by the President of India at a function in Rashtrapati Bhavan.
    Additional information:
    ● Gandhi Peace Prize is an annual award instituted by Government of India in 1995, on the occasion of 125th Birth Anniversary of Mahatma Gandhi as a tribute to the ideals espoused by Mahatma Gandhi.
    ● The award is open to all persons regardless of nationality, race, language, caste, creed or gender.
    ● The Award may be divided between two persons / institutions who are considered by the Jury to be equally deserving of recognition in a given year.
    ● Work by a person since deceased cannot be the subject of an Award. If, however, his death occurred subsequent to a proposal having been submitted to the Jury(headed by the Prime Minister) in the manner stipulated in the Code of Procedure of the Prize, then a Posthumous Award may be made.
    ● Established in 1923, Gita Press is one of the world’s largest publishers, having published 41.7 crore books in 14 languages, including 16.21 crore Shrimad Bhagvad Gita.
    ● The institution has never relied on advertisement in its publications, for revenue generation. Gita Press, along with its affiliated organizations, strives for the betterment of life and the wellbeing of all.

    Incorrect

    Answer: D
    Context: In June 2023, the Gandhi prize was awarded to Gita Press for its ‘outstanding contribution towards social, economic and political transformation through non-violent and other Gandhian methods’.
    Explanation:
    Statement 1 is incorrect: This award is given to individuals, associations, institutions or organizations who have worked selflessly for peace, non-violence and amelioration of human sufferings.
    Statement 2 is incorrect: The award carries an amount of Rs. 1 crore, a citation, a plaque and an exquisite traditional handicraft/handloom item. The award is conferred by the President of India at a function in Rashtrapati Bhavan.
    Additional information:
    ● Gandhi Peace Prize is an annual award instituted by Government of India in 1995, on the occasion of 125th Birth Anniversary of Mahatma Gandhi as a tribute to the ideals espoused by Mahatma Gandhi.
    ● The award is open to all persons regardless of nationality, race, language, caste, creed or gender.
    ● The Award may be divided between two persons / institutions who are considered by the Jury to be equally deserving of recognition in a given year.
    ● Work by a person since deceased cannot be the subject of an Award. If, however, his death occurred subsequent to a proposal having been submitted to the Jury(headed by the Prime Minister) in the manner stipulated in the Code of Procedure of the Prize, then a Posthumous Award may be made.
    ● Established in 1923, Gita Press is one of the world’s largest publishers, having published 41.7 crore books in 14 languages, including 16.21 crore Shrimad Bhagvad Gita.
    ● The institution has never relied on advertisement in its publications, for revenue generation. Gita Press, along with its affiliated organizations, strives for the betterment of life and the wellbeing of all.

window.wpAdvQuizInitList = window.wpAdvQuizInitList || []; window.wpAdvQuizInitList.push({ id: '#wpAdvQuiz_588', init: { quizId: 588, mode: 0, globalPoints: 10, timelimit: 0, resultsGrade: [0], bo: 0, qpp: 0, catPoints: [10], formPos: 0, lbn: "Finish quiz", json: {"2734":{"type":"single","id":2734,"catId":0,"points":2,"correct":[0,0,1,0]},"2735":{"type":"single","id":2735,"catId":0,"points":2,"correct":[0,1,0,0]},"2736":{"type":"single","id":2736,"catId":0,"points":2,"correct":[0,1,0,0]},"2737":{"type":"single","id":2737,"catId":0,"points":2,"correct":[1,0,0,0]},"2738":{"type":"single","id":2738,"catId":0,"points":2,"correct":[0,0,0,1]}} } });




Day-548 | Daily MCQs | UPSC Prelims | ECONOMY

Day-548

Time limit: 0

Quiz-summary

0 of 5 questions completed

Questions:

  1. 1
  2. 2
  3. 3
  4. 4
  5. 5

Information

DAILY MCQ

You have already completed the quiz before. Hence you can not start it again.

Quiz is loading...

You must sign in or sign up to start the quiz.

You have to finish following quiz, to start this quiz:

Results

0 of 5 questions answered correctly

Your time:

Time has elapsed

You have reached 0 of 0 points, (0)

Categories

  1. Not categorized 0%
  1. 1
  2. 2
  3. 3
  4. 4
  5. 5
  1. Answered
  2. Review
  1. Question 1 of 5
    1. Question

    1. Consider the following:
    1. Access to a larger and more liquid market
    2. Savings on the costs and time involved in the Initial Public Offering (IPO) process
    3. Avoidance of the dilution of ownership and control
    4. Exposure to the currency fluctuations and market volatility of the foreign exchange
    How many of the above are the possible benefits of direct listing of shares on a foreign stock exchange?

    Correct

    Answer: C
    Explanation:
    ● Direct listing is a process by which a company can list its shares on a foreign stock exchange without issuing new shares or raising capital from investors.
    ● Direct listing is different from the traditional Initial Public Offering (IPO), where a company sells a portion of its shares to the public and raises funds from investors.
    ● Direct listing is also different from the Depository Receipt (DR) route, where a company issues its shares to a custodian bank, which then issues DRs to foreign investors.
    Not a possible benefit of direct foreign listing:
    ● Exposure to the currency fluctuations and market volatility of the foreign exchange is one of the effects of direct foreign list. However, it can affect their share price and returns. So, it is a challenge not a benefit.
    Benefits of Direct Foreign Listing:
    ● Access to a larger and more liquid market, which can increase the demand and valuation of their shares.
    ● Savings on the costs and time involved in the IPO or DR process, such as underwriting fees, listing fees, legal fees, etc.
    ● Avoidance of the dilution of ownership and control that comes with issuing new shares or DRs.
    ● Ability to reach out to a wider and more sophisticated investor base, which can enhance their reputation and credibility.
    ● Startups and unicorns may benefit from this avenue of raising funds and increasing their global profile.
    ● Exposure to the best practices and regulations of the foreign jurisdiction can improve their governance and transparency.
    Additional information:
    How do Indian Companies Currently List on Foreign Exchanges?
    ● Currently, Indian companies list on foreign bourses using depository receipts, including American Depository Receipts (ADRs) and Global Depository Receipts (GDRs).
    ● To list on foreign stock exchanges, Indian companies entrust their shares to an Indian custodian, who then issues depository receipts (DRs) to foreign investors.
    ● Between 2008 and 2018, 109 companies raised over Rs 51,000 crore through ADRs/GDRs.
    ● However, after 2018, no Indian companies pursued overseas listings through this route.
    ● ADR refers to a negotiable certificate issued by a U.S. depositary bank representing a specified number of shares, usually one share of a foreign company’s stock.
    ● GDRs is a certificate issued by a depository bank that represents shares in a foreign company and deposits them in an account. GDRs are mostly traded on the European markets.
    Challenges Involved in Direct Foreign Listing:
    ● Compliance with the laws and rules of the foreign jurisdiction, which may be different from or more stringent than those in India.
    ● Challenges in direct foreign listings include valuation issues, as global investors may not offer the same valuations as in India, potentially impacting the company’s market perception and pricing.
    ● Potential conflicts or disputes with the existing shareholders, regulators, or tax authorities in India or abroad.
    ● Clarity is needed on which classes of public companies can use this route, the classes of securities that can be listed, the foreign jurisdictions and permitted stock exchanges for listing, and the exemptions offered to such companies in terms of procedural compliances.

    Incorrect

    Answer: C
    Explanation:
    ● Direct listing is a process by which a company can list its shares on a foreign stock exchange without issuing new shares or raising capital from investors.
    ● Direct listing is different from the traditional Initial Public Offering (IPO), where a company sells a portion of its shares to the public and raises funds from investors.
    ● Direct listing is also different from the Depository Receipt (DR) route, where a company issues its shares to a custodian bank, which then issues DRs to foreign investors.
    Not a possible benefit of direct foreign listing:
    ● Exposure to the currency fluctuations and market volatility of the foreign exchange is one of the effects of direct foreign list. However, it can affect their share price and returns. So, it is a challenge not a benefit.
    Benefits of Direct Foreign Listing:
    ● Access to a larger and more liquid market, which can increase the demand and valuation of their shares.
    ● Savings on the costs and time involved in the IPO or DR process, such as underwriting fees, listing fees, legal fees, etc.
    ● Avoidance of the dilution of ownership and control that comes with issuing new shares or DRs.
    ● Ability to reach out to a wider and more sophisticated investor base, which can enhance their reputation and credibility.
    ● Startups and unicorns may benefit from this avenue of raising funds and increasing their global profile.
    ● Exposure to the best practices and regulations of the foreign jurisdiction can improve their governance and transparency.
    Additional information:
    How do Indian Companies Currently List on Foreign Exchanges?
    ● Currently, Indian companies list on foreign bourses using depository receipts, including American Depository Receipts (ADRs) and Global Depository Receipts (GDRs).
    ● To list on foreign stock exchanges, Indian companies entrust their shares to an Indian custodian, who then issues depository receipts (DRs) to foreign investors.
    ● Between 2008 and 2018, 109 companies raised over Rs 51,000 crore through ADRs/GDRs.
    ● However, after 2018, no Indian companies pursued overseas listings through this route.
    ● ADR refers to a negotiable certificate issued by a U.S. depositary bank representing a specified number of shares, usually one share of a foreign company’s stock.
    ● GDRs is a certificate issued by a depository bank that represents shares in a foreign company and deposits them in an account. GDRs are mostly traded on the European markets.
    Challenges Involved in Direct Foreign Listing:
    ● Compliance with the laws and rules of the foreign jurisdiction, which may be different from or more stringent than those in India.
    ● Challenges in direct foreign listings include valuation issues, as global investors may not offer the same valuations as in India, potentially impacting the company’s market perception and pricing.
    ● Potential conflicts or disputes with the existing shareholders, regulators, or tax authorities in India or abroad.
    ● Clarity is needed on which classes of public companies can use this route, the classes of securities that can be listed, the foreign jurisdictions and permitted stock exchanges for listing, and the exemptions offered to such companies in terms of procedural compliances.

  2. Question 2 of 5
    2. Question

    2. Consider the following statements about the Index of Industrial Production (IIP):
    1. It measures the short-term changes in the volume of production of a basket of industrial products.
    2. The Central Statistical Organization (CSO) computes and publishes the IIP index on a quarterly basis.
    3. The current base year for the IIP series in India is 2011-12.
    How many of the statements given above are correct?

    Correct

    Answer: B
    Explanation:
    Statement 1 is correct: The IIP measures the short-term changes in the volume of production of a basket of industrial products.
    Statement 2 is incorrect: The Central Statistical Organization (CSO) computes and publishes the IIP index on a monthly basis.
    Statement 3 is correct: The current base year for the IIP series in India is 2011-12.
    Additional information:
    ● IIP is an important composite indicator in India that measures the changes in the volume of production of a basket of industrial products. IIP measures the growth of manufacturing, mining, and electricity sectors.
    ● The aim of IIP is to capture the direction and the trend of industrial production in the country, not the absolute value of industrial production.
    ● This index gives the growth rates of different industry groups of the economy over a specified time period.
    ● The eight core industries of India represent about 40% of the weight of items that are included in the IIP. The Eight Core Sectors/Industries are: Electricity; Steel; Refinery products; Crude oil; Coal; Cement; Natural gas; Fertilizers.
    ● The industry groups that it measures are classified under the following:
    1. Broad sectors like manufacturing, mining, and electricity.
    2. Use-based sectors like capital goods, basic goods, intermediate goods, infrastructure goods, consumer durables, and consumer non-durables.

    Incorrect

    Answer: B
    Explanation:
    Statement 1 is correct: The IIP measures the short-term changes in the volume of production of a basket of industrial products.
    Statement 2 is incorrect: The Central Statistical Organization (CSO) computes and publishes the IIP index on a monthly basis.
    Statement 3 is correct: The current base year for the IIP series in India is 2011-12.
    Additional information:
    ● IIP is an important composite indicator in India that measures the changes in the volume of production of a basket of industrial products. IIP measures the growth of manufacturing, mining, and electricity sectors.
    ● The aim of IIP is to capture the direction and the trend of industrial production in the country, not the absolute value of industrial production.
    ● This index gives the growth rates of different industry groups of the economy over a specified time period.
    ● The eight core industries of India represent about 40% of the weight of items that are included in the IIP. The Eight Core Sectors/Industries are: Electricity; Steel; Refinery products; Crude oil; Coal; Cement; Natural gas; Fertilizers.
    ● The industry groups that it measures are classified under the following:
    1. Broad sectors like manufacturing, mining, and electricity.
    2. Use-based sectors like capital goods, basic goods, intermediate goods, infrastructure goods, consumer durables, and consumer non-durables.

  3. Question 3 of 5
    3. Question

    3. With reference to the Angel tax in India, consider the following statements:
    1. Angel tax is an income tax, applicable only on unlisted companies.
    2. Angel tax can be imposed only on investments made by a resident investor.
    3. All government recognised startups are exempt from Angel tax.
    How many of the statements given above are correct?

    Correct

    Answer: A
    Explanation:
    ● Statement 1 is correct: Angel Tax is a term basically used to refer to the income tax payable on the capital raised by unlisted companies via the issue of shares through off-market transactions.
    ● Statement 2 is incorrect: Before budget 2023-24, angel tax was imposed only on investments made by a resident investor. i.e., it was not applicable in case the investments are made by any non-resident or venture capital funds. With Budget 2023-24, the government has decided to include foreign investors in the ambit. That means when a start-up raises funding from a foreign investor, that too will now be counted as income and be taxable. However, these foreign investors will not need to pay any angel tax while investing in a government-recognised startup in India similar to the provision for domestic investors.
    ● Statement 3 is incorrect: All government recognised startups are not exempt from the Angel Tax. It is only those start-ups which meet certain criteria are exempted from this tax. Moreover, the Central Board of Direct Taxes (CBDT) has clarified that government recognised startups will not be scrutinized for the changes brought about by the Budget 2023-24.
    Additional information:
    ● Angel tax is levied on the capital raised via the issue of shares by unlisted companies from an Indian investor/ foreign investor from certain countries if the share price of issued shares is seen in excess of the fair market value of the company.
    ● Rule related to Angel Tax is described in Section 56(2)(viib) of the Income Tax Act, 1961.
    ● This clause was inserted into the act in 2012 to prevent laundering of black money, round-tripping via investments with a large premium into unlisted companies.
    ● Allaying the concerns of the startup community, the govt had also exempted investments made by the domestic investors in companies approved by an inter-ministerial panel from Angel Tax.
    ● Government recognised startups, upon meeting certain criteria are exempted from this tax.
    ● Currently, angel tax is levied at the rate of 30.6 per cent.

    Incorrect

    Answer: A
    Explanation:
    ● Statement 1 is correct: Angel Tax is a term basically used to refer to the income tax payable on the capital raised by unlisted companies via the issue of shares through off-market transactions.
    ● Statement 2 is incorrect: Before budget 2023-24, angel tax was imposed only on investments made by a resident investor. i.e., it was not applicable in case the investments are made by any non-resident or venture capital funds. With Budget 2023-24, the government has decided to include foreign investors in the ambit. That means when a start-up raises funding from a foreign investor, that too will now be counted as income and be taxable. However, these foreign investors will not need to pay any angel tax while investing in a government-recognised startup in India similar to the provision for domestic investors.
    ● Statement 3 is incorrect: All government recognised startups are not exempt from the Angel Tax. It is only those start-ups which meet certain criteria are exempted from this tax. Moreover, the Central Board of Direct Taxes (CBDT) has clarified that government recognised startups will not be scrutinized for the changes brought about by the Budget 2023-24.
    Additional information:
    ● Angel tax is levied on the capital raised via the issue of shares by unlisted companies from an Indian investor/ foreign investor from certain countries if the share price of issued shares is seen in excess of the fair market value of the company.
    ● Rule related to Angel Tax is described in Section 56(2)(viib) of the Income Tax Act, 1961.
    ● This clause was inserted into the act in 2012 to prevent laundering of black money, round-tripping via investments with a large premium into unlisted companies.
    ● Allaying the concerns of the startup community, the govt had also exempted investments made by the domestic investors in companies approved by an inter-ministerial panel from Angel Tax.
    ● Government recognised startups, upon meeting certain criteria are exempted from this tax.
    ● Currently, angel tax is levied at the rate of 30.6 per cent.

  4. Question 4 of 5
    4. Question

    4. Which one of the following statements about the Sukanya Samriddhi Scheme is incorrect?

    Correct

    Answer: A
    Explanation:
    Statement 1 is incorrect: Parents or legal guardians can open deposits on behalf of a girl child (including adopted girl child) for up to 2 daughters aged below 10.
    Statement 2 is correct: Only one account can be opened in the name of a girl child.
    Statement 3 is correct: Maximum period up to which deposits can be made 15 years from the date of opening of the account.
    Statement 4 is correct: Interest that accrues against this account which gets compounded annually is also exempt from tax under Section 10 of the Income Tax Act.
    Additional information:
    ● Sukanya Samriddhi Yojana (SSY) Scheme was Introduced in 2016, the Sukanya Samriddhi Yojana Account (SSA) is a central government scheme aimed to cater to a girl child.
    ● The Sukanya Samriddhi Yojana is a government savings scheme created with the intention to benefit girl child under the initiative called “Beti Bachao – Beti Padhao”.
    ● This scheme carries a higher interest rate along with several tax benefits.
    ● The principal amount deposited, interest earned during the entire tenure and maturity benefits are tax-exempt. The principal amount is deductible under section 80C up to Rs 1.5 lakh.
    ● The proceeds received upon maturity/withdrawal are also exempt from income tax.
    ● The account matures after 21 years of opening the account or in the event of the marriage of the girl child after she gains the age of 18 years.
    ● A premature withdrawal of up to 50% of the investment is allowed after the child gains the age of 18 years even if she is not getting married.
    ● Three girl children, in case of twin girls as second birth or the first birth itself results in three girl children.
    ● Minimum of Rs 250 of initial deposit with multiple of Rs 150 thereafter with annual ceiling of Rs.150000 in a financial year.
    ● The account shall mature on completion of 21 years from the date of opening of account or on the marriage of Account holder whichever is earlier.

    Incorrect

    Answer: A
    Explanation:
    Statement 1 is incorrect: Parents or legal guardians can open deposits on behalf of a girl child (including adopted girl child) for up to 2 daughters aged below 10.
    Statement 2 is correct: Only one account can be opened in the name of a girl child.
    Statement 3 is correct: Maximum period up to which deposits can be made 15 years from the date of opening of the account.
    Statement 4 is correct: Interest that accrues against this account which gets compounded annually is also exempt from tax under Section 10 of the Income Tax Act.
    Additional information:
    ● Sukanya Samriddhi Yojana (SSY) Scheme was Introduced in 2016, the Sukanya Samriddhi Yojana Account (SSA) is a central government scheme aimed to cater to a girl child.
    ● The Sukanya Samriddhi Yojana is a government savings scheme created with the intention to benefit girl child under the initiative called “Beti Bachao – Beti Padhao”.
    ● This scheme carries a higher interest rate along with several tax benefits.
    ● The principal amount deposited, interest earned during the entire tenure and maturity benefits are tax-exempt. The principal amount is deductible under section 80C up to Rs 1.5 lakh.
    ● The proceeds received upon maturity/withdrawal are also exempt from income tax.
    ● The account matures after 21 years of opening the account or in the event of the marriage of the girl child after she gains the age of 18 years.
    ● A premature withdrawal of up to 50% of the investment is allowed after the child gains the age of 18 years even if she is not getting married.
    ● Three girl children, in case of twin girls as second birth or the first birth itself results in three girl children.
    ● Minimum of Rs 250 of initial deposit with multiple of Rs 150 thereafter with annual ceiling of Rs.150000 in a financial year.
    ● The account shall mature on completion of 21 years from the date of opening of account or on the marriage of Account holder whichever is earlier.

  5. Question 5 of 5
    5. Question

    5. Consider the following statements:
    1. When the RBI buys government securities, it can increase borrowing costs in the market.
    2. When the RBI sells government securities, it can depreciate the rupees.
    Which of the statements given above is/are correct?

    Correct

    Answer: D
    Explanation:
    Statement 1 is incorrect: Open Market Operations (OMOs) have a significant impact on interest rates. When the RBI buys government securities, it injects liquidity in the market. Banks have excess funds to lend to borrowers. This reduces the cost of borrowing in the market.
    Statement 2 is incorrect: OMOs can also influence the exchange rate of the Indian rupee. When the RBI sells government securities, it receives rupees from buyers, reducing the supply of rupees in the market and putting upward pressure on the exchange rate, making the rupee stronger.
    Additional information:
    ● Open market operations (OMOs) are one of the tools that the Reserve Bank of India (RBI) uses to regulate the money supply and liquidity conditions in the economy.
    ● OMOs are the sale and purchase of government securities (G-Secs) and treasury bills (T-Bills) by the RBI in the open market. When RBI wants to inject liquidity into the system, it buys G-Secs and T-Bills from the market, thereby increasing the money supply.
    ● Conversely, when RBI wants to absorb excess liquidity from the system, it sells G-Secs and T-Bills to the market, thereby reducing the money supply.
    ● OMO plays a crucial role in ensuring consistent liquidity availability throughout the year, thus maintaining desired interest rate levels.
    ● Regular OMO activities are conducted to strike a balance between controlling inflation and ensuring that banks maintain sufficient funds for lending.
    ● Factors contributing to this tightening include reduced government expenditure, heightened currency demand during the festive season, and fluctuations in foreign institutional investments and tax payments.

    Incorrect

    Answer: D
    Explanation:
    Statement 1 is incorrect: Open Market Operations (OMOs) have a significant impact on interest rates. When the RBI buys government securities, it injects liquidity in the market. Banks have excess funds to lend to borrowers. This reduces the cost of borrowing in the market.
    Statement 2 is incorrect: OMOs can also influence the exchange rate of the Indian rupee. When the RBI sells government securities, it receives rupees from buyers, reducing the supply of rupees in the market and putting upward pressure on the exchange rate, making the rupee stronger.
    Additional information:
    ● Open market operations (OMOs) are one of the tools that the Reserve Bank of India (RBI) uses to regulate the money supply and liquidity conditions in the economy.
    ● OMOs are the sale and purchase of government securities (G-Secs) and treasury bills (T-Bills) by the RBI in the open market. When RBI wants to inject liquidity into the system, it buys G-Secs and T-Bills from the market, thereby increasing the money supply.
    ● Conversely, when RBI wants to absorb excess liquidity from the system, it sells G-Secs and T-Bills to the market, thereby reducing the money supply.
    ● OMO plays a crucial role in ensuring consistent liquidity availability throughout the year, thus maintaining desired interest rate levels.
    ● Regular OMO activities are conducted to strike a balance between controlling inflation and ensuring that banks maintain sufficient funds for lending.
    ● Factors contributing to this tightening include reduced government expenditure, heightened currency demand during the festive season, and fluctuations in foreign institutional investments and tax payments.

window.wpAdvQuizInitList = window.wpAdvQuizInitList || []; window.wpAdvQuizInitList.push({ id: '#wpAdvQuiz_587', init: { quizId: 587, mode: 0, globalPoints: 10, timelimit: 0, resultsGrade: [0], bo: 0, qpp: 0, catPoints: [10], formPos: 0, lbn: "Finish quiz", json: {"2729":{"type":"single","id":2729,"catId":0,"points":2,"correct":[0,0,1,0]},"2730":{"type":"single","id":2730,"catId":0,"points":2,"correct":[0,1,0,0]},"2731":{"type":"single","id":2731,"catId":0,"points":2,"correct":[1,0,0,0]},"2732":{"type":"single","id":2732,"catId":0,"points":2,"correct":[1,0,0,0]},"2733":{"type":"single","id":2733,"catId":0,"points":2,"correct":[0,0,0,1]}} } });